Re: [obm-l] Determinante de uma Matriz

2020-03-13 Por tôpico Ralph Teixeira
Sim, determinante eh algo um pouco "estranho" sim inicialmente, nao eh um
conceito tao natural quanto outros que se apresentam no ensino medio.

Mas dou aqui algumas dicas de como pensar nele inicialmente:

1. UMA ABORDAGEM ALGEBRICA
1a. Caso 2x2.
Ao resolver o sistema linear:
ax+by=A
cx+dy=B
voce obtem **tentativamente**
x=(Ad-Bb)/(ad-bc)
y=(-Ac+Ba)/(ad-bc)

Digo "tentativamente" pois, se ad-bc=0, o que eu escrevi estah errado. Eu
devia ter dito o seguinte:

i) Se ad-bc<>0, entao a unica solucao do sistema eh aquele x e y ali em
cima;
ii) Se ad-bc=0... bom, depende dos outros caras, mas em suma o sistema vai
ser IMPOSSIVEL ou INDETERMINADO.

Em suma, o valor (0 ou nao 0) de ad-bc diz sozinho se o sistema tem uma
unica solucao ou nao.  Compare isso com o b^2-4ac da quadratica, que diz
sozinho quantas raizes a quadratica tem! Da mesma forma que chamamos
DELTA=b^2-4ac e analisamos esse cara para entender melhor a quadratica,
vamos chamar det([a,b;c,d])=ad-bc, uma especie de "discriminante" do
sistema linear... Ok, mas o nome oficial eh DETERMINANTE da MATRIZ
[a,b;c,d].

1b. Caso nxn.
Ao resolver o sistema linear
Mx=b
onde M eh uma matriz nxn, x eh um vetor (incognita) nx1 e b eh um
vetor (dado) nx1, nota-se que este sistema tem raiz unica quando uma certa
quantidade (que depende apenas de M, nao de b -- surpreendente, nao?) NAO
vale 0. Esta quantidade eh o DETERMINANTE da matriz M, e infelizmente tem
uma expressao feiosa quando n eh grande...

Em suma: o determinante de uma matriz M diz se o sistema Mx=b tem
solucao unica ou nao

(Agora eu teria que convencer voce que sistemas lineares sao relevantes
Bom, deixa eu dizer que SAO. :D)

---///---
Mas tem gente que acha isso algebrico demais. Vamos tentar algo mais
geometrico!

2. UMA ABORDAGEM GEOMETRICA
2a. Caso 2x2.
Considere um paralelogramos cujos lados sao os vetores v=(a,b) e w=(c,d).
Qual a area deste paralelogramo?

Se voce fizer a conta, voce descobre que a area eh ad-bc (ou o negativo
disto, depende da ordem dos vetores). Puxa, entao talvez seja legal definir
uma especie de "area com sinal" dada por:

AREA COM SINAL = [v,w] = ad-bc

Ou, que tal chamar isto de DETERMINANTE da matriz cujas colunas sao v e w?
Ok, feito!

(O sinal pode ser explicado com a regra da mao direita Mas deixa eu
ficar no basicao, acho que voce soh quer o SIGNIFICADO, nao detalhes.)

Em suma, o DETERMINANTE de uma matriz 2x2 eh a AREA (com sinal) do
paralelogramo cujos lados sao equivalentes aos vetores colunas da matriz.

2b. Caso 3x3.
Considere um paralelepipedo P, 3-dimensional, cujos lados sao os vetores
v1, v2 e v3. Vamos DEFINIR o determinante da matriz cujas colunas sao v1,
v2 e v3 como sendo o VOLUME desse paralelepipedo P (com sinal, que depnde
da orientacao de v1/v2/v3). Tem gente que ateh escreve assim:

det(v1,v2,v3) = expressao horrorosa envolvendo as coordenadas dos 3 vetores
= volume (P) (com sinal)

como se o determinante fosse uma funcao de 3 vetores ao inves de ser de uma
matriz...

2n. Caso nxn
Sejam v1, v2, ..., vn vetores com n coordenadas cada. O determinante da
matriz cujas colunas sao v1, v2, ..., vn pode ser DEFINIDO como o volume do
paralelepipedo cujas arestas sao v1, v2, ..., vn respectivamente (com sinal
que depende da orientacao, mas a discussao sobre o que significa
"orientacao" deixo para depois; com certeza, o volume eh o MODULO desse
determinante).

Ficou um pouco mais natural assim? Dah ateh para enxergar algumas das
propriedades basicas pensando assim. Por exemplo, se dois dos vetores forem
paralelos, o seu paralelepipedo fica "achatado" e portanto o volume eh 0 --
ou seja, se duas das colunas forem multiplas uma da outra, o determinante
eh 0.

Abraco, Ralph.

On Fri, Mar 13, 2020 at 11:30 AM Luiz Antonio Rodrigues <
rodrigue...@gmail.com> wrote:

> Olá, pessoal!
> Tudo bem?
> Há bastante tempo eu venho fazendo pesquisas sobre o significado do
> determinante de uma matriz.
> Livros, professores, internet...
> Não adianta...
> Parece que o determinante de uma matriz é algo nebuloso...
> E o cálculo de um determinante é mais misterioso ainda...
> Parece maluquice.
> Alguém já leu ou ouviu algo interessante sobre isso?
> Muito obrigado!
> Abraços!
> Luiz
>
> --
> Esta mensagem foi verificada pelo sistema de antivírus e
> acredita-se estar livre de perigo.

-- 
Esta mensagem foi verificada pelo sistema de antiv�rus e
 acredita-se estar livre de perigo.



Re: [obm-l] Zero Elevado a Zero

2020-02-16 Por tôpico Ralph Teixeira
Curiosidade: na calculadora do Google, e também na calculadora padrão do
Windows 10, 0^0=1.

Em contrapartida: o Wolfram Alpha diz "undefined".

Abraco, Ralph.

On Sun, Feb 16, 2020 at 4:19 PM Luiz Antonio Rodrigues <
rodrigue...@gmail.com> wrote:

> Olá, Bernardo!
> Olá, Artur!
> Muito obrigado pela resposta.
> Eu vou acessar o link para ver a argumentação do Ralph, que eu desconheço.
> Eu sei que é uma discussão meio "inútil", mas gosto desse tipo de troca de
> ideias.
> Acho que aprendo muito!
> Principalmente porque esse era um assunto "resolvido" para mim...
> Não tinha dúvidas quanto a isso e agora vejo que não é bem assim...
> Abraços!
> Luiz
>
>
> Em sáb, 15 de fev de 2020 9:22 PM, Artur Costa Steiner <
> steinerar...@gmail.com> escreveu:
>
>> É inútil discutir o valoe de 0^0. Não há como provar nada com relação a
>> isso. Comumente se define que 0^0 =1 porque esta é uma definição
>> conveniente. Por exemplo, em séries de potências.
>>
>> Artur
>>
>> Em sáb, 15 de fev de 2020 20:55, Luiz Antonio Rodrigues <
>> rodrigue...@gmail.com> escreveu:
>>
>>> Olá, pessoal!
>>> Tudo bem?
>>> Eu aprendi que qualquer número elevado a zero é 1, mas com exceção do
>>> zero.
>>> Também aprendi que 0^0, assim como 0/0, representam indeterminações.
>>> Na minha calculadora científica, a operação 0^0 resulta em erro.
>>> Acontece que há pouco tempo eu vi num livro que era utilizado num
>>> conceituado colégio de São Paulo que 0^0=1.
>>> Resolvi consultar a internet e descobri que essa discussão já existia no
>>> tempo em que Euler era vivo...
>>> Um dos sites que eu consultei diz que existem três respostas possíveis,
>>> dependendo do contexto:
>>> a) 0^0 é inexistente
>>> b) 0^0 é indeterminado
>>> c) 0^0=1
>>> Não fiquei convencido e acho que o item (b) é o correto, da forma como
>>> eu aprendi.
>>> O que vocês pensam sobre isso?
>>> Abraços!
>>> Luiz
>>>
>>> --
>>> Esta mensagem foi verificada pelo sistema de antivírus e
>>> acredita-se estar livre de perigo.
>>
>>
>> --
>> Esta mensagem foi verificada pelo sistema de antivírus e
>> acredita-se estar livre de perigo.
>
>
> --
> Esta mensagem foi verificada pelo sistema de antivírus e
> acredita-se estar livre de perigo.

-- 
Esta mensagem foi verificada pelo sistema de antiv�rus e
 acredita-se estar livre de perigo.



[obm-l] Re: [obm-l] Re: [obm-l] Re: [obm-l] Cálculo do Volume de um Sólido

2020-02-12 Por tôpico Ralph Teixeira
Vamos fixar um z (entre 0 e 2) para desenhar a seção horizontal. Como
x+y=z^2 e x+y=2z são duas retas paralelas, a seção horizontal é um trapézio
mais ou menos assim:

|\
| \
|  \
|   \
|\
 \\
  \\

As retas inclinadas são x+y=z^2, e x+y=2z. A reta vertical é o eixo y entre
z^2 e z, e a horizontal é o eixo x entre z^2 e z.

Então o problema na sua integral é que nem sempre o x vai de z^2-y até
2z-y, depende do valor de y! Trace uma vareta horizontal atravessando o
trapézio:

-- Na parte "de baixo" do trapézio, a vareta fura o trapézio nas duas retas
inclinadas, ou seja, ali temos o x variando de z^2-y até 2z-y que nem você
falou. Mas isso é só na parte de baixo, ou seja, apenas quando 0 wrote:

> Bom dia!
> Alguém poderia me ajudar e mostrar onde errei os limites? Resolvendo por
> integral tripla, usando f(x,y,z)=1.
>
> Grato,
> PJMS
>
> Em ter, 11 de fev de 2020 13:11, Pedro José 
> escreveu:
>
>> Boa tarde!
>>
>> Resolvi por método numérico usando, pelo menos penso eu, os mesmos
>> limites e encontrei 2,1329, muito próximo da resposta. Gostaria que alguém
>> me ajudasse onde errei na integral tripla.
>> Usei z^2-y e 2z-y como os limites para integral em dx. Em seguida, z^2 e
>> 2z para dy e finalmente 0 e 2 para dz.
>> Onde está o erro?
>> Grato,
>> PJMS
>>
>> Em ter, 11 de fev de 2020 12:49, Claudio Buffara <
>> claudio.buff...@gmail.com> escreveu:
>>
>>> O sólido é a região do 1o octante (todas as coordenadas positivas)
>>> compreendida entre os planos x-z e y-z, acima do plano z = (x+y)/2 e abaixo
>>> da z = raiz(x+y).
>>> A superfície e o plano se intersectam numa reta:
>>> raiz(x+y) = (x+y)/2 ==> x+y = (x+y)^2/4 ==> x+y = 4, contida no plano z
>>> = 2.
>>>
>>> Assim, o volume pode ser dado pela diferença entre duas integrais
>>> duplas, calculadas sobre o domínio D, no plano x-y, dado por x > 0, y > 0 e
>>> x+y = 4.
>>> Volume = Integral(D) raiz(x+y)*dA - Integral(D) (x+y)/2*dA.
>>>
>>> Usando coordenadas cartesianas, a primeira integral fica:
>>> Integral(x=0...4)Integral(y=0...4-x)*raiz(x+y)*dy*dx
>>> = Integral(0...4) (2/3)*(4^(3/2) - x^(3/2))*dx
>>> = Integral(0...4) (16/3 - (2/3)*x^(3/2))
>>> = 64/3 - (4/15)*4^(5/2)
>>> = 64/3 - 128/15
>>> = 64/5
>>>
>>> A segunda integral é:
>>> Integral(x=0...4)Integral(y=0...4-x) (x+y)/2*dy*dx
>>> = Integral(x=0...4) (1/2)*(x*(4-x) + (4-x)^2/2)*dx
>>> = Integral(0...4) (4 - x^2/4)*dx
>>> = 32/3
>>>
>>> Logo, o volume é 64/5 - 32/3 = 32/15  (se não errei nenhuma conta...)
>>>
>>> []s,
>>> Claudio.
>>>
>>>
>>> On Mon, Feb 3, 2020 at 8:55 PM Luiz Antonio Rodrigues <
>>> rodrigue...@gmail.com> wrote:
>>>
 Olá, pessoal!
 Tudo bem?
 Estou tentando resolver o seguinte problema:

 Ache o volume da região tridimensional definida por:

 z^2>>>
 Sendo que:
 x>0 e y>0 e z>0

 Com o auxílio de um software eu consegui visualizar o sólido em questão.
 Eu calculei o volume do sólido girando em torno do eixo z e dividindo o
 resultado por 4.
 A resposta que eu obtive foi (16*pi)/15, que não está correta.
 Já refiz os cálculos muitas vezes e chego sempre na mesma resposta.
 Alguém pode me ajudar?
 Muito obrigado e um abraço!

 --
 Esta mensagem foi verificada pelo sistema de antivírus e
 acredita-se estar livre de perigo.
>>>
>>>
>>> --
>>> Esta mensagem foi verificada pelo sistema de antivírus e
>>> acredita-se estar livre de perigo.
>>
>>
> --
> Esta mensagem foi verificada pelo sistema de antivírus e
> acredita-se estar livre de perigo.

-- 
Esta mensagem foi verificada pelo sistema de antiv�rus e
 acredita-se estar livre de perigo.



[obm-l] Re: [obm-l] Re: [obm-l] Re: [obm-l] Questão OBM - U

2020-01-23 Por tôpico Ralph Teixeira
Seja ABCD o quadrilatero convexo, e seja P o encontro das diagonais.

No triangulo APB, temos AP+PB>AB. Escreva as desigualdades analogas para os
triangulos BPC, CPD e DPA. Somando-as, voce vai obter que

2(AC+BD)>perimetro=8

Ou seja, o infimo tem que ser pelo menos 4.

Agora, para chegar no infimo, voce vai ter que "degenerar" os triangulos...
Entao considere um quadrilatero do tipo ABCB (ou seja, tome D=B), com,
digamos, AC=BC=2. Note que o perimetro eh 8, enquando AC=4 e BB=0, ou seja,
a soma das diagonais eh 4.

Mas alguns diriam que isso nao eh um quadrilatero convexo (bom, depende da
sua definicao de quadrilatero!)... Entao se "quadrilateros" nao incluem
casos degenrados, para fazer isso ficar rigoroso, voce teria que tomar um
quadrilatero convexo QUASE degenerado de perimetro 8 (um losango serve, a
conta fica facil), e mostrar que a soma das diagonais fica tao perto de 4
quanto voce queira.

Abraco, Ralph.

On Thu, Jan 23, 2020 at 7:24 AM gilberto azevedo 
wrote:

> Tentei com o retângulo e o quadrado, porém não obtive a resposta...  O
> gabarito é 4.
>
> Em sáb, 11 de jan de 2020 12:03, Bernardo Freitas Paulo da Costa <
> bernardo...@gmail.com> escreveu:
>
>> On Sat, Jan 11, 2020 at 11:24 AM gilberto azevedo 
>> wrote:
>> >
>> > Qual o ínfimo sobre todos os quadriláteros convexos com perímetro 8
>> da soma dos comprimentos de suas diagonais ?
>>
>> Quais são os quadriláteros que você tentaria?
>> --
>> Bernardo Freitas Paulo da Costa
>>
>> --
>> Esta mensagem foi verificada pelo sistema de antivírus e
>>  acredita-se estar livre de perigo.
>>
>>
>> =
>> Instruções para entrar na lista, sair da lista e usar a lista em
>> http://www.mat.puc-rio.br/~obmlistas/obm-l.html
>> =
>>
>
> --
> Esta mensagem foi verificada pelo sistema de antivírus e
> acredita-se estar livre de perigo.

-- 
Esta mensagem foi verificada pelo sistema de antiv�rus e
 acredita-se estar livre de perigo.



Re: [obm-l] Uma soma

2020-01-17 Por tôpico Ralph Teixeira
https://en.wikipedia.org/wiki/Tetrahedral_number

On Thu, Jan 16, 2020 at 6:13 PM marcone augusto araújo borges <
marconeborge...@hotmail.com> wrote:

> Como calcular 1 + (1+2) + (1+2+3) +... +(1+2+...+n)?
> --
> Esta mensagem foi verificada pelo sistema de antivírus e
> acredita-se estar livre de perigo.
>

-- 
Esta mensagem foi verificada pelo sistema de antiv�rus e
 acredita-se estar livre de perigo.



Re: [obm-l] Integrais Definidas

2020-01-01 Por tôpico Ralph Teixeira
Quando voce muda a variavel numa integral definida, tem que lembrar de
mudar tambem os limites de integracao.

Entao, vamos "calcular" G(x). Temos:
G(x) = Int (0,x) cos((pi*u^2)/2) du
Como voce sugeriu, tomemos t = raiz(pi/2) u. Entao:

i) dt=raiz(pi/2) du
ii) Quando u varia de 0 a x, temos que t varia de...?
Oras, quando u=0, temos t=raiz(pi/2).0=0...
...e quando u=x, temos t=raiz(pi/2).x.
Entao o intervalo de integracao para t deve ser (0,raiz(pi/2)x).

Assim:

G(x) = Int (0,raiz(pi/2)x) cos(t^2) dt / raiz(pi/2) = raiz(2/pi) *
F(raiz(pi/2).x)

Abraco, Ralph.

On Wed, Jan 1, 2020 at 12:01 PM Luiz Antonio Rodrigues <
rodrigue...@gmail.com> wrote:

> Olá, pessoal!
> Feliz Ano Novo!
> Estou tentando resolver o seguinte problema há alguns dias:
>
> São dadas:
>
> F(x)=integral de zero a x de cos(t^2)dt
>
> G(x)=integral de zero a x de cos((pi*u^2)/2)du
>
> Faça uma mudança de variável e mostre que:
>
> G(x)=a*F(b*x)
>
> Quais são os valores de a e b?
>
> Eu consegui achar o valor de a, que é:
>
> sqrt(2)/sqrt(pi)
>
> Está correto!
>
> O problema é que não consigo achar o valor de b. Acho que estou me
> atrapalhando com as variáveis x e t.
> Alguém pode me ajudar?
> Muito obrigado e um abraço!
> Luiz
>
>
>
>
>
>
>
> --
> Esta mensagem foi verificada pelo sistema de antivírus e
> acredita-se estar livre de perigo.

-- 
Esta mensagem foi verificada pelo sistema de antiv�rus e
 acredita-se estar livre de perigo.



Re: [obm-l]

2019-11-22 Por tôpico Ralph Teixeira
Algo estranho ali... Se não houver nenhuma restrição adicional ao
dominio... O minimo vale 0, quando x=0, pois todos os termos da expressão
são >=0.

Mas era isso que a gente queria?

Abraco, Ralph.

On Fri, Nov 22, 2019 at 1:07 AM gilberto azevedo 
wrote:

> Como achar o mínimo de :
> x² * √(x²/(x-12)) , usando apenas desigualdades comuns ?
>
> --
> Esta mensagem foi verificada pelo sistema de antivírus e
> acredita-se estar livre de perigo.

-- 
Esta mensagem foi verificada pelo sistema de antiv�rus e
 acredita-se estar livre de perigo.



[obm-l] Re: [obm-l] Re: [obm-l] Domínio de uma Função

2019-10-29 Por tôpico Ralph Teixeira
Muitas calculadoras evitam elevar números negativos a frações (que
realmente costumam dar problemas -- se você trocar a=2/3 por um número real
muito próximo, a função x^a pode NÃO estar definida para x<0). E em x^(2/3)
você faz o 2/3 antes de exponenciar, então a calculadora não sabe que "tem
um quadrado" ali.

Experimente o seguinte: tente (x^2)^(1/3), ao invés de x^(2/3) -- a
calculadora ainda reclama? Melhor ainda, tem algum símbolo específico para
raiz cúbica? Talvez algo como raiz3(x^2)? Aposto que ela se sai melhor se
escrever assim.

Abraço, Ralph.

On Tue, Oct 29, 2019 at 10:49 AM Claudio Buffara 
wrote:

> Estritamente falando, o domínio da função não foi definido.
> Nestes casos, o usual é tomar por domínio o maior subconjunto de R no qual
> a fórmula faz sentido.
> E, neste caso específico, a fórmula faz sentido para todo x real.
>
> O gráfico de h(x) = x^(2/3) tem uma "ponta" em x = 0, de modo que  a
> derivada h'(x) não é definida na origem.
>
> Mas não deveria haver problema algum em x = -1.
>
>
> On Tue, Oct 29, 2019 at 4:57 AM Luiz Antonio Rodrigues <
> rodrigue...@gmail.com> wrote:
>
>> Olá, pessoal!
>> Tudo bem?
>> Estou tentando descobrir os pontos  de máximo e mínimo da função:
>>
>> f(x)=1.5*(x)^(2/3)+x
>>
>> A primeira derivada se anula em x=-1.
>> Mas porque -1 não pertence ao domínio da função?
>> Vi isso numa calculadora gráfica.
>> Eu não consigo entender isso...
>> Não estou tirando a raiz cúbica de um número ao quadrado?
>> Alguém pode me ajudar?
>> Muito obrigado!
>> Luiz
>>
>> --
>> Esta mensagem foi verificada pelo sistema de antivírus e
>> acredita-se estar livre de perigo.
>
>
> --
> Esta mensagem foi verificada pelo sistema de antivírus e
> acredita-se estar livre de perigo.

-- 
Esta mensagem foi verificada pelo sistema de antiv�rus e
 acredita-se estar livre de perigo.



[obm-l] Re: [obm-l] Equação exponencial

2019-10-16 Por tôpico Ralph Teixeira
Depende!

(Esta discussao eh analoga aaquela outra de "Afinal, 0 eh natural ou
nao?"... cuja resposta eh "Decida como quiser, diga para todos como voce
decidiu, e seja coerente. De preferencia, escreva as coisas para evitar a
pergunta.")

O problema eh a convenção: quanto vale 0^0 ? Ha duas opções: alguns
matemáticos usam que 0^0=1; outros (acho que a maioria?) preferem dizer que
0^0 nao eh uma operação permitida.

Eu pessoalmente prefiro dizer que 0^0=1 (sou minoria?). Veja bem, eh uma
convenção, apenas uma convenção, então não tem "certo" ou "errado"... Mas
tenho alguns argumentos a favor disto:
A1) Se f(x) e g(x) sao funcoes **analiticas** em torno de x=a, com lim f(x)
= lim g(x) = 0 quando x->a, e f nao eh identicamente nula perto de a, entao
lim f^g=1 quando x->a. Por este motivo, 99% dos exercicios de Calculo que
caem numa "indeterminacao" do tipo 0^0 acabam dando 1!
A2) Com esta convencao, a funcao f(x)=x^0 vale 1 para todo x real, sem
excecao.
A3) Tecnicamente, (A2) de novo, mas agora explicando onde isso eh util:
para descrever um polinomio generico (ou uma serie de potencias, que a
gente usa bastante para resolver algumas EDOs), a gente escreve SUM (k=0 a
n) a_k x^k (ou SUM (k=0 a Inf) a_k x^k) -- aqui SUM eh um somatório. Pois
bem, o primeiro termo ali, quando k=0, eh a_0.x^0, e eu quero que isso
valha a_0 para todo x, inclusive para x=0. Se voce eh da escola do "0^0 nao
eh permitido", você vai ter que escrever o a_0 fora do somatório sozinho,
ou abrir uma exceção, ou fingir que nao viu o problema. :(

Para fazer o contraponto, vejo argumentos a favor de definir 0^0 como
"operacao invalida":
B1) A funcao g(x,y)=x^y (x>0, y>0) NAO TEM LIMITE quando (x,y)->(0,0),
então nao faz sentido botar um valor especifico para g(0,0).
B2) Ok, 99% dos limites do tipo 0^0 dao 1, mas os outros 1% NAO DAO 1, e
isto poderia causar confusao!
B3) A funcao f(x)=0^x eh continua em (0,Inf). Colocando f(0)=1, ela fica
descontinua em x=0.

Ainda assim, prefiro 0^0=1 -- acho (A3) forte, acho MUITO mais conveniente
pensar que 0^0=1 para nao ter que me separar aquele a_0 do polinomio.

Abraco, Ralph.

On Wed, Oct 16, 2019 at 4:36 PM Prof. Douglas Oliveira <
profdouglaso.del...@gmail.com> wrote:

> Amigos, me ajudem por favor.
>
> Afinal de contas, zero, é ou não é raiz da equação
> (sqrt(x))^x=x^(sqrt(x=)?
>
> Douglas Oliveira.
>
> --
> Esta mensagem foi verificada pelo sistema de antivírus e
> acredita-se estar livre de perigo.

-- 
Esta mensagem foi verificada pelo sistema de antiv�rus e
 acredita-se estar livre de perigo.



[obm-l] Re: [obm-l] Re: [obm-l] Re: [obm-l] Aproximação Linear e Quadrática

2019-10-12 Por tôpico Ralph Teixeira
Pois eh, para mim essas sao as respostas corretas: "0" e "0" de novo. Se
voce usar Serie de Taylor, faz sentido! Perto de 0, x^3 fica mais bem
aproximado pela expressao "0" do que qualquer outra funcao afim ou
quadratica!

Abraco, Ralph.

On Sat, Oct 12, 2019 at 7:29 PM Luiz Antonio Rodrigues <
rodrigue...@gmail.com> wrote:

> Olá, Ralph!
> Tudo bem?
> Sim, eu pensei nisso...
>
> Para a aproximação linear eu usei:
> L(x) ~= f(0) + f'(0)*x = 0
>
> Para a quadrática:
> Q(x) ~= f(0) + f'(0)*x + (1/2)*f''(0)*x^2 = 0
>
> Estranho, não é?
>
>
> On Sat, Oct 12, 2019, 7:09 PM Ralph Teixeira  wrote:
>
>> Hm, por que nao eh a resposta correta? x^3 eh BEM perto de 0 quando x eh
>> pequeno...
>>
>> Abraco, Ralph.
>>
>> On Sat, Oct 12, 2019 at 5:15 PM Luiz Antonio Rodrigues <
>> rodrigue...@gmail.com> wrote:
>>
>>> Olá, pessoal!
>>> Boa tarde!
>>> Tudo bem?
>>> Preciso de uma dica.
>>> Estou calculando as aproximações linear e quadrática de:
>>>
>>> f(x)=x^3
>>>
>>> Nas duas eu obtive zero, usando a série de Taylor, que não é a resposta
>>> correta.
>>> Alguém tem alguma ideia?
>>> Muito obrigado!
>>> Luiz
>>>
>>> --
>>> Esta mensagem foi verificada pelo sistema de antivírus e
>>> acredita-se estar livre de perigo.
>>
>>
>> --
>> Esta mensagem foi verificada pelo sistema de antivírus e
>> acredita-se estar livre de perigo.
>
>
> --
> Esta mensagem foi verificada pelo sistema de antivírus e
> acredita-se estar livre de perigo.

-- 
Esta mensagem foi verificada pelo sistema de antiv�rus e
 acredita-se estar livre de perigo.



[obm-l] Re: [obm-l] Aproximação Linear e Quadrática

2019-10-12 Por tôpico Ralph Teixeira
Hm, por que nao eh a resposta correta? x^3 eh BEM perto de 0 quando x eh
pequeno...

Abraco, Ralph.

On Sat, Oct 12, 2019 at 5:15 PM Luiz Antonio Rodrigues <
rodrigue...@gmail.com> wrote:

> Olá, pessoal!
> Boa tarde!
> Tudo bem?
> Preciso de uma dica.
> Estou calculando as aproximações linear e quadrática de:
>
> f(x)=x^3
>
> Nas duas eu obtive zero, usando a série de Taylor, que não é a resposta
> correta.
> Alguém tem alguma ideia?
> Muito obrigado!
> Luiz
>
> --
> Esta mensagem foi verificada pelo sistema de antivírus e
> acredita-se estar livre de perigo.

-- 
Esta mensagem foi verificada pelo sistema de antiv�rus e
 acredita-se estar livre de perigo.



[obm-l] Re: [obm-l] Retas envolvendo uma parábola

2019-10-11 Por tôpico Ralph Teixeira
Eu penso "EDO de Clairaut", que fornece uma maneira de encontrar o envelope
de uma familia de retas dadas.

Abraco, Ralph.

On Fri, Oct 11, 2019 at 10:40 PM Luís Lopes  wrote:

> Sauda,c~oes,
>
> Numa troca de mensagens sobre um procedimento de resolução do problema
> "construir um triângulo ABC dados " apareceu a seguinte
> afirmação:
>
> When two points move along two intersecting
> straight lines at constant speeds (not necessarily equal)
> line connecting them envelopes a parabola.
>
> Alguém poderia me dar a prova ou referências sobre isso ?
>
> Obrigado.
>
> Luís
>
>
>
> --
> Esta mensagem foi verificada pelo sistema de antivírus e
> acredita-se estar livre de perigo.
>

-- 
Esta mensagem foi verificada pelo sistema de antiv�rus e
 acredita-se estar livre de perigo.



[obm-l] Re: [obm-l] Triângulos

2019-09-26 Por tôpico Ralph Teixeira
Hmmm... Esse enunciado, como estah , nao funciona... O problema eh definir
o que significa escolher um triangulo "ao acaso". Algumas opcoes:

-- Escolher 3 numeros uniformemente na regiao do R^3 definida por
0 Inf depois.)
-- Escolher 3 pontos uniformemente dentro de um quadrado de lado R, que
seriam os vertices. (Se necessario, tome R-> Inf depois.)
-- Escolher 3 pontos uniformemente numa circunferencia de raio R, que
seriam os vertices. (Se necessario, tome R-> Inf depois.)

Todas estas opcoes sao razoaveis para interpretar "ao acaso", mas nao levam
aa mesma resposta... :(

Abraco, Ralph.

On Thu, Sep 26, 2019 at 5:12 PM João Maldonado 
wrote:

> Eaí galera.
> Fica um problema legal de probabilidade pra vocês resolverem (e me
> ajudarem).
>
> Um triângulo é dito aproximadamente equilátero quando o maior de seus
> lados não excede o menor por 10%. Um triângulo é selecionado ao acaso. Qual
> a chance de ele ser aproximadamente equilátero?
>
> Pensei em prosseguir da seguinte forma.
> Sendo c o maior lado, b o do meio e a o menor. Sendo P1 a probabilidade
> pedida.
> Temos que:
> P1=6*P(a<= b<= c<= mín(1.1a, a+b))= 6*P(a<=b<=c<=1.1a)
>
> Aí eu pensei em pegar um cubo de lado 1 e fazer uma integral tripla com
> esses limites.
> O problema é que um cubo de lado 1 não é um subespaço amostral
> equivalente. Pense comigo:
> Se tivermos um dos lados valendo 0.9  o outro 0.7, o terceiro poderia
> valer até 1.6, e isso estaria fora do cubo, mesmo os dois primeiros estando
> dentro. Dessa forma não saberíamos qual seria o "denominador" do nosso P.
>
> Alguém consegue me ajudar?
>
>
> --
> Esta mensagem foi verificada pelo sistema de antivírus e
> acredita-se estar livre de perigo.
>

-- 
Esta mensagem foi verificada pelo sistema de antiv�rus e
 acredita-se estar livre de perigo.



[obm-l] Re: [obm-l] Re: [obm-l] Combinatória

2019-09-08 Por tôpico Ralph Teixeira
A face de baixo eh P1-P2-P3-P4, a de cima eh P8-P7-P6-P5 (P8 acima do P1,
etc.). Desse jeito, as 12 arestas sao as 8 do ciclo
P1-P2-P3-P4-P5-P6-P7-P8-P1, mais os 4 pares P1-P4, P2-P7, P3-P6, P5-P8.

Cada "maneira de rotular" vai ser representada por uma linha com 8 numeros
(o rotulo do ponto Pj na coluna j). Por exemplo, uma rotulacao seria [1 7 5
3 8 2 4 6], indicando que o ponto P1 levou o numero 1, o ponto P2 levou o
7, etc.  Note que este exemplo eh, de fato, valido -- nao ha vizinhos com
diferenca 1 (no ciclo P1-...P8), e os pares P1-P4=1-3, P2-P7=7-4,... tambem
nao tem distancia 1,-1,7 nem -7 entre si.

Bom, ai chutei o balde e botei no MatLab: :D :D :D

Perm1=perms([1 2 3 4 5 6 7 8])   % Acha as 40320
permutacoes; exagerei, podia fixar um deles, mas o Matlab aguenta.
Diff=abs(Perm1(:,[1:8,1,2,3,5])-Perm1(:,[2:8,1,4,7,6,8]));  % Calcula
diferencas de rotulos entre vertices vizinhos -- cada coluna significa uma
aresta.
Test=ismember(Diff,[1 7])% Encontra
todos as diferencas proibidas. Gera uma matriz com 1 para cada elemento 1
ou 7 de Diff, e 0 caso contrario.
Index=(sum(Test,2)==0)  % Para uma
linha ser valida, TODOS os elementos tem que ser 0, ou seja, a soma da
linha tem que dar 0.
Validas=Perm1(Index,:)   % Seleciona a
submatriz com as linhas validas.

Muito bem, achei 480 permutacoes validas... Aqui as 60 que comecam com 1,
talvez inspire alguem a achar uma solucao no braco, dividindo em casos...
Hm, vejo umas simetrias adicionais que poderiam ser exploradas...

Abraco, Ralph.

 1 7 5 3 8 2 4 6
 1 7 5 3 6 8 2 4
 1 7 4 6 8 2 5 3
 1 7 4 6 3 8 2 5
 1 7 4 6 2 8 3 5
 1 7 3 6 8 5 2 4
 1 7 3 5 8 6 2 4
 1 7 3 5 2 8 4 6
 1 7 2 5 3 8 4 6
 1 7 2 4 8 6 3 5
 1 7 2 4 8 5 3 6
 1 7 2 4 6 8 5 3
 1 6 8 4 2 5 3 7
 1 6 8 3 7 5 2 4
 1 6 8 3 7 4 2 5
 1 6 8 3 5 2 4 7
 1 6 4 7 5 2 8 3
 1 6 4 7 3 8 2 5
 1 6 4 7 2 8 3 5
 1 6 3 7 2 5 8 4
 1 6 3 5 2 8 4 7
 1 6 2 5 7 4 8 3
 1 6 2 5 3 8 4 7
 1 6 2 4 7 5 8 3
 1 5 8 4 6 2 7 3
 1 5 8 4 2 6 3 7
 1 5 8 3 6 2 7 4
 1 5 7 4 6 2 8 3
 1 5 7 3 8 4 2 6
 1 5 7 3 6 2 8 4
 1 5 3 7 4 8 2 6
 1 5 3 7 2 6 8 4
 1 5 3 6 4 8 2 7
 1 5 2 7 4 8 3 6
 1 5 2 6 8 4 7 3
 1 5 2 6 4 8 3 7
 1 4 8 6 3 5 2 7
 1 4 8 5 7 2 6 3
 1 4 8 5 3 6 2 7
 1 4 7 5 8 2 6 3
 1 4 7 3 8 5 2 6
 1 4 6 3 8 2 7 5
 1 4 6 3 7 2 8 5
 1 4 6 3 5 8 2 7
 1 4 2 7 5 8 6 3
 1 4 2 7 3 6 8 5
 1 4 2 7 3 5 8 6
 1 4 2 6 8 5 7 3
 1 3 8 6 4 2 5 7
 1 3 8 6 2 5 7 4
 1 3 8 6 2 4 7 5
 1 3 8 5 7 2 6 4
 1 3 7 5 8 2 6 4
 1 3 7 5 2 4 8 6
 1 3 7 4 2 5 8 6
 1 3 6 4 8 2 7 5
 1 3 6 4 7 2 8 5
 1 3 6 4 2 8 5 7
 1 3 5 7 4 2 8 6
 1 3 5 7 2 8 6 4

On Sun, Sep 8, 2019 at 9:08 AM Anderson Torres 
wrote:

> Em sáb, 7 de set de 2019 às 02:23, marcone augusto araújo borges
>  escreveu:
> >
> > De quantas maneiras podemos atribuir um número de 1 a 8 a cada vértice
> de um cubo de modo que não apareçam números consecutivos nas extremidades
> de uma mesma aresta, sendo o 1 e o 8 considerados consecutivos e  a dois
> vértices não seja atribuído um mesmo número?

[obm-l] Re: [obm-l] Re: [obm-l] Re: [obm-l] Dúvida basica equação polar

2019-09-02 Por tôpico Ralph Teixeira
Pois bem, se voce parametrizar com relacao ao centro, teria
x(teta)=1+cos(teta) e y(teta)=sin(teta). Se fosse assim, teria que ser
0 wrote:

> Caro Ralf, obrigado pela resposta.Para mim ficou confuso pq pensei que a
> parametrização do círculo se daria colocando como referencia o novo centro
> do mesmo. Quando penso em circulos diferentes , por exemplo residindo em
> apenas um quadrante tenho dificuldade de imaginar varrendo todos os pontos
> . Vou refletir sobre esses casos pois parecem ser obtidos como vc disse de
> fato.
>
> Att.Gabriel
>
> Em Seg, 2 de set de 2019 18:04, Ralph Teixeira 
> escreveu:
>
>> Bom, vale a pena fazer uma figura primeiro... Fez? Note como este circulo
>> estah nos primeiro e quarto quadrantes apenas.
>>
>> Entao suponho que voce fez as contas e descobriu que r=2cos(teta). No
>> quarto quadrante vale -pi/2> onde pi/2> terceiro quadrante). Por isso que -pi/2>
>> Para ser mas exato, o que aconteceria na equacao r=2cos(teta) para teta
>> entre pi/2 e 3pi/2... Tipo, experimente pensar em teta=pi para fazer um
>> exemplo. Jogando na equacao, ficaria r=2cos(pi)=-2?!?
>>
>> Aqui ha duas opcoes:
>> a) Alguns livros vao insistir que r>=0 sempre. Neste caso, fica claro que
>> pi/2> cosseno fica negativo.
>> b) Alguns outros livros sao mais "liberais" e permitem r<0 -- a
>> interpretacao seria que quando r eh negativo voce anda na reta que forma
>> angulo teta com o eixo x NO SENTIDO OPOSTO. Por exemplo, r=-2 e teta=pi eh
>> de fato o ponto (2,0) (voce anda na direcao NEGATIVA do eixo x, mas voce
>> anda -2, entao acaba andando para a DIREITA duas unidades). Neste caso,
>> colocar teta=pi dah um ponto no circulo sim senhor! Mas, mesmo assim, eu
>> usaria apenas -pi/2> ponto (2,0) JAH APARECEU com teta=0, e nao vejo porque conta-lo duas vezes
>> (e, dependendo da aplicacao, voce NAO QUER contar cada ponto duas vezes).
>>
>> Abraco, Ralph.
>>
>> On Mon, Sep 2, 2019 at 4:55 PM Gabriel Lopes  wrote:
>>
>>> Boa tarde, tenho uma duvida básica da representação em equação polar do
>>> círculo  (x-1)^2 +y^2= 1.
>>>
>>> Pq os intervalo de teta é de -pi/2 a pi/2 e nao de 0 a 2pi?
>>>
>>> --
>>> Esta mensagem foi verificada pelo sistema de antivírus e
>>> acredita-se estar livre de perigo.
>>
>>
>> --
>> Esta mensagem foi verificada pelo sistema de antivírus e
>> acredita-se estar livre de perigo.
>
>
> --
> Esta mensagem foi verificada pelo sistema de antivírus e
> acredita-se estar livre de perigo.

-- 
Esta mensagem foi verificada pelo sistema de antiv�rus e
 acredita-se estar livre de perigo.



[obm-l] Re: [obm-l] Dúvida basica equação polar

2019-09-02 Por tôpico Ralph Teixeira
Bom, vale a pena fazer uma figura primeiro... Fez? Note como este circulo
estah nos primeiro e quarto quadrantes apenas.

Entao suponho que voce fez as contas e descobriu que r=2cos(teta). No
quarto quadrante vale -pi/2=0 sempre. Neste caso, fica claro que
pi/2 wrote:

> Boa tarde, tenho uma duvida básica da representação em equação polar do
> círculo  (x-1)^2 +y^2= 1.
>
> Pq os intervalo de teta é de -pi/2 a pi/2 e nao de 0 a 2pi?
>
> --
> Esta mensagem foi verificada pelo sistema de antivírus e
> acredita-se estar livre de perigo.

-- 
Esta mensagem foi verificada pelo sistema de antiv�rus e
 acredita-se estar livre de perigo.



[obm-l] Re: [obm-l] Re: [obm-l] Re: [obm-l] Re: [obm-l] Números primos

2019-08-29 Por tôpico Ralph Teixeira
Exato, 6 é um número pequeno com "muitos" divisores, então é um bom ponto
de partida...

Claro, a gente podia continuar analisando o problema e achando mais e mais
restrições (módulo 12... módulo 15... módulo 120...)... Mas, em algum
momento, você tem que partir para tentar uns números e ver o que acontece,
senão não fecha nunca. :D

On Thu, Aug 29, 2019 at 1:02 PM Claudio Buffara 
wrote:

> Acho que apenas o fato de que, apesar de existirem 6 restos possíveis ao
> se dividir um inteiro por 6, os primos maiores que 3 deixam apenas resto 1
> ou resto 5 (== -1).
>
>
> On Thu, Aug 29, 2019 at 12:42 PM Carlos Monteiro <
> cacacarlosalberto1...@gmail.com> wrote:
>
>> Valeu!
>> Tem alguma motivação para a congruência mod 6?
>>
>>
>> Em qui, 29 de ago de 2019 12:12, Ralph Teixeira 
>> escreveu:
>>
>>> Resposta curta: 3, 7 e 13 servem.
>>>
>>> Resposta longa:
>>> Sejam p1>> p1=2, porque então a soma seria par.
>>> Afirmo que p1=3. De fato, caso contrário, todos eles deixariam resto 1
>>> ou -1 (hm, eu devia dizer 5, mas vou escrever -1 mesmo) na divisão por 6.
>>> Mas então seus quadrados deixariam resto 1 na divisão por 6, e a soma dos
>>> quadrados deixaria resto 3, absurdo.
>>> Note que p2 e p3 têm que deixar o mesmo resto (1 ou -1) na divisão por 6
>>> (caso contrário, p2+p3=6a+1+6b-1 seria divisível por 6, então 3+p2+p3 seria
>>> divisível por 3).
>>> Então a gente quer coisas do tipo {3,6a+1,6b+1} ou {3,6a-1,6b-1}. Isto
>>> me leva a tentar
>>> {3,5,11} -- soma 19, soma dos quadrados 155; Quebrei a cara.
>>> {3,7,13} -- soma 23, soma dos quadrados 227. Ambos primos! Funcionou!
>>>
>>> Abraço, Ralph.
>>>
>>> On Thu, Aug 29, 2019 at 11:35 AM Carlos Monteiro <
>>> cacacarlosalberto1...@gmail.com> wrote:
>>>
>>>> Encontre três números primos distintos dois a dois tais que sua soma e
>>>> a soma dos seus quadrados são números primos também.
>>>>
>>>> --
>>>> Esta mensagem foi verificada pelo sistema de antivírus e
>>>> acredita-se estar livre de perigo.
>>>
>>>
>>> --
>>> Esta mensagem foi verificada pelo sistema de antivírus e
>>> acredita-se estar livre de perigo.
>>
>>
>> --
>> Esta mensagem foi verificada pelo sistema de antivírus e
>> acredita-se estar livre de perigo.
>
>
> --
> Esta mensagem foi verificada pelo sistema de antivírus e
> acredita-se estar livre de perigo.

-- 
Esta mensagem foi verificada pelo sistema de antiv�rus e
 acredita-se estar livre de perigo.



[obm-l] Re: [obm-l] Números primos

2019-08-29 Por tôpico Ralph Teixeira
Resposta curta: 3, 7 e 13 servem.

Resposta longa:
Sejam p1 wrote:

> Encontre três números primos distintos dois a dois tais que sua soma e a
> soma dos seus quadrados são números primos também.
>
> --
> Esta mensagem foi verificada pelo sistema de antivírus e
> acredita-se estar livre de perigo.

-- 
Esta mensagem foi verificada pelo sistema de antiv�rus e
 acredita-se estar livre de perigo.



[obm-l] Re: [obm-l] Distribuição de probabilidade da soma de números arredondados

2019-08-08 Por tôpico Ralph Teixeira
Não sei a resposta, mas a distribuição deve depender de n  Por exemplo,
se n=2, claramente p(100)=1, enquanto se n é muito grande, eu aposto que
p(0)~1 (escolhendo 10 googlelhões de termos, muito provavelmente quase
todos serão menores que 1/2, e portanto eu aposto que todos arredondam para
0, com muita probabilidade).

Será que dá para achar uma recorrência? Acho que deveríamos começar
pensando no problemas mais genérico e mais simples:

"Dividindo o intervalo [a,b] em dois pedaços, medindo cada pedaço,
arredondando e somando, qual a distribuição de probabilidade da soma S?"
(Aliás, acho que melhor ainda seria perguntar a distribuição de S-(b-a),
isto é, ver o quanto você "ganha" pu "perde" quando subdivide um intervalo.
Acho que isso só depende da parte fracionária de b-a, mas tem que ver
direitinho.)

Afinal, passar de n para n+1 amostras significa escolher um ponto a mais, o
que se resume mais ou menos a esta questão anterior -- bom tem que pesar a
probabilidade de cair em cada intervalo antigo, mas talvez saia algo
pensando assim.

Abraço, Ralph.

On Wed, Aug 7, 2019 at 2:31 PM Rodrigo Ângelo 
wrote:

> Vi o seguinte prolbema num outro grupo que faço parte, e como não teve
> solução por lá, resolvi trazer pra esta lista (irei postar tradução livre
> feita por mim abaixo)
>
> F(n) is the random variable received by partitioning 100 into n parts,
>> rounding those parts, and adding the results. An example partition would
>> be: 49.7, 49.7, 0.6, which rounded becomes 50, 50, 1, added becomes 101.
>> The partition is created by choosing n-1 real numbers in [0,100] uniformly,
>> which implicitly defines a partition. What is the distribution of F(n)?
>
>
> Seja F(n) uma variável aleatória definida particionando o número 100 em n
> partes, arredondando essas partes e adicionando os resultados do
> arredondamento. Um exemplo seria 49,7; 49,7; 0,6; que arredondando fica
> 50; 50; 1; resultando em 101. A partição é criada escolhendo n-1 números
> reais no intervalo [0,100] com distribuição uniforme, que implicitamente
> define uma partição. Qual a distribuição de F(n)?
>
> No exemplo anterior, temos n=3 e os n-1 números sorteados foram 49,7 e
> 99,4.
>
> O arredondamento é feito de forma a minimizar a distância até o inteiro
> mais próximo.
>
> Casos em que o inteiro antecessor e o sucessor são equidistantes (ex: 2,5)
> podem ser desconsiderados, porque têm probabilidade zero.
>
> Casos em que um número é sorteado mais de uma vez também tem probabilidade
> zero.
>
> Fiz uma simulação
> https://drigoangelo.shinyapps.io/MonteCarlo_RoundProblem/ e aparentemente
> a função de probabilidade de F seria aproximadamente (independente de n):
>
> p(100) = 0,600
> p(99) = p(101) = 0,196
> p(98) = p(102) = 0,200
> p(F) = 0 para F não pertencente a {98, 99, 100, 101, 102}.
>
> Não consegui encontrar uma distribuição para F analiticamente, usando a
> definição de fdp. O caminho que eu tentei foi usar que cada número pode ser
> arredondado para cima com distribuição Bernoulli(0,5), mas não consegui
> avançar depois disso.
>
> Atenciosamente,
> Rodrigo de Castro Ângelo
>
> --
> Esta mensagem foi verificada pelo sistema de antivírus e
> acredita-se estar livre de perigo.

-- 
Esta mensagem foi verificada pelo sistema de antiv�rus e
 acredita-se estar livre de perigo.



[obm-l] Re: [obm-l] Decomposição

2019-08-08 Por tôpico Ralph Teixeira
Acho que a série binomial pode ajudar:
https://pt.wikipedia.org/wiki/S%C3%A9rie_binomial

Em geral, "séries de potências" (ou seja, Séries de Taylor) podem abrir uma
função suave em soma de termos.

Abraço, Ralph.

On Thu, Aug 8, 2019 at 1:09 PM Alexandre Antunes <
prof.alexandreantu...@gmail.com> wrote:

>
> Bom dia,
>
> É possível decompor
>
> (1-v^2)^(q/(1-q)), q real diferente de zero,  em algum tipo de soma de
> parcelas?
>
> Tem algum livro que trate esse tipo de operação?
>
> Atenciosamente,
>
> Prof. Msc. Alexandre Antunes
> www alexandre antunes com br
>
>
> 
>  Livre
> de vírus. www.avast.com
> .
> <#m_-348732496361712317_DAB4FAD8-2DD7-40BB-A1B8-4E2AA1F9FDF2>
>
> --
> Esta mensagem foi verificada pelo sistema de antivírus e
> acredita-se estar livre de perigo.

-- 
Esta mensagem foi verificada pelo sistema de antiv�rus e
 acredita-se estar livre de perigo.



Re: [obm-l] Conjuntos

2019-08-03 Por tôpico Ralph Teixeira
Vou escrever n(A)=a e n(B)=b para facilitar. Voce sabe que n(P(A))=2^a e
n(P(B))=2^b, sim?

Como A e B sao disjuntos, entao P(A) e P(B) sao disjuntos EXCETO pelo
conjunto vazio que aparece em ambos. Assim:
n(P(A) U P(B))=n(P(A)) + n(P(B)) - 1 = 2^a+2^b-1

Juntando tudo, temos:
2^a+2^b=2^(a+b)
2^(a+b)-2^a-2^b=0
(2^a-1)(2^b-1)=1
Como sao inteiros:
2^a-1=2^b-1=1 (pois -1 cada nao seria possivel)
a=b=1
a-b=0

Ou seja, se eu nao errei bobagem, a resposta eh (a).


On Sat, Aug 3, 2019 at 5:07 PM Joao Breno  wrote:

> Sejam A e B dois conjuntos disjuntos, ambos finitos e não vazios, tais que
> n(P(A) ∪ P(B)) + 1 = n(P(A∪B)). Então, a diferença n(A) - n(B) pode assumir:
>
> a)um unico valor
>
> b)dois valores distintos
>
> c)tres valores distintos
>
> d)quatro valores distintos
>
> e)mais que quatro valores distintos
>
> OBS: P(A) é o conjunto de partes de A.
>
> Alquem pode me ajudar nessa questao?
> Att. Breno.
>
> --
> Esta mensagem foi verificada pelo sistema de antivírus e
> acredita-se estar livre de perigo.

-- 
Esta mensagem foi verificada pelo sistema de antiv�rus e
 acredita-se estar livre de perigo.



Re: [obm-l] Determinante

2019-07-24 Por tôpico Ralph Teixeira
Ah, tenho uma ideia rapida para a 4a raiz: note que o termo em z^3 nao
existe... Entao a soma das raizes eh 0. Assim, se z1=w, z2=x e z3=y, entao
devemos ter z4=-w-x-y.

Abraco, Ralph.

On Wed, Jul 24, 2019 at 11:22 AM Ralph Teixeira  wrote:

> Eu entendi a dica assim: finja momentanemante (apenas para ajudar a
> pensar) que x, y e w sao constantes, digamos, 3, pi e 111. Entao abrindo o
> determinante pela ultima coluna, voce vai ficar com um polinomio de quarto
> grau em z, correto? Pois bem, se as raizes desses polinomio forem z1, z2,
> z3 e z4, entao o polinomio tem que ser P(z)=a(z-z1)(z-z2)(z-z3)(z-z4), onde
> a eh o coeficiente de z^4 no polinomio.
>
> Entao, vamos fazer isso, pensando que z eh a unica variavel e x,y e w sao
> constantes. O coeficiente de z^4 eh o determinante 3x3 do canto superior
> esquerdo, que eh Vandermonde, entao a=(x-w)(y-w)(y-x). Claramente (sim?),
> z1=w, z2=x e z3=y sao raizes, entao jah temos P(z)=(x-w)(y-w)(y-x).
> (z-w)(z-x)(z-y). (z-z4). Falta apenas mostrar que z4=-w-x-y eh a ultima
> raiz, ou seja, se voce mostrar que aquele determinante se anula sempre que
> x+y+z+w=0, acabou...
>
> Abraco, Ralph.
>
> On Wed, Jul 24, 2019 at 12:24 AM Vanderlei Nemitz 
> wrote:
>
>> Pessoal, como posso calcular o seguinte determinante, utilizando um
>> polinômio em z?
>>
>> 1   1  1  1
>> w   x  y   z
>> w^2   x^2   y^2   z^2
>> w^4   x^4   y^4   z^4
>>
>> A resposta é  (z − y)(z − x)(z − w)(y − x)(y − w)(x − w)(w + x + y + z).
>>
>> Vi em uma lista e a dica é essa:
>> Expanda o determinante ao longo da última coluna e encontre seus zeros
>> como um polinômio em z.
>>
>> Não conheço esse truque.
>>
>> Muito obrigado!
>>
>>
>>
>> --
>> Esta mensagem foi verificada pelo sistema de antivírus e
>> acredita-se estar livre de perigo.
>
>

-- 
Esta mensagem foi verificada pelo sistema de antiv�rus e
 acredita-se estar livre de perigo.



Re: [obm-l] Determinante

2019-07-24 Por tôpico Ralph Teixeira
Eu entendi a dica assim: finja momentanemante (apenas para ajudar a pensar)
que x, y e w sao constantes, digamos, 3, pi e 111. Entao abrindo o
determinante pela ultima coluna, voce vai ficar com um polinomio de quarto
grau em z, correto? Pois bem, se as raizes desses polinomio forem z1, z2,
z3 e z4, entao o polinomio tem que ser P(z)=a(z-z1)(z-z2)(z-z3)(z-z4), onde
a eh o coeficiente de z^4 no polinomio.

Entao, vamos fazer isso, pensando que z eh a unica variavel e x,y e w sao
constantes. O coeficiente de z^4 eh o determinante 3x3 do canto superior
esquerdo, que eh Vandermonde, entao a=(x-w)(y-w)(y-x). Claramente (sim?),
z1=w, z2=x e z3=y sao raizes, entao jah temos P(z)=(x-w)(y-w)(y-x).
(z-w)(z-x)(z-y). (z-z4). Falta apenas mostrar que z4=-w-x-y eh a ultima
raiz, ou seja, se voce mostrar que aquele determinante se anula sempre que
x+y+z+w=0, acabou...

Abraco, Ralph.

On Wed, Jul 24, 2019 at 12:24 AM Vanderlei Nemitz 
wrote:

> Pessoal, como posso calcular o seguinte determinante, utilizando um
> polinômio em z?
>
> 1   1  1  1
> w   x  y   z
> w^2   x^2   y^2   z^2
> w^4   x^4   y^4   z^4
>
> A resposta é  (z − y)(z − x)(z − w)(y − x)(y − w)(x − w)(w + x + y + z).
>
> Vi em uma lista e a dica é essa:
> Expanda o determinante ao longo da última coluna e encontre seus zeros
> como um polinômio em z.
>
> Não conheço esse truque.
>
> Muito obrigado!
>
>
>
> --
> Esta mensagem foi verificada pelo sistema de antivírus e
> acredita-se estar livre de perigo.

-- 
Esta mensagem foi verificada pelo sistema de antiv�rus e
 acredita-se estar livre de perigo.



Re: [obm-l] Bug em probabilidade

2019-06-21 Por tôpico Ralph Teixeira
Lendo o que voce escreveu... Ok, a discussao vai um pouco pelo "aleatorio",
e o que isso significa. O fato eh que tem varias maneiras de escolher
"aleatoriamente", e nem sempre elas sao iguais.

Um exemplo que eu gosto de dar: na minha sala de aula, tem 651 homens e 1
mulher. Vou sortear um premio "aleatoriamente"...

Opcao 1: Dou 652 numeros (de 1 a 652) aos alunos, vou no Random.org e mando
ele escolher um numero "aleatoriamente" entre 1 e 652.
Opcao 2: Jogo uma moeda "aleatoriamente" para decidir o sexo do ganhador;
depois escolho, dentro do sexo dado, um numero "aleatorio" (como na Opcao
1) para decidir quem ganhou.

Tudo parece justo e aleatorio, a moeda nao eh viciada, e todas as escolhas
foram feitas com geradores de numeros aleatorios justos, sem vies algum...
mas na Opcao 2 a mulher sozinha tem 50% de chance de ganhar! O vies nao
estah na "alaetoriedade", estah no metodo de escolha... Alias, para ser
mais exato, nao eh nem um problema do metodo em si, eh um problema do
metodo EM FACE A POPULACAO DA MINHA SALA.

Entao, no seu caso:
Opcao 1: Numere as caixas, 1, 2, 3. Para cada bola, escolha um numero de 1
a 3 no random.org, coloque a bola na caixa com aquele numero.
Opcao 2: Considere todas as listas de (x,y,z) que podem dar no final, sao
36 delas: (7,0,0),(6,1,0),...(0,0,7). Coloque 36 papeizinhos numa urna com
estas triplas, sorteie uma, coloque as bolas como no papel sorteado.

Ambas sao "justas" em algum sentido, mas essas duas maneiras de encher as
caixas sao, probabilisticamente, muito diferentes! Na primeira, botar 7
bolas na caixa 1 tem chance 1/3^7; na segunda botar 7 bolas na caixa 1 tem
chance 1/136, como voce mesmo disse! No caso do enunciado do problema, acho
que a Opcao 1 eh a maneira mais natural de interpretar o problema.

Abraco, Ralph.

On Fri, Jun 21, 2019 at 4:22 PM Vanderlei Nemitz 
wrote:

> Pessoal, fiquei confuso com a seguinte questão:
>
> Distribuindo-se aleatoriamente 7 bolas iguais em 3 caixas diferentes, qual
> é a probabilidade de que uma delas contenha exatamente 4 bolas?
>
> Como as bolas são iguais, existem 36 maneiras de alocar as bolas nas
> caixas, que é o número de soluções naturais da equação x + y + z = 7.
> Dessas, 12 tem 4 bolas em uma das caixas e a probabilidade é 1/3.
>
> Mas...e se considerarmos que existem 3^8 maneiras de alocar as bolas?
> Nesse caso, (C7,4).(2^3).3 = 840 maneiras correspondem a 4 bolas em uma das
> caixas e a probabilidade não é 1/3.
>
> Porque as respostas são diferentes?
>
> Claro que no primeiro caso as bolas são iguais e no segundo diferentes.
>
> Mas para uma distribuição aleatória, as probabilidades não deveriam ser
> iguais?
> Alguém com os olhos fechados, colocando as bolas nas caixas não teria a
> mesma chance em qualquer caso?
>
> Fiquei confuso...
>
> Errei alguma conta ou não?
>
> Muito obrigado,
>
> Vanderlei
>
> --
> Esta mensagem foi verificada pelo sistema de antivírus e
> acredita-se estar livre de perigo.

-- 
Esta mensagem foi verificada pelo sistema de antiv�rus e
 acredita-se estar livre de perigo.



Re: [obm-l] Bug em probabilidade

2019-06-21 Por tôpico Ralph Teixeira
Oi, Vanderlei.

Minha frase predileta, razão de 90% das confusões que fazemos (todos nos,
inclusive eu!) em probabilidade:

"NEM TODO ESPACO EH EQUIPROVAVEL"
ou, traduzindo
"SOH PORQUE TEM N MANEIRAS DE ALGO ACONTECER, NAO SIGNIFICA QUE TODAS AS
MANEIRAS TEM PROBABILIDADE 1/N".

Hm, ok, talvez eu tenha exagerado no CAPS, mas acho que vale a pena. Por
conta disso, muita gente (eu tambem!) erra problemas de probabilidade, em
varios niveis:

a) Obvio: "Eu posso ganhar a mega-sena amanha, ou nao. 50% de chance.
oops!"
b) Menos obvio: "A soma desses dois dados pode dar de 2 a 12... Entao cada
numero tem 1/11 de probabilidade oops!"
c) Super sutis: "Tem 2 portas fechadas nestes show, entao cada uma tem 50%
de chande de ter um bode oops!"

---///---

No seu caso: o que significa "distribuir aleatoriamente" as bolas? Para
mim, o mais natural eh pegar cada bola, escolher para ela uma das 3 caixas
(com probabilidade 1/3 para cada, que nao estah explicito no enunciado mas
parece razoavel)... Entao a sua segunda interpretacao eh a correta, existem
3^8 maneiras EQUIPROVAVEIS de colocar as bolas nas caixas.

Note como a outra maneira estah furada: a gente acha mesmo que
(x,y,z)=(7,0,0) tem a mesma probabilidade de ocorrer que (x,y,z)=(3,2,2)?
Nao, a segunda tripla eh bem mais provavel! Entao nao faz sentido contar
numero de solucoes ali.

Abraco, Ralph.

---///---

P.S.: Eu culpo o jeito que a gente introduz probabilidade no ensino
medio O problema eh muitos livros e professores escrevem assim, em
letras enormes, cercado por um quadrinho para a gente decorar:

* Probabilidade = Numero de Casos Favoraveis / Numero de Casos
Possiveis **

E isto eh FALSO, porque ele devia escrever antes "SE OS CASOS TEM A MESMA
PROBABILIDADE"

Abraco, Ralph.


On Fri, Jun 21, 2019 at 4:22 PM Vanderlei Nemitz 
wrote:

> Pessoal, fiquei confuso com a seguinte questão:
>
> Distribuindo-se aleatoriamente 7 bolas iguais em 3 caixas diferentes, qual
> é a probabilidade de que uma delas contenha exatamente 4 bolas?
>
> Como as bolas são iguais, existem 36 maneiras de alocar as bolas nas
> caixas, que é o número de soluções naturais da equação x + y + z = 7.
> Dessas, 12 tem 4 bolas em uma das caixas e a probabilidade é 1/3.
>
> Mas...e se considerarmos que existem 3^8 maneiras de alocar as bolas?
> Nesse caso, (C7,4).(2^3).3 = 840 maneiras correspondem a 4 bolas em uma das
> caixas e a probabilidade não é 1/3.
>
> Porque as respostas são diferentes?
>
> Claro que no primeiro caso as bolas são iguais e no segundo diferentes.
>
> Mas para uma distribuição aleatória, as probabilidades não deveriam ser
> iguais?
> Alguém com os olhos fechados, colocando as bolas nas caixas não teria a
> mesma chance em qualquer caso?
>
> Fiquei confuso...
>
> Errei alguma conta ou não?
>
> Muito obrigado,
>
> Vanderlei
>
> --
> Esta mensagem foi verificada pelo sistema de antivírus e
> acredita-se estar livre de perigo.

-- 
Esta mensagem foi verificada pelo sistema de antiv�rus e
 acredita-se estar livre de perigo.



Re: [obm-l] Bug em probabilidade

2019-06-21 Por tôpico Ralph Teixeira
P.S.: Engracado, eu tambem digitei 3^8... VOCE ME LEVOU PARA O MAU CAMINHO!
:D :D :D

On Fri, Jun 21, 2019 at 4:36 PM Ralph Teixeira  wrote:

> Oi, Vanderlei.
>
> Minha frase predileta, razão de 90% das confusões que fazemos (todos nos,
> inclusive eu!) em probabilidade:
>
> "NEM TODO ESPACO EH EQUIPROVAVEL"
> ou, traduzindo
> "SOH PORQUE TEM N MANEIRAS DE ALGO ACONTECER, NAO SIGNIFICA QUE TODAS AS
> MANEIRAS TEM PROBABILIDADE 1/N".
>
> Hm, ok, talvez eu tenha exagerado no CAPS, mas acho que vale a pena. Por
> conta disso, muita gente (eu tambem!) erra problemas de probabilidade, em
> varios niveis:
>
> a) Obvio: "Eu posso ganhar a mega-sena amanha, ou nao. 50% de chance.
> oops!"
> b) Menos obvio: "A soma desses dois dados pode dar de 2 a 12... Entao cada
> numero tem 1/11 de probabilidade oops!"
> c) Super sutis: "Tem 2 portas fechadas nestes show, entao cada uma tem 50%
> de chande de ter um bode oops!"
>
> ---///---
>
> No seu caso: o que significa "distribuir aleatoriamente" as bolas? Para
> mim, o mais natural eh pegar cada bola, escolher para ela uma das 3 caixas
> (com probabilidade 1/3 para cada, que nao estah explicito no enunciado mas
> parece razoavel)... Entao a sua segunda interpretacao eh a correta, existem
> 3^8 maneiras EQUIPROVAVEIS de colocar as bolas nas caixas.
>
> Note como a outra maneira estah furada: a gente acha mesmo que
> (x,y,z)=(7,0,0) tem a mesma probabilidade de ocorrer que (x,y,z)=(3,2,2)?
> Nao, a segunda tripla eh bem mais provavel! Entao nao faz sentido contar
> numero de solucoes ali.
>
> Abraco, Ralph.
>
> ---///---
>
> P.S.: Eu culpo o jeito que a gente introduz probabilidade no ensino
> medio O problema eh muitos livros e professores escrevem assim, em
> letras enormes, cercado por um quadrinho para a gente decorar:
>
> * Probabilidade = Numero de Casos Favoraveis / Numero de Casos
> Possiveis **
>
> E isto eh FALSO, porque ele devia escrever antes "SE OS CASOS TEM A MESMA
> PROBABILIDADE"
>
> Abraco, Ralph.
>
>
> On Fri, Jun 21, 2019 at 4:22 PM Vanderlei Nemitz 
> wrote:
>
>> Pessoal, fiquei confuso com a seguinte questão:
>>
>> Distribuindo-se aleatoriamente 7 bolas iguais em 3 caixas diferentes,
>> qual é a probabilidade de que uma delas contenha exatamente 4 bolas?
>>
>> Como as bolas são iguais, existem 36 maneiras de alocar as bolas nas
>> caixas, que é o número de soluções naturais da equação x + y + z = 7.
>> Dessas, 12 tem 4 bolas em uma das caixas e a probabilidade é 1/3.
>>
>> Mas...e se considerarmos que existem 3^8 maneiras de alocar as bolas?
>> Nesse caso, (C7,4).(2^3).3 = 840 maneiras correspondem a 4 bolas em uma das
>> caixas e a probabilidade não é 1/3.
>>
>> Porque as respostas são diferentes?
>>
>> Claro que no primeiro caso as bolas são iguais e no segundo diferentes.
>>
>> Mas para uma distribuição aleatória, as probabilidades não deveriam ser
>> iguais?
>> Alguém com os olhos fechados, colocando as bolas nas caixas não teria a
>> mesma chance em qualquer caso?
>>
>> Fiquei confuso...
>>
>> Errei alguma conta ou não?
>>
>> Muito obrigado,
>>
>> Vanderlei
>>
>> --
>> Esta mensagem foi verificada pelo sistema de antivírus e
>> acredita-se estar livre de perigo.
>
>

-- 
Esta mensagem foi verificada pelo sistema de antiv�rus e
 acredita-se estar livre de perigo.



Re: [obm-l] desigualdades

2019-06-10 Por tôpico Ralph Teixeira
Ah, errei sim! Poderia ser a≥b≥c≥d≤a, claro! :-(

On Mon, Jun 10, 2019, 21:55 Ralph Teixeira  wrote:

> Uma ideia: cada uma das 4 frações é <1... Se você mostrar que duas delas
> são ≤ 1/2, acabou o problema.
>
> Então, se a≤b≤c então a/(a+b)≤a/(a+a)=1/2, e idem para b/(b+c). De fato,
> se houver 3 números consecutivos em ordem crescente na lista cíclica
> (a,b,c,d), este argumento mata o problema.
>
> Agora, para não ter 3 em ordem crescente no ciclo, você vai ter que ter
> a≤b≥c≤d≥a (ou exatamente o contrário disso tudo, que é análogo). Mas então
> a/(a+b)≤1/2 e c/(c+d)≤1/2 também!
>
> Abraço, Ralph.
>
> P.S.: meu raciocínio parece ter muita "folga", errei algo? Alguém sabe se
> o máximo daquela expressão está perto de 3 mesmo?
>
>
>
> On Mon, Jun 10, 2019 at 11:12 AM Carlos Monteiro <
> cacacarlosalberto1...@gmail.com> wrote:
>
>> Prove que se a, b, c, d são reais positivos, então
>> a/(a+b)  +  b/(b+c) +  c/(c+d)   +   d/(d+a)  <  3
>>
>> --
>> Esta mensagem foi verificada pelo sistema de antivírus e
>> acredita-se estar livre de perigo.
>
>

-- 
Esta mensagem foi verificada pelo sistema de antiv�rus e
 acredita-se estar livre de perigo.



Re: [obm-l] desigualdades

2019-06-10 Por tôpico Ralph Teixeira
Uma ideia: cada uma das 4 frações é <1... Se você mostrar que duas delas
são ≤ 1/2, acabou o problema.

Então, se a≤b≤c então a/(a+b)≤a/(a+a)=1/2, e idem para b/(b+c). De fato, se
houver 3 números consecutivos em ordem crescente na lista cíclica
(a,b,c,d), este argumento mata o problema.

Agora, para não ter 3 em ordem crescente no ciclo, você vai ter que ter
a≤b≥c≤d≥a (ou exatamente o contrário disso tudo, que é análogo). Mas então
a/(a+b)≤1/2 e c/(c+d)≤1/2 também!

Abraço, Ralph.

P.S.: meu raciocínio parece ter muita "folga", errei algo? Alguém sabe se o
máximo daquela expressão está perto de 3 mesmo?



On Mon, Jun 10, 2019 at 11:12 AM Carlos Monteiro <
cacacarlosalberto1...@gmail.com> wrote:

> Prove que se a, b, c, d são reais positivos, então
> a/(a+b)  +  b/(b+c) +  c/(c+d)   +   d/(d+a)  <  3
>
> --
> Esta mensagem foi verificada pelo sistema de antivírus e
> acredita-se estar livre de perigo.

-- 
Esta mensagem foi verificada pelo sistema de antiv�rus e
 acredita-se estar livre de perigo.



[obm-l] Re: [obm-l] Re: [obm-l] Re: [obm-l] Re: [obm-l] Pontuação de Campeonato

2019-06-07 Por tôpico Ralph Teixeira
Pois é, por serem 4 rebaixados, a chave é olhar com carinho para o 17o
time, que é o primeiro rebaixado. Por isso tem que olhar 17 times naquela
conta!

Por exemplo, o pior caso possível, em que seu time vai bem pra caramba mas
rebaixa assim mesmo, é o caso em que tem 17 times bons (incluindo o seu) e
3 times horrorosos, que dão seus pontos para os outros 17 -- vide o exemplo
que explicitei agora há pouco.

Abraço, Ralph.

On Fri, Jun 7, 2019 at 9:44 AM Mauricio de Araujo <
mauricio.de.ara...@gmail.com> wrote:

> Não seria, 16  ao invés de 17 a ser considerado nas contas? Afinal "4"
> times descem...
>
> Att,
> __
> Mauricio de Araujo
> 
>
>
> Em sex, 7 de jun de 2019 às 07:44, Jeferson Almir <
> jefersonram...@gmail.com> escreveu:
>
>> Valeu Ralph !!
>> Só pra terminar eu precisaria exibir uma tabela que 33 pontos não é
>> suficiente??? Abraço.
>>
>> Em sex, 7 de jun de 2019 às 00:22, Ralph Teixeira 
>> escreveu:
>>
>>> RESPOSTA: 34 pontos.
>>>
>>> Quando o campeonato termina, os 17 melhores times jogaram 17x16/2=17x8
>>> partidas entre si, mais 17x3 partidas com os ultimos 3 times. Assim, esses
>>> 17 "melhores" times tem acesso a, no maximo, 17x11x3 pontos, ou seja, 33
>>> pontos cada um na media (no maximo!).
>>>
>>> Isso significa que, se voce faz 34 pontos, com certeza escapa do
>>> rebaixamento (eh impossivel que os 17 primeiros times tenham >=34 pontos
>>> cada).
>>>
>>> Isto dito, com MUITO azar pode ser que algum time com 33 pontos seja
>>> rebaixado. Basta imaginar que os 17 primeiros ganham TODOS os jogos dos 3
>>> outros, e entre si cada um dos 17 primeiros ganha 8 e perde 8 (para mostrar
>>> categoricamente que isso eh possivel, ponha os 17 times num circulo e
>>> imagine que cada time ganha dos 8 mais proximos no sentido horario, e perde
>>> dos outros 8). Entao voce tem uma tabela onde 17 times terminam com (cada
>>> um) 11 vitorias e 8 derrotas, ou seja, 33 pontos cada, e um deles seria
>>> rebaixado (em algum criterio de desempate).
>>>
>>> Abraco, Ralph.
>>>
>>> On Thu, Jun 6, 2019 at 10:44 PM Jeferson Almir 
>>> wrote:
>>>
>>>> Qual a pontuação mínima de um campeonato com 20 times  para que um time
>>>> fique livre do rebaixamento( 4 últimos times  descem ) sabendo que
>>>> cada time joga com todos os outros somente uma única vez??. E que vitória
>>>> vale 3 pontos empate vale 1 ponto.
>>>>
>>>> --
>>>> Esta mensagem foi verificada pelo sistema de antivírus e
>>>> acredita-se estar livre de perigo.
>>>
>>>
>>> --
>>> Esta mensagem foi verificada pelo sistema de antivírus e
>>> acredita-se estar livre de perigo.
>>
>>
>> --
>> Esta mensagem foi verificada pelo sistema de antivírus e
>> acredita-se estar livre de perigo.
>
>
> --
> Esta mensagem foi verificada pelo sistema de antivírus e
> acredita-se estar livre de perigo.

-- 
Esta mensagem foi verificada pelo sistema de antiv�rus e
 acredita-se estar livre de perigo.



[obm-l] Re: [obm-l] Re: [obm-l] Re: [obm-l] Pontuação de Campeonato

2019-06-07 Por tôpico Ralph Teixeira
Sim... eu esbocei o argumento ali nos parênteses, deixa eu escrever melhor:

Numere os times assim: T1, T2, T3, ..., T17, R1, R2, R3.

O resultado dos jogos será assim:
a) Ti sempre ganha de Rj (i=1,..,17; j=1,2,3);
b) Ti ganha de Tj se, e somente se, (j-i mod 17) < 8, onde (j-i mod 17) é o
resto **ENTRE 0 e 16** de j-i na divisão por 17
(Em outras palavras:
T1 ganha de T2, T3,... T9;
T2 ganha de T3, T4, ..., T10;
 ...
T13 ganha de T14, T15, T16, T17, T1, T2, T3 e T4.)
c) Os jogos entre os Ri não fazem diferença; faça eles todos empatarem, se
desejar! :D

Deste jeito, cada Ti ganha exatamente 11 partidas (8 dos times bons, e 3
dos Vasc... huh, quero dizer, dos Rj :D :D :D). Então serão 17 times com 33
pontos cada, e algum rebaixa.

Abraço, Ralph.

On Fri, Jun 7, 2019 at 7:44 AM Jeferson Almir 
wrote:

> Valeu Ralph !!
> Só pra terminar eu precisaria exibir uma tabela que 33 pontos não é
> suficiente??? Abraço.
>
> Em sex, 7 de jun de 2019 às 00:22, Ralph Teixeira 
> escreveu:
>
>> RESPOSTA: 34 pontos.
>>
>> Quando o campeonato termina, os 17 melhores times jogaram 17x16/2=17x8
>> partidas entre si, mais 17x3 partidas com os ultimos 3 times. Assim, esses
>> 17 "melhores" times tem acesso a, no maximo, 17x11x3 pontos, ou seja, 33
>> pontos cada um na media (no maximo!).
>>
>> Isso significa que, se voce faz 34 pontos, com certeza escapa do
>> rebaixamento (eh impossivel que os 17 primeiros times tenham >=34 pontos
>> cada).
>>
>> Isto dito, com MUITO azar pode ser que algum time com 33 pontos seja
>> rebaixado. Basta imaginar que os 17 primeiros ganham TODOS os jogos dos 3
>> outros, e entre si cada um dos 17 primeiros ganha 8 e perde 8 (para mostrar
>> categoricamente que isso eh possivel, ponha os 17 times num circulo e
>> imagine que cada time ganha dos 8 mais proximos no sentido horario, e perde
>> dos outros 8). Entao voce tem uma tabela onde 17 times terminam com (cada
>> um) 11 vitorias e 8 derrotas, ou seja, 33 pontos cada, e um deles seria
>> rebaixado (em algum criterio de desempate).
>>
>> Abraco, Ralph.
>>
>> On Thu, Jun 6, 2019 at 10:44 PM Jeferson Almir 
>> wrote:
>>
>>> Qual a pontuação mínima de um campeonato com 20 times  para que um time
>>> fique livre do rebaixamento( 4 últimos times  descem ) sabendo que cada
>>> time joga com todos os outros somente uma única vez??. E que vitória vale 3
>>> pontos empate vale 1 ponto.
>>>
>>> --
>>> Esta mensagem foi verificada pelo sistema de antivírus e
>>> acredita-se estar livre de perigo.
>>
>>
>> --
>> Esta mensagem foi verificada pelo sistema de antivírus e
>> acredita-se estar livre de perigo.
>
>
> --
> Esta mensagem foi verificada pelo sistema de antivírus e
> acredita-se estar livre de perigo.

-- 
Esta mensagem foi verificada pelo sistema de antiv�rus e
 acredita-se estar livre de perigo.



[obm-l] Re: [obm-l] Pontuação de Campeonato

2019-06-06 Por tôpico Ralph Teixeira
RESPOSTA: 34 pontos.

Quando o campeonato termina, os 17 melhores times jogaram 17x16/2=17x8
partidas entre si, mais 17x3 partidas com os ultimos 3 times. Assim, esses
17 "melhores" times tem acesso a, no maximo, 17x11x3 pontos, ou seja, 33
pontos cada um na media (no maximo!).

Isso significa que, se voce faz 34 pontos, com certeza escapa do
rebaixamento (eh impossivel que os 17 primeiros times tenham >=34 pontos
cada).

Isto dito, com MUITO azar pode ser que algum time com 33 pontos seja
rebaixado. Basta imaginar que os 17 primeiros ganham TODOS os jogos dos 3
outros, e entre si cada um dos 17 primeiros ganha 8 e perde 8 (para mostrar
categoricamente que isso eh possivel, ponha os 17 times num circulo e
imagine que cada time ganha dos 8 mais proximos no sentido horario, e perde
dos outros 8). Entao voce tem uma tabela onde 17 times terminam com (cada
um) 11 vitorias e 8 derrotas, ou seja, 33 pontos cada, e um deles seria
rebaixado (em algum criterio de desempate).

Abraco, Ralph.

On Thu, Jun 6, 2019 at 10:44 PM Jeferson Almir 
wrote:

> Qual a pontuação mínima de um campeonato com 20 times  para que um time
> fique livre do rebaixamento( 4 últimos times  descem ) sabendo que cada
> time joga com todos os outros somente uma única vez??. E que vitória vale 3
> pontos empate vale 1 ponto.
>
> --
> Esta mensagem foi verificada pelo sistema de antivírus e
> acredita-se estar livre de perigo.

-- 
Esta mensagem foi verificada pelo sistema de antiv�rus e
 acredita-se estar livre de perigo.



Re: [obm-l] Determinante

2019-06-06 Por tôpico Ralph Teixeira
Grande Secco!

Sim, voce tem razao, obrigado! :D

Abraco, Ralph.

On Wed, Jun 5, 2019 at 10:32 PM Matheus Secco 
wrote:

> Oi, Ralph, acho que você quis dizer trocar a linha 3 por essa combinação
> linear que colocou.
> Você só pode trocar uma linha por ela mais uma combinação linear das
> *outras*, certo?
>
> Abraços
>
> Em qua, 5 de jun de 2019 22:20, Ralph Teixeira 
> escreveu:
>
>> As propriedades importantes aqui sao:
>>
>> -- O determinante nao muda se voce trocar uma linha (ou coluna) por uma
>> combinacao linear dela com as outras;
>> -- O determinante eh linear em CADA linha (ou coluna); em particular, se
>> uma linha eh divisivel por 13, voce pode "fatorar" este 13 desta linha para
>> fora do determinante.
>>
>> Entao, experimente trocar a linha L1 por 100*L1+10*L2+L3... Agora use o
>> que voce tinha visto para "tirar" o 13 da primeira linha, e o que sobra eh
>> claramente um inteiro.
>>
>> Abraco, Ralph.
>>
>> On Wed, Jun 5, 2019 at 9:49 PM Daniel da Silva <
>> danielrochadasi...@icloud.com> wrote:
>>
>>> Boa noite pessoal,
>>>
>>> Não estou conseguindo um argumento para essa questão:
>>>
>>> Mostrar sem desenvolver que o determinate de:
>>> 1  2  5
>>> 6  7  4
>>> 9  3  6
>>>
>>> É divisível por 13.
>>>
>>> Reparei que 169, 273, 546 são divisíveis por 13, mas não consegui
>>> pensar em nada para usar isso.
>>>
>>> Obrigado,
>>> Daniel
>>> --
>>> Esta mensagem foi verificada pelo sistema de antivírus e
>>>  acredita-se estar livre de perigo.
>>>
>>>
>>> =
>>> Instruções para entrar na lista, sair da lista e usar a lista em
>>> http://www.mat.puc-rio.br/~obmlistas/obm-l.html
>>> =
>>>
>>
>> --
>> Esta mensagem foi verificada pelo sistema de antivírus e
>> acredita-se estar livre de perigo.
>
>
> --
> Esta mensagem foi verificada pelo sistema de antivírus e
> acredita-se estar livre de perigo.

-- 
Esta mensagem foi verificada pelo sistema de antiv�rus e
 acredita-se estar livre de perigo.



Re: [obm-l] Determinante

2019-06-05 Por tôpico Ralph Teixeira
As propriedades importantes aqui sao:

-- O determinante nao muda se voce trocar uma linha (ou coluna) por uma
combinacao linear dela com as outras;
-- O determinante eh linear em CADA linha (ou coluna); em particular, se
uma linha eh divisivel por 13, voce pode "fatorar" este 13 desta linha para
fora do determinante.

Entao, experimente trocar a linha L1 por 100*L1+10*L2+L3... Agora use o que
voce tinha visto para "tirar" o 13 da primeira linha, e o que sobra eh
claramente um inteiro.

Abraco, Ralph.

On Wed, Jun 5, 2019 at 9:49 PM Daniel da Silva <
danielrochadasi...@icloud.com> wrote:

> Boa noite pessoal,
>
> Não estou conseguindo um argumento para essa questão:
>
> Mostrar sem desenvolver que o determinate de:
> 1  2  5
> 6  7  4
> 9  3  6
>
> É divisível por 13.
>
> Reparei que 169, 273, 546 são divisíveis por 13, mas não consegui
> pensar em nada para usar isso.
>
> Obrigado,
> Daniel
> --
> Esta mensagem foi verificada pelo sistema de antivírus e
>  acredita-se estar livre de perigo.
>
>
> =
> Instruções para entrar na lista, sair da lista e usar a lista em
> http://www.mat.puc-rio.br/~obmlistas/obm-l.html
> =
>

-- 
Esta mensagem foi verificada pelo sistema de antiv�rus e
 acredita-se estar livre de perigo.



[obm-l] Re: [obm-l] Combinatória

2019-06-03 Por tôpico Ralph Teixeira
C(4,2)=6 não é múltiplo de 4.

(Se n fosse primo, o que você disse seria verdade.)

On Mon, Jun 3, 2019 at 9:59 AM israelmchrisostomo <
israelmchrisost...@gmail.com> wrote:

>
>
> Ola pessoal .Seja o binomio (n escolhe k) é possível dizer que esse
> binomio é múltiplo de n excero para k=0 e n=k
> Enviado do meu smartphone Samsung Galaxy.
>
> --
> Esta mensagem foi verificada pelo sistema de antivírus e
> acredita-se estar livre de perigo.
>

-- 
Esta mensagem foi verificada pelo sistema de antiv�rus e
 acredita-se estar livre de perigo.



Re: [obm-l] Probabilidade

2019-05-28 Por tôpico Ralph Teixeira
Aqui um artigo bem completo sobre o assunto:
https://en.wikipedia.org/wiki/Boy_or_Girl_paradox

Abraco, Ralph.

On Tue, May 28, 2019 at 7:02 PM Pedro José  wrote:

> Boa noite!
> Creio que o a palavra "outro", implica que os dois devam ser do sexo
> masculino. O enunciado poderia ter ajudado com a palavra também para dar
> ênfase. Mas creio que "outro" já é suficiente.
>
> Saudações,
> PJMS
>
> Em ter, 28 de mai de 2019 às 18:17, Rodrigo Ângelo 
> escreveu:
>
>> A velha história do problema mal formulado
>>
>> Eu concordo 100% com a interpretação do Pedro, mas analisando o texto do
>> problema, também cabe espaço para a seguinte interpretação:
>>
>> João e Maria tem dois filhos: A e B, e sabe-se que *um dos filhos* é um
>> menino, ou seja,A é menino ou B é menino. Se P(A é menino) = 0,5*, *é
>> correto afirmar que
>>  P(B é menino | "A é menino ou B é menino" && P(A é menino) = 0,5) é
>> igual a  ...?
>>
>> Neste caso, a probabilidade de B ser menino muda por causa da conjunção
>> das duas informações que a gente tem:
>> - Pelo menos um deles é menino
>> - A tem 50% de chance de ser menino
>>
>> Atenciosamente,
>> Rodrigo de Castro Ângelo
>>
>>
>> Em ter, 28 de mai de 2019 às 17:31, Pedro Angelo 
>> escreveu:
>>
>>> Eu acho que o enunciado foi bem claro. Num primeiro momento, o
>>> enunciado fala "sabe-se que *um* dos filhos é um menino". Em seguida,
>>> ele pergunta "qual a probabilidade de *o outro* ser menino". Os termos
>>> "um" no primeiro momento e "o outro" no final estão especificando os
>>> filhos, então a resposta é 1/2. A pergunta que está sendo feita é
>>> "qual a probabilidade do segundo filho ser H sabendo que o primeiro é
>>> H", ao invés de "qual a probabilidade de ambos serem H sabendo que um
>>> deles é H".
>>>
>>> Le mar. 28 mai 2019 à 17:03, matematica10complicada
>>>  a écrit :
>>> >
>>> > Valeu Ralph, obrigado, eu tive a mesma interpretação, e acredito que o
>>> problema podia ter sido melhor elaborado.
>>> > Mas de qualquer forma, obrigado.
>>> >
>>> >
>>> > Um abraço do
>>> > Douglas Oliveira.
>>> >
>>> > Em ter, 28 de mai de 2019 16:36, Ralph Teixeira 
>>> escreveu:
>>> >>
>>> >> Problema de difícil resposta, depende de como interpretar a frase "um
>>> dos filhos é menino"... Do ponto de vista probabilísitco, depende de como a
>>> informação de que um deles é menino foi obtida.
>>> >>
>>> >> Vou supor que os filhos estão numa certa ordem, e colocar H para
>>> menino e M para menina. Então, vou dizer que os filhos são "HM" se o
>>> primeiro for homem e o segundo for mulher.
>>> >>
>>> >> Portanto, **a priori**, o universo de possibilidades seria
>>> {HH,HM,MH,MM}. Supondo que a probabilidade de cada um ser H é 50% (está no
>>> enunciado), e supondo que os sexos dos dois filhos são independentes um do
>>> outro (não está no enunciado, mas não é uma hipótese tão horrível... além
>>> disso, sem ela a gente não sai do lugar), então cada um desses 4 eventos
>>> tem probabilidade 1/4=25%.
>>> >>
>>> >> Até aqui, o problema não costuma ser muito polêmico... Agora,
>>> surpreendentemente, as coisas complicam:
>>> >>
>>> >> ---///---
>>> >> INTERPRETAÇÃO #1:
>>> >> Se você ler a frase estritamente, sabemos que PELO MENOS UM DELES é
>>> menino, sem saber qual. Ou seja, o "novo universo" é {HH,HM,MH}, já que não
>>> pode ser MM. Então a probabilidade do outro ser menino também é a
>>> probabilidade de ambos serem meninos, ou seja, queremos a probabilidade de
>>> HH neste novo universo. A reposta é 1/3.
>>> >>
>>> >> Se você quiser ser mais formal: seja "A" o evento "pelo menos um é
>>> menino", e "B" o evento "o outro também é menino". Então A={HH,HM,MH} e
>>> B={HH}. O que se pediu foi a probabilidade condicional:
>>> >> Pr(B|A)=Pr(A e B) / Pr(A) = (1/4)/(3/4)=1/3.
>>> >>
>>> >> Esta interpretação é razoável por exemplo se a informação foi obtida
>>> da seguinte forma: você perguntou ao João se ele tem *algum* filho menino,
>>> e ele disse "sim, tenho!", sem dar a menor indicação de qual é o menino.
>>&g

Re: [obm-l] Probabilidade

2019-05-28 Por tôpico Ralph Teixeira
Problema de difícil resposta, depende de como interpretar a frase "um dos
filhos é menino"... Do ponto de vista probabilísitco, depende de como a
informação de que um deles é menino foi obtida.

Vou supor que os filhos estão numa certa ordem, e colocar H para menino e M
para menina. Então, vou dizer que os filhos são "HM" se o primeiro for
homem e o segundo for mulher.

Portanto, **a priori**, o universo de possibilidades seria {HH,HM,MH,MM}.
Supondo que a probabilidade de cada um ser H é 50% (está no enunciado), e
supondo que os sexos dos dois filhos são independentes um do outro (não
está no enunciado, mas não é uma hipótese tão horrível... além disso, sem
ela a gente não sai do lugar), então cada um desses 4 eventos tem
probabilidade 1/4=25%.

Até aqui, o problema não costuma ser muito polêmico... Agora,
surpreendentemente, as coisas complicam:

---///---
INTERPRETAÇÃO #1:
Se você ler a frase estritamente, sabemos que PELO MENOS UM DELES é menino,
sem saber qual. Ou seja, o "novo universo" é {HH,HM,MH}, já que não pode
ser MM. Então a probabilidade do outro ser menino também é a probabilidade
de ambos serem meninos, ou seja, queremos a probabilidade de HH neste novo
universo. A reposta é 1/3.

Se você quiser ser mais formal: seja "A" o evento "pelo menos um é menino",
e "B" o evento "o outro também é menino". Então A={HH,HM,MH} e B={HH}. O
que se pediu foi a probabilidade condicional:
Pr(B|A)=Pr(A e B) / Pr(A) = (1/4)/(3/4)=1/3.

Esta interpretação é razoável por exemplo se a informação foi obtida da
seguinte forma: você perguntou ao João se ele tem *algum* filho menino, e
ele disse "sim, tenho!", sem dar a menor indicação de qual é o menino.
---///---
INTERPRETAÇÂO #2:
Mas pode ser que "um" em "um deles é menino" seja um ESPECÍFICO, o que é
diferente! Tipo, se você pergunta ao João se o filho **mais velho** é
menino, e ele diz "Sim, o mais velho é menino", agora eu sei QUEM é esse
menino, e isto afeta sim a probabilidade!

Agora o novo universo seria {HH,MH}, então a probabilidade do mais novo ser
menino é 1/2 -- que é a resposta que quase todo mundo dá a este problema,
porque na hora de calcular a probabilidade todo mundo imagina que um filho
ESPECÍFICO é menino, e se pergunta sobre o outro.
---///---

Qual a resposta correta? De novo, depende do que você entende por "um
dos filhos é menino", que em Português é ligeiramente vago. Eu fico com a
interpretação #1, que acho que é mais estritamente o que foi dito no
enunciado.

Abraço, Ralph.






On Tue, May 28, 2019 at 11:35 AM matematica10complicada <
profdouglaso.del...@gmail.com> wrote:

> Olá amigos, o que acham desse problema?
>
> Qual seria a resposta?
>
> João e Maria tem dois filhos, e sabe-se que um dos filhos é um menino. Se
> a probabilidade de um filho ser do sexo masculino é igual a 50%, é correto
> afirmar que a probabilidade de o outro filho do casal ser um menino é igual
> a:
>
>
> Att
> Douglas Oliveira.
>
>
>
>
> --
> Esta mensagem foi verificada pelo sistema de antivírus e
> acredita-se estar livre de perigo.

-- 
Esta mensagem foi verificada pelo sistema de antiv�rus e
 acredita-se estar livre de perigo.



Re: [obm-l] Probabilidade de Moedas

2019-05-27 Por tôpico Ralph Teixeira
Ah, esse eh um problema classico e MUITO bonito! :D

Seja A o evento: "Tem mais caras nas vermelhas do que coroas nas pretas."
Seja B o evento: "Tem mais coroas nas vermelhas do que caras nas pretas."

Queremos p(A). Note que p(A)=p(B) por simetria (moedas honestas, nada muda
se trocar cara por coroa).

Enfim, note que um, e apenas um dos dois eventos A e B acontece! De fato,
sejam KV (e KP) o numero de caras vermelhas (e pretas) obtidas, CV (e CP) o
numero de coroas vermelhas (e pretas). Em particular, CV+KV=n+1 e CP+KP=n.
Assim:

-- Se ambos A e B falhassem, teriamos KV<=CP e CV<=KP, portanto
KV+CV<=KP+CP, absurdo.
-- Se A e B ambos valessem, teriamos KV>CP e CV>KP, o que nos inteiros
implica KV>=CP+1 e CV>=KP+1, ou seja KV+CV>=KP+CP+2, absurdo tambem!

Em suma, p(A)=p(B)=1/2, ou seja, a resposta eh 50%.

(Note que para ter esta solucao sem conta quase nenhuma, os numeros TEM QUE
SER n+1 e n. Se fossem n moedas de cada, ou n+2 de uma e n da outra,
teriamos que fazer umas contas mais complicadas...)

Abraco, Ralph.


On Mon, May 27, 2019 at 9:45 PM Jeferson Almir 
wrote:

> Dispomos de 2n+1 moedas honestas, sendo n+1 vermelhas e n pretas. Uma
> pessoa arremessa as 2n+1 moedas simultaneamente, qual a probabilidade de se
> obter MAIS caras de vermelhas do que coroas de pretas ?
> Peço ajuda nesse problema.
>
> --
> Esta mensagem foi verificada pelo sistema de antivírus e
> acredita-se estar livre de perigo.

-- 
Esta mensagem foi verificada pelo sistema de antiv�rus e
 acredita-se estar livre de perigo.



Re: [obm-l] Desigualdades

2019-05-25 Por tôpico Ralph Teixeira
2xy+2xz+2yz-6= (x+1)(y+z-2) + (y+1)(x+z-2) + (z+1)(x+y-2)>=0

:D






---///---





Ok, eu nao fiz assim de cara Eu primeiro defini u=x+1, v=y+1 e w=z+1.
Entao as condicoes dadas seriam:

u,v,w>=0
u+v, u+w, v+w >= 4

Entao
(u-1)(v-1)+(u-1)(w-1)+(v-1)(w-1) >= 3
vira
uv+uw+vw -2u -2v -2w >= 0
e aqui eu tentei ajeitar o lado esquerdo com as coisas dadas:
u(v+w-4)+v(u+w-4)+w(u+v-4)
e deu certo!

DEPOIS disso tudo, voltei ao problema original e traduzi essas coisas para
x,y,z para nem precisar das novas coordenadas e fingir que era um problema
de uma linha. ;D

Abraco, Ralph.





On Sat, May 25, 2019 at 7:58 AM Carlos Monteiro <
cacacarlosalberto1...@gmail.com> wrote:

> Sejam x, y e z reais satisfazendo x,y,z >= -1 e x+y >= 2, x+z >= 2, y+z >=
> 2. Prove que xy+xz+yz >= 3.
> --
> Esta mensagem foi verificada pelo sistema de antivírus e
> acredita-se estar livre de perigo.

-- 
Esta mensagem foi verificada pelo sistema de antiv�rus e
 acredita-se estar livre de perigo.



Re: [obm-l] Blocos decrescentes e maximais

2019-05-24 Por tôpico Ralph Teixeira
P.S.: Outro jeito de fazer: defina o numero de "crescencia" como o numero
de blocos CRESCENTES maximais. Mostre que, para cada permutacao, a soma do
numero de cadencia com o de "crescencia" eh n+1 (confira isto!). Entao o
somatorio das cadencias com as crescencias de todas as permutacoes dah
(n+1).n!=(n+1)!. Por simetria, a soma das cadencias eh igual aa soma das
crescencias, entao (n+1)!/2 para cada soma.

On Fri, May 24, 2019 at 8:13 PM Ralph Teixeira  wrote:

> Note que um bloco acaba em a_k se, e somente se, a_k numero de cadencia de uma permutacao eh exatamente igual ao numero de vezes
> em que a_k
> Por exemplo, em σ = (4, 2, 1, 5, 6, 3), temos apenas dois pares
> consecutivos crescentes, a saber, 1<5 e 5<6. Portanto, temos 2+1=3 blocos.
>
> Entao contar o numero de cadencia de todas as permutacoes eh o mesmo que
> contar todas as sequencias consecutivas crescentes (do tipo a_k todas elas, e somar (n!) (por causa daquele "mais um " em cada permutacao).
>
> Isto dito, se voce olhar todos os pares consecutivos que aparecem em todas
> as permutacoes, por simetria, metade deles serah crescente e metade
> decrescente. Ou seja, de todos os (n-1).n! pares consecutivos, temos
> (n-1).n!/2 que sao crescentes.
>
> Entao a resposta eh (n-1).n!/2+n!=(n+1)!/2.
>
> Abraco, Ralph.
>
> On Fri, May 24, 2019 at 5:23 PM Carlos Monteiro <
> cacacarlosalberto1...@gmail.com> wrote:
>
>> Sejam n um inteiro positivo e σ = (a1, . . . , an) uma permutação de {1,
>> . . . , n}. O número
>> de cadência de σ é o número de blocos decrescentes maximais. Por exemplo,
>> se n = 6 e
>> σ = (4, 2, 1, 5, 6, 3), então o número de cadência de σ é 3, pois σ
>> possui 3 blocos (4, 2, 1), (5),
>> (6, 3) descrescentes e maximais. Note que os blocos (4, 2) e (2, 1) são
>> decrescentes, mas não
>> são maximais, já que estão contidos no bloco (4, 2, 1).
>> Calcule a soma das cadências de todas as permutações de {1, . . . , n}.
>>
>> --
>> Esta mensagem foi verificada pelo sistema de antivírus e
>> acredita-se estar livre de perigo.
>
>

-- 
Esta mensagem foi verificada pelo sistema de antiv�rus e
 acredita-se estar livre de perigo.



Re: [obm-l] Blocos decrescentes e maximais

2019-05-24 Por tôpico Ralph Teixeira
Note que um bloco acaba em a_k se, e somente se, a_k wrote:

> Sejam n um inteiro positivo e σ = (a1, . . . , an) uma permutação de {1, .
> . . , n}. O número
> de cadência de σ é o número de blocos decrescentes maximais. Por exemplo,
> se n = 6 e
> σ = (4, 2, 1, 5, 6, 3), então o número de cadência de σ é 3, pois σ possui
> 3 blocos (4, 2, 1), (5),
> (6, 3) descrescentes e maximais. Note que os blocos (4, 2) e (2, 1) são
> decrescentes, mas não
> são maximais, já que estão contidos no bloco (4, 2, 1).
> Calcule a soma das cadências de todas as permutações de {1, . . . , n}.
>
> --
> Esta mensagem foi verificada pelo sistema de antivírus e
> acredita-se estar livre de perigo.

-- 
Esta mensagem foi verificada pelo sistema de antiv�rus e
 acredita-se estar livre de perigo.



[obm-l] Re: [obm-l] Re: [obm-l] Re: [obm-l] Soluções inteiras

2019-05-17 Por tôpico Ralph Teixeira
Oops, sim, eu errei, voce consertou, era y=6a+p e x=5a+p. Tambem poderia
ser y=6a-p e x=5a-p, mas entao x vai ser negativo, o que pode ser obtido
diretamente das solucoes positivas trocando sinais.

Na pratica, a ideia eh a seguinte: tome (11+2raiz(30))^n para varios
valores de n.

Por exemplo, para n=2, temos:
(11+2raiz(30))^2=241+44raiz(30)
Eu afirmo que p=241 e a=44 tambem servem -- confira que p^2-30a^2=1 de novo!
Colocando isto na quadratica do y, voce acha y, e depois acha x:
y=6a+p=505 e x=y-a=461
(Confira que este cara serve! Tambem tem as solucoes trocando os sinais de
x e y, mas nao vou falar muito delas, vou me concentrar nas positivas, as
outras vem por tais trocas de sinal.)

Para n=3:
(11+2raiz(30))^3=5291+966raiz(30). Entao p=5291, a=966 servem, levando a
y=6a+p=11087 e x=y-a=10121

Para cada n, voce terah uma escolha de p e a, e portanto uma escolha de x e
y... Ou seja, o problema tem infinitas solucoes!

(Sim, o metodo vao sempre gerar p=impar e a=par, entao todas as solucoes
serao x=5a+p=impar e y=6a+p=impar)

As respostas que faltam -- (A) POR QUE isso gera solucoes? (B) Esta ideia
ACHA TODAS as solucoes (bom, com as devidas trocas de sinal que sempre
existem)?

---///---
(A) POR QUE gera solucoes?

Lema: Seja m um numero natural positivo que NAO EH quadrado perfeito.
Considere a Equacao de Pell p^2-m.a^2=1 (normalmente o pessoal usa x e y,
mas vou usar p e a para ficar parecido com minha notacao ali em cima). Se
p=p0 e a=a0 eh uma solucao, entao p=pn e a=an tambem eh, onde pn e an sao
inteiros determinados pela formula
(p0+a0.raiz(m))^n=pn+an.raiz(m).

Demonstracao: Fatorando, vem que (p0+a0.raiz(m)).(p0-a0.raiz(m))=1.

Elevando os dois lados a potencia n, vem (p0+a0.raiz(m))^n .
(p0-a0.raiz(m))^n =1.

Mas o primeiro fator do produto eh exatamente pn+raiz(m).an (pela nossa
definicao de an e pn), e nao eh dificil ver que, se m nao eh quadrado
perfeito, o segundo fator tem de ser exatamente o "conjugado" pn-raiz(m).an
(abra o binomio de Newton se necessario para enxergar isso).

Portanto, temos (pn+an.raiz(m)).(pn-an.raiz(m))=1, ou seja pn^2-m.an^2=1
tambem!

---///---

Repito, esse lema mostra que o processo GERA solucoes, mas falta mostrar
(B): que existe alguma especie de "solucao fundamental" que gera TODAS as
outras por este processo... Bom, a resposta eh SIM, esta solucao
"fundamental" existe, e eu **acho** que neste caso eh (11,2)... mas para
mostrar isso, veja o artigo da Eureka, acho que este E-mail ficou muito
comprdo... :D

Abraco, Ralph.

On Fri, May 17, 2019 at 6:05 PM Pedro José  wrote:

> Boa tarde!
> Se fizer s=x^2 e t=y^2 temos 6s-5t=1; cuja solução é s=5a+1 e t=6a+1, com
> a >=0. Então, x e y não deveriam ser ímpares?
> As soluções que achei:
> (-1,-1);(-1,1);(1,-1) e (1,1) essa no lápis. para a=0
> (-21,-23);(-21,23);(21,-23) e (21,23) com auxílio do Excel para a=88.
>
> Não sei se há mais soluções. Porém creio que as soluções são em 2Z+1.
>
> Se fosse:
> y=6a+p
> x=5a+p
> (p,a)=(11,2) daria a solução (x,y) = (21,23)
>
> Não consegui alcançar seu pensamento. Mas creio que pela solução da
> equação diofantina, tanto x como y deveriam ser ímpares.
>
> Saudações,
> PJMS
>
>
>
> Em sex, 17 de mai de 2019 às 14:02, Ralph Teixeira 
> escreveu:
>
>> Escreva x=y-a com a inteiro. Ficamos com y^2-12ay+6a^2-1=0.
>>
>> Pense nisso como uma quadrática em y. Para haver soluções inteiras, o
>> discriminante tem que ser quadrado perfeito:
>>
>> D = 144a^2 -4 (6a^2-1) = 120a^2+4 = 4p^2 (tem que ser par, por isso já
>> coloquei o 4)
>> 30a^2+1=p^2
>> p^2-30a^2=1
>>
>> Isso é uma Equação de Pell, cuja teoria não é difícil, mas está bem além
>> das congruências... Veja o artigo do Caminha na Eureka 7, por exemplo:
>> https://www.obm.org.br/content/uploads/2017/01/eureka7.pdf
>>
>> Em suma, você acha uma solução fundamental (acho que é (p,a)=(11,2) neste
>> caso) e gerar as outras olhando para
>> (11+2raiz(30))^n  (para cada n=0,1,2,..., a parte inteira disso dá um
>> possível p, o coeficiente de raiz(30) dá um possível a).
>>
>> Enfim, encontrados p e a, teremos:
>> y=6a+-2p
>> x=5a+-2p
>>
>> Ou seja, creio haver infinitas soluções!
>>
>> Abraço, Ralph.
>>
>> On Fri, May 17, 2019 at 7:25 AM matematica10complicada <
>> profdouglaso.del...@gmail.com> wrote:
>>
>>> Olá meus caros, gostaria de uma ajuda sem usar congruência para resolver
>>> e achar todos os inteiros da equação
>>>  6x^2-5y^2=1.
>>>
>>>
>>> Obrigado e grande abraço.
>>> Douglas oliveira
>>>
>>>
>>> --
>>> Esta mensagem foi verificada pelo sistema de antivírus e
>>> acredita-se estar livre de perigo.
>>
>>
>> --
>> Esta mensagem foi verificada pelo sistema de antivírus e
>> acredita-se estar livre de perigo.
>
>
> --
> Esta mensagem foi verificada pelo sistema de antivírus e
> acredita-se estar livre de perigo.

-- 
Esta mensagem foi verificada pelo sistema de antiv�rus e
 acredita-se estar livre de perigo.



[obm-l] Re: [obm-l] Soluções inteiras

2019-05-17 Por tôpico Ralph Teixeira
Escreva x=y-a com a inteiro. Ficamos com y^2-12ay+6a^2-1=0.

Pense nisso como uma quadrática em y. Para haver soluções inteiras, o
discriminante tem que ser quadrado perfeito:

D = 144a^2 -4 (6a^2-1) = 120a^2+4 = 4p^2 (tem que ser par, por isso já
coloquei o 4)
30a^2+1=p^2
p^2-30a^2=1

Isso é uma Equação de Pell, cuja teoria não é difícil, mas está bem além
das congruências... Veja o artigo do Caminha na Eureka 7, por exemplo:
https://www.obm.org.br/content/uploads/2017/01/eureka7.pdf

Em suma, você acha uma solução fundamental (acho que é (p,a)=(11,2) neste
caso) e gerar as outras olhando para
(11+2raiz(30))^n  (para cada n=0,1,2,..., a parte inteira disso dá um
possível p, o coeficiente de raiz(30) dá um possível a).

Enfim, encontrados p e a, teremos:
y=6a+-2p
x=5a+-2p

Ou seja, creio haver infinitas soluções!

Abraço, Ralph.

On Fri, May 17, 2019 at 7:25 AM matematica10complicada <
profdouglaso.del...@gmail.com> wrote:

> Olá meus caros, gostaria de uma ajuda sem usar congruência para resolver e
> achar todos os inteiros da equação
>  6x^2-5y^2=1.
>
>
> Obrigado e grande abraço.
> Douglas oliveira
>
>
> --
> Esta mensagem foi verificada pelo sistema de antivírus e
> acredita-se estar livre de perigo.

-- 
Esta mensagem foi verificada pelo sistema de antiv�rus e
 acredita-se estar livre de perigo.



[obm-l] Re: [obm-l] Re: [obm-l] Re: [obm-l] Polinômio Inteiro

2019-04-30 Por tôpico Ralph Teixeira
Oi, Jeferson.

Sua ideia funciona: comece com P(x,y)=(y+ix)^180+1. Como voce disse,
P(s,c)=0 onde c=cos1º e s=sin1º.

Agora olhemos para a parte real deste polinomio: ateh dah para escrever
explicitamente, mas eu vou me limitar a dizer que eh algo do tipo
R(x,y)=SOMA(a_k*y^(2k)*x^(180-2k))+1 onde os a_k sao coeficientes inteiros
(que incluem os sinais negativos que porventura venham das potencias pares
de i).

Enfim, o truque eh perceber que todas as potencias de y ali sao pares. Ou
seja, trocando y^2 por 1-x^2 em todos os termos, voce vai ficar com um
polinomio P(x) que satisfaz o que voce quer.

Abraco, Ralph.

On Tue, Apr 30, 2019 at 6:02 PM Jeferson Almir 
wrote:

> Eu estou tentando através do binômio de Newton obter tal polinômio pegando
> a parte real do número complexo. Sen1º não é transcende.
>
> Em ter, 30 de abr de 2019 às 17:35, Pedro José 
> escreveu:
>
>> Boa tarde!
>> Não compreendi
>> sen1º é um número transcendente, ou não??
>>
>> Sds,
>> PJMS
>>
>>
>> Em ter, 30 de abr de 2019 às 14:30, Jeferson Almir <
>> jefersonram...@gmail.com> escreveu:
>>
>>> Mostre que existe um polinômio P(x) de coeficientes inteiros que possui
>>> sen1º como raiz de P(x).
>>>
>>>
>>> Eu tentei usar a forma exponencial de números complexos (Euler)
>>>  e^(i.pi/180) = cos1º + isen1º e depois elevando 180 e pegando a parte real
>>> do complexo mas ainda não consegui .
>>>
>>> --
>>> Esta mensagem foi verificada pelo sistema de antivírus e
>>> acredita-se estar livre de perigo.
>>
>>
>> --
>> Esta mensagem foi verificada pelo sistema de antivírus e
>> acredita-se estar livre de perigo.
>
>
> --
> Esta mensagem foi verificada pelo sistema de antivírus e
> acredita-se estar livre de perigo.

-- 
Esta mensagem foi verificada pelo sistema de antiv�rus e
 acredita-se estar livre de perigo.



[obm-l] Re: [obm-l] Re: [obm-l] Re: [obm-l] Combinatória (permutações)

2019-04-25 Por tôpico Ralph Teixeira
Sim, voce tem razao, os termos em portugues nao estao corretos... A ideia
(que eu nao escrevi) eh que cada sequencia que foi contada multiplas vezes
num termo vai ser descontada nos termos seguintes, por isso tudo funciona.
Vejamos se dah para expressar melhor o que foi de fato feito...

Considere os conjuntos A1={permutacoes que mantem o 2 fixo},
A2={permutacoes que mantem o 4 fixo}, A3={permutacoes que mantem o 6 fixo}
e A4={permutacoes que mantem o 8 fixo}.

Entao, agora sim, por inclusao-exclusao:

 #(A1UA2UA3UA4) = #(A1) + #(A2) + #(A3) + #(A4) - #(A1UA2) - #(A1UA3) -
#(A1UA4) - ... -#(A3UA4) + #(A1UA2UA3) + #(A1UA2UA4) + #(A1UA3UA4) +
#(A2UA3UA4) - #(A1UA2UA3UA4)

Mas, por simetria, cada termo desses soh depende de QUANTOS conjuntos
aparecem naquela uniao -- mais exatamente, os termos com 1 indice sao 8!
(pois fixamos um termo, os outros podem ser permutados de qualquer jeito),
os com 2 indices sao 7! (dois numeros fixos, os outros como quisermos), e
assim por diante. Portanto:

 #(A1UA2UA3UA4) =  C(4,1).8! - C(4,2) . 7! + C(4,3).6! - 5!

Bom, isso sao as permutacoes em que PELO MENOS um dos 4 numeros estah fixo.
Como queremos o contrario (NENHUM fica fixo), a reposta eh:

#(permutacoes) = 9! - (4. 8! - C(4,2) . 7! + 4 . 6! - 5!)

Acho que agora ficou *bem* melhor escrito! :D

Abraco, Ralph.



On Thu, Apr 25, 2019 at 4:44 PM Pedro Lazéra  wrote:

> Ralph, eu fiquei com uma dúvida.
>
> Apesar de a sua resposta bater com o gabarito, os termos que você
> expressou com números batem mesmo com os termos que você expressou com
> palavras? Por exemplo, "#(permutações que pelo menos 1 dos pares fica no
> lugar)" =  "4.8!" ? Eu tenho a impressão que "4.8!" é maior, porque contou,
> por exemplo, a sequência (1,2,3,4,5,6,7,8,9) mais de uma vez.
>
> Por exemplo, se o mesmo enunciado fosse aplicado a 1,2,3,4, você acharia
> como resposta, por analogia, 4! - 2*3! + C(2,2)*2! = 14, que é a resposta
> certa, mas "#(permutações que pelo menos 1 dos pares fica no lugar)" = 10,
> que é diferente de "2*3!". Não? Neste caso, 2*3! conta (1,2,3,4) e
> (3,2,1,4) duas vezes.
>
> Abraços,
> Pedro
>
> On Thu, Apr 25, 2019 at 2:32 PM Ralph Teixeira  wrote:
>
>> Por inclusão-exclusão, eu achei:
>>
>> #(permutações) = #(total) - #(permutações em que pelo menos um dos pares
>> fica no lugar) + #(permutações que pelo menos 2 dos pares ficam no lugar) -
>> #(permutações que pelo menos 3 dos pares ficam no lugar) + #(permutações em
>> que todos os pares ficam no lugar)
>>  = 9! - 4.8! + C(4,2).7! - C(4,3). 6! +5! = 229080
>>
>> On Thu, Apr 25, 2019 at 7:03 AM Vanderlei Nemitz 
>> wrote:
>>
>>> Bom dia!
>>>
>>> Resolvi a questão a seguir, encontrei como resposta 229080, mas
>>> encontrei essa resposta em uma lista e 133800 em outra. Gostaria de
>>> confirmar qual é a correta. Para mim, 133800 é o número de permutações em
>>> que pelo menos um algarismo par permanece em sua posição original.
>>>
>>> Muito obrigado!
>>>
>>>
>>> *De quantas maneiras podemos permutar os inteiros 1, 2, 3, 4, 5, 6, 7,
>>> 8, 9 de forma que nenhum inteiro par fique em sua posição natural?*
>>>
>>> --
>>> Esta mensagem foi verificada pelo sistema de antivírus e
>>> acredita-se estar livre de perigo.
>>
>>
>> --
>> Esta mensagem foi verificada pelo sistema de antivírus e
>> acredita-se estar livre de perigo.
>
>
> --
> Esta mensagem foi verificada pelo sistema de antivírus e
> acredita-se estar livre de perigo.

-- 
Esta mensagem foi verificada pelo sistema de antiv�rus e
 acredita-se estar livre de perigo.



[obm-l] Re: [obm-l] Combinatória (permutações)

2019-04-25 Por tôpico Ralph Teixeira
Por inclusão-exclusão, eu achei:

#(permutações) = #(total) - #(permutações em que pelo menos um dos pares
fica no lugar) + #(permutações que pelo menos 2 dos pares ficam no lugar) -
#(permutações que pelo menos 3 dos pares ficam no lugar) + #(permutações em
que todos os pares ficam no lugar)
 = 9! - 4.8! + C(4,2).7! - C(4,3). 6! +5! = 229080

On Thu, Apr 25, 2019 at 7:03 AM Vanderlei Nemitz 
wrote:

> Bom dia!
>
> Resolvi a questão a seguir, encontrei como resposta 229080, mas encontrei
> essa resposta em uma lista e 133800 em outra. Gostaria de confirmar qual é
> a correta. Para mim, 133800 é o número de permutações em que pelo menos um
> algarismo par permanece em sua posição original.
>
> Muito obrigado!
>
>
> *De quantas maneiras podemos permutar os inteiros 1, 2, 3, 4, 5, 6, 7, 8,
> 9 de forma que nenhum inteiro par fique em sua posição natural?*
>
> --
> Esta mensagem foi verificada pelo sistema de antivírus e
> acredita-se estar livre de perigo.

-- 
Esta mensagem foi verificada pelo sistema de antiv�rus e
 acredita-se estar livre de perigo.



Re: [obm-l] Algebra solucoes reais.

2019-04-04 Por tôpico Ralph Teixeira
Vou completar a ideia do Pedro Jose.

Se fosse x^1980+y^1980=1, como ele disse, claramente deveriamos ter
|x|,|y|<=1.

Mas entao |x^2|<=1, |xy|<=1 e |y^2|<=1. Entao |x^2+xy+y^2|<=3, e a
igualdade soh valeria se fossem |x^2|=|y^2|=|xy|=1, que rapidamente ve-se
que nao presta.

Abraco, Ralph.

On Thu, Apr 4, 2019 at 11:01 AM Pedro José  wrote:

> Bom dia!
> No momento bastante atarefado.
> Se x=y ==> 2x^1980=1 ==> x=y= (1/2)^(1/1980) ou x=y=(-1/2)^(1/1980)
> Se x<>y
> (x^3-y^3) = 3(x-y)
> (x-y)(x^2+xy+y^2)= 3(x-y)  ==> (x^2+xy+y^2) = 3.
> Agora creio que seja achar a matriz diagonal. muda as coordenadas e
> identificar a cônica e mostrar que essa cônica não intercepta o disco
> aberto x^2+y^2 < 1; pois para atender
> x^1980 + y^1980 = 1 ==> |x| e |y| no intervalo ]0,1[
>
> Se sobrar um tempo faço a transformação, até seria bom para relembrar.
>
> Sds,
> PJMS
>
>
> Em qua, 3 de abr de 2019 às 15:36, matematica10complicada <
> profdouglaso.del...@gmail.com> escreveu:
>
>> Encontre  todas as soluções reais do sistema abaixo.
>>
>> x^3-3x=y^3-3y e x^1980+y^1980=1.
>>
>> Douglas Oliveira.
>>
>> --
>> Esta mensagem foi verificada pelo sistema de antivírus e
>> acredita-se estar livre de perigo.
>
>
> --
> Esta mensagem foi verificada pelo sistema de antivírus e
> acredita-se estar livre de perigo.

-- 
Esta mensagem foi verificada pelo sistema de antiv�rus e
 acredita-se estar livre de perigo.



[obm-l] Re: [obm-l] Polinômio

2019-03-20 Por tôpico Ralph Teixeira
Um jeito de fazer eh ir direto no polinomio interpolador de Lagrange e
fazer as contas.
(https://en.wikipedia.org/wiki/Lagrange_polynomial)

Outro jeito que parece mais elegante (mas no final das contas eh a mesma
coisa): o polinomio xP(x)-1 tem grau n+1 e todos aqueles n+1 numeros sao
raizes dele. Entao:

xP(x)-1=a(x-1)(x-2)...(x-2^n)

Tomando x=0 ali em cima, descobre-se que a(-1)(-2)...(-2^n)=-1.

Portanto, P(x)=[a(x-1)...(x-2^n)+1] / x, o que pode parecer estranho, mas
lembre que o numerador vai ter um "x" para fatorar e cortar com o
denominador (o a que calculamos garante exatamente isso!). Entao P(0) eh o
termo indepenente do que sobra depois que cortar o x, isto eh, a soma dos
coeficientes em x do numerador... bom, para obter um termo em x, voce vai
ter que multiplicar todos a*(-1)*(-2)*...*(-2^n), EXCETO uma das potencias
(pois o x entra no lugar dela), isto eh, sao varios termos do tipo
(-1)/(-2^k).x. Portanto o que voce quer eh

P(0)=SOMA (1/2^k) = 2 - 1/2^n

pois a soma eh uma PG de razao 1/2 com n+1 termos.

Abraco, Ralph.




On Wed, Mar 20, 2019 at 11:08 PM Vanderlei Nemitz 
wrote:

> Alguém tem uma dica para esse problema?
> Muito obrigado!
>
> *Seja P(x) é um polinômio de grau n tal que P(k) = 1/k para k = 1, 2, 2^2,
> ..., 2^n. Determine o valor de P(0) em função de n.*
>
> --
> Esta mensagem foi verificada pelo sistema de antivírus e
> acredita-se estar livre de perigo.

-- 
Esta mensagem foi verificada pelo sistema de antiv�rus e
 acredita-se estar livre de perigo.



[obm-l] Re: [obm-l] Re: Característica

2019-03-11 Por tôpico Ralph Teixeira
Pelo Teorema do Núcleo e da Imagem, se o núcleo de uma transformação linear
de R^3 em R^3 tem dimensão 2 (é assim que interpreto os dados do problema),
então o posto (isso que é característica, suponho?) tem que ser 1.

Abraço, Ralph.

On Mon, Mar 11, 2019 at 6:52 AM Vanderlei Nemitz 
wrote:

> Bom dia, pessoal!
> Alguém chegou a pensar nessa questão que enviei?
> Eu consigo apenas pensar que o posto da matriz dos coeficientes do sistema
> é 1.
>
> Muito obrigado!
>
> Em ter, 5 de mar de 2019 às 07:06, Vanderlei Nemitz 
> escreveu:
>
>> Na seguinte questão, consigo pensar em um sistema com 2 variáveis livres,
>> mas não com apenas 1. De acordo com o gabarito, a resposta é c.
>>
>> É possível um sistema que satisfaça esse enunciado?
>>
>>
>> Obrigado!
>>
>>
>> *Um sistema linear homogêneo de três equações e três incógnitas admite
>> como soluções os ternos (1, 3, 5) e (2, 4, 5), mas não o terno (1, 1, 1). A
>> característica do sistema é:*
>>
>> *a) 0  b) 1  c) 2  d) 3  e) nenhuma das
>> respostas anteriores*
>>
>>
>> 
>>  Livre
>> de vírus. www.avast.com
>> .
>>
>> <#m_8429769216578852236_m_7884441411983649374_DAB4FAD8-2DD7-40BB-A1B8-4E2AA1F9FDF2>
>>
>
> --
> Esta mensagem foi verificada pelo sistema de antivírus e
> acredita-se estar livre de perigo.

-- 
Esta mensagem foi verificada pelo sistema de antiv�rus e
 acredita-se estar livre de perigo.



[obm-l] Re: [obm-l] Re: [obm-l] Torneio das Cidades ( Número mínimo de Tentativas )

2019-02-24 Por tôpico Ralph Teixeira
Provando que n+2 eh otimo no item (a):

Suponha que voce arrumou um jeito de testar n+1 pares e garantir que
funciona. Vou mostrar que tah errado.

Afinal, nos seus n+1 pares tem 2n+2 baterias, contando repeticoes.
Portanto, alguma das 2n+1 baterias aparece em (pelo menos) dois dos seus
pares. Marque essa bateria com a letra R e olhe para os OUTROS pares, fora
esses 2 (ou mais)... Destes n-1 (ou menos) pares, escolha uma bateria de
cada e marque com a letra R tambem (pode ser repetido se quiser). No final
das contas, marcamos com a letra R um total de 1+(n-1)=n baterias (ou
menos, se tiver repeticoes). Pois bem, se essas baterias fossem as ruins,
os seus pares NAO achariam uma combinacao boa, entao seu jeito nao GARANTE
que a lanterna vai acender.

Agora falta mostrar que n+3 eh otimo para (b) -- ou arrumar um jeito melhor.

Abraco, Ralph.

On Sun, Feb 24, 2019 at 6:49 PM Ralph Teixeira  wrote:

> Era n+2 para o item (a); o que eu falei ali foi um jeito de fazer em n+3
> para o item (b), melhor que o n+4 que eu tinha falado antes.
>
> Abraco, Ralph.
>
> On Sun, Feb 24, 2019 at 5:14 PM Pedro José  wrote:
>
>> Boa tarde!
>> Ralph,
>> também não sei se é ótimo. Postei a resposta para provocar.
>> Só que você afirmou ter um método melhor, mas não foi. Para a) com n+2
>> estava garantido acender. Com o que você propôs podemos atingir n+3. Então
>> não foi melhor.
>> Ou talvez não tenha compreendido.
>>
>> Sds,
>> PJMS
>>
>> Em dom, 24 de fev de 2019 às 15:27, Pedro José 
>> escreveu:
>>
>>> Boa tarde!
>>> a) Você pode ter n baterias com falha e n+1 sem estar em modo de falha.
>>> Seu pior caso é sempre pegar uma ruim e uma boa, pois aí você nem acende
>>> a lâmpada nem esgota rapidamente as em modo de falha.
>>> Quando você fizer n tentativas, a que sobrou é boa.
>>> E em cada lote tem uma boa e uma em falha.
>>> Você pode demorar duas se na primeira escolher a que está em modo de
>>> falha. Portanto, você poderá gastar n+2 tentativas no máximo.
>>> Isto é interpretando o problema da seguinte forma. qual o menor número
>>> de tentativas que garanta a funcionabilidade da lâmpada. Pois a lâmpada
>>> pode funcionar com apenas uma tentaiva.
>>>
>>> b) O pior caso é n tentativas. Sendo sempre uma ruim e uma boa.
>>> Pegando dois lotes temos três chances para não acender a lâmpada RB, BR
>>> e RR e uma para acender BB. Portanto, no pior caso teríamos n+4 tentativas.
>>>
>>> Creio que seja isso.
>>> Todavia, recomendaria um voltímetro.
>>>
>>> Sds,
>>> PJMS
>>>
>>>
>>>
>>> Em dom, 24 de fev de 2019 às 11:40, Jeferson Almir <
>>> jefersonram...@gmail.com> escreveu:
>>>
>>>> Peço ajuda aos amigos da lista, sei que existe um problemas da obm
>>>> "parecido",  aguardo dicas ou soluções. Eu tentei formar um grafo de
>>>> tentativas e penso como otimizar ele.
>>>>
>>>> a.) Existem 2n + 1 (n> 2) baterias. Não sabemos quais baterias são
>>>> boas e quais são ruins, mas sabemos que o número de baterias boas é maior
>>>> do que o número de baterias ruins. Uma lâmpada usa duas baterias e só
>>>> funciona se ambas forem boas. Qual é o menor número de tentativas
>>>> suficientes para fazer a lâmpada funcionar?
>>>>
>>>> b.) O mesmo problema, mas o número total de baterias é 2n (n> 2) e os
>>>> números de baterias boas e ruins são iguais.
>>>>
>>>>
>>>>
>>>> --
>>>> Esta mensagem foi verificada pelo sistema de antivírus e
>>>> acredita-se estar livre de perigo.
>>>
>>>
>> --
>> Esta mensagem foi verificada pelo sistema de antivírus e
>> acredita-se estar livre de perigo.
>
>

-- 
Esta mensagem foi verificada pelo sistema de antiv�rus e
 acredita-se estar livre de perigo.



[obm-l] Re: [obm-l] Re: [obm-l] Torneio das Cidades ( Número mínimo de Tentativas )

2019-02-24 Por tôpico Ralph Teixeira
Era n+2 para o item (a); o que eu falei ali foi um jeito de fazer em n+3
para o item (b), melhor que o n+4 que eu tinha falado antes.

Abraco, Ralph.

On Sun, Feb 24, 2019 at 5:14 PM Pedro José  wrote:

> Boa tarde!
> Ralph,
> também não sei se é ótimo. Postei a resposta para provocar.
> Só que você afirmou ter um método melhor, mas não foi. Para a) com n+2
> estava garantido acender. Com o que você propôs podemos atingir n+3. Então
> não foi melhor.
> Ou talvez não tenha compreendido.
>
> Sds,
> PJMS
>
> Em dom, 24 de fev de 2019 às 15:27, Pedro José 
> escreveu:
>
>> Boa tarde!
>> a) Você pode ter n baterias com falha e n+1 sem estar em modo de falha.
>> Seu pior caso é sempre pegar uma ruim e uma boa, pois aí você nem acende
>> a lâmpada nem esgota rapidamente as em modo de falha.
>> Quando você fizer n tentativas, a que sobrou é boa.
>> E em cada lote tem uma boa e uma em falha.
>> Você pode demorar duas se na primeira escolher a que está em modo de
>> falha. Portanto, você poderá gastar n+2 tentativas no máximo.
>> Isto é interpretando o problema da seguinte forma. qual o menor número de
>> tentativas que garanta a funcionabilidade da lâmpada. Pois a lâmpada
>> pode funcionar com apenas uma tentaiva.
>>
>> b) O pior caso é n tentativas. Sendo sempre uma ruim e uma boa.
>> Pegando dois lotes temos três chances para não acender a lâmpada RB, BR e
>> RR e uma para acender BB. Portanto, no pior caso teríamos n+4 tentativas.
>>
>> Creio que seja isso.
>> Todavia, recomendaria um voltímetro.
>>
>> Sds,
>> PJMS
>>
>>
>>
>> Em dom, 24 de fev de 2019 às 11:40, Jeferson Almir <
>> jefersonram...@gmail.com> escreveu:
>>
>>> Peço ajuda aos amigos da lista, sei que existe um problemas da obm
>>> "parecido",  aguardo dicas ou soluções. Eu tentei formar um grafo de
>>> tentativas e penso como otimizar ele.
>>>
>>> a.) Existem 2n + 1 (n> 2) baterias. Não sabemos quais baterias são boas
>>> e quais são ruins, mas sabemos que o número de baterias boas é maior do que
>>> o número de baterias ruins. Uma lâmpada usa duas baterias e só funciona
>>> se ambas forem boas. Qual é o menor número de tentativas suficientes
>>> para fazer a lâmpada funcionar?
>>>
>>> b.) O mesmo problema, mas o número total de baterias é 2n (n> 2) e os
>>> números de baterias boas e ruins são iguais.
>>>
>>>
>>>
>>> --
>>> Esta mensagem foi verificada pelo sistema de antivírus e
>>> acredita-se estar livre de perigo.
>>
>>
> --
> Esta mensagem foi verificada pelo sistema de antivírus e
> acredita-se estar livre de perigo.

-- 
Esta mensagem foi verificada pelo sistema de antiv�rus e
 acredita-se estar livre de perigo.



[obm-l] Re: [obm-l] Torneio das Cidades ( Número mínimo de Tentativas )

2019-02-24 Por tôpico Ralph Teixeira
b) Tem um jeito melhor: comece testando ab,ac,bc. Se der errado, significa
que tem (pelo menos) 2 ruins aqui, entao sobram 2n-3 baterias onde tem mais
boas do que ruins... O que eh exatamente o item (a)! (Bom, trocando n por
n-2). Entao pelo metodo da (a), conseguimos um par de baterias boas em
(n-2)+2=n tentativas extra. Total: n+3 tentativas!

Agora, isso eh o otimo?

Abraco, Ralph.

On Sun, Feb 24, 2019 at 2:55 PM Ralph Teixeira  wrote:

> Bom, tenho estrategias boas, mas tem que provar que sao otimas (ou arrumar
> uma melhor):
>
> a) Faca n tentativas com 2 baterias cada, sem intersecao. Se nenhuma
> dessas tentativas der certo, voce eh muito azarado e cada par tinha
> exatamente uma bateria ruim. Bom, entao a bateria que nao foi testada tem
> que ser boa -- tente-a com cada uma do ultimo par testado, vai ter que
> funcionar. Total: n+2 tentativas.
>
> b) Idem ao de cima... Mas se apos n tentativas nao deu certo, tome os dois
> ultimos pares (digamos, ab e cd) e tente as combinacoes com 1 bateria de
> cada (digo: ac, ad, bc, bd). Uma dessas vai ter que funcionar. Total: n+4
> tentativas.
> (Sinto um cheirinho de que esta aqui NAO eh otima Mas tem que pensar
> mais.)
>
> Abraco, Ralph.
>
> On Sun, Feb 24, 2019 at 11:39 AM Jeferson Almir 
> wrote:
>
>> Peço ajuda aos amigos da lista, sei que existe um problemas da obm
>> "parecido",  aguardo dicas ou soluções. Eu tentei formar um grafo de
>> tentativas e penso como otimizar ele.
>>
>> a.) Existem 2n + 1 (n> 2) baterias. Não sabemos quais baterias são boas
>> e quais são ruins, mas sabemos que o número de baterias boas é maior do que
>> o número de baterias ruins. Uma lâmpada usa duas baterias e só funciona
>> se ambas forem boas. Qual é o menor número de tentativas suficientes
>> para fazer a lâmpada funcionar?
>>
>> b.) O mesmo problema, mas o número total de baterias é 2n (n> 2) e os
>> números de baterias boas e ruins são iguais.
>>
>>
>>
>> --
>> Esta mensagem foi verificada pelo sistema de antivírus e
>> acredita-se estar livre de perigo.
>
>

-- 
Esta mensagem foi verificada pelo sistema de antiv�rus e
 acredita-se estar livre de perigo.



[obm-l] Re: [obm-l] Torneio das Cidades ( Número mínimo de Tentativas )

2019-02-24 Por tôpico Ralph Teixeira
Bom, tenho estrategias boas, mas tem que provar que sao otimas (ou arrumar
uma melhor):

a) Faca n tentativas com 2 baterias cada, sem intersecao. Se nenhuma dessas
tentativas der certo, voce eh muito azarado e cada par tinha exatamente uma
bateria ruim. Bom, entao a bateria que nao foi testada tem que ser boa --
tente-a com cada uma do ultimo par testado, vai ter que funcionar. Total:
n+2 tentativas.

b) Idem ao de cima... Mas se apos n tentativas nao deu certo, tome os dois
ultimos pares (digamos, ab e cd) e tente as combinacoes com 1 bateria de
cada (digo: ac, ad, bc, bd). Uma dessas vai ter que funcionar. Total: n+4
tentativas.
(Sinto um cheirinho de que esta aqui NAO eh otima Mas tem que pensar
mais.)

Abraco, Ralph.

On Sun, Feb 24, 2019 at 11:39 AM Jeferson Almir 
wrote:

> Peço ajuda aos amigos da lista, sei que existe um problemas da obm
> "parecido",  aguardo dicas ou soluções. Eu tentei formar um grafo de
> tentativas e penso como otimizar ele.
>
> a.) Existem 2n + 1 (n> 2) baterias. Não sabemos quais baterias são boas e
> quais são ruins, mas sabemos que o número de baterias boas é maior do que o
> número de baterias ruins. Uma lâmpada usa duas baterias e só funciona se
> ambas forem boas. Qual é o menor número de tentativas suficientes para
> fazer a lâmpada funcionar?
>
> b.) O mesmo problema, mas o número total de baterias é 2n (n> 2) e os
> números de baterias boas e ruins são iguais.
>
>
>
> --
> Esta mensagem foi verificada pelo sistema de antivírus e
> acredita-se estar livre de perigo.

-- 
Esta mensagem foi verificada pelo sistema de antiv�rus e
 acredita-se estar livre de perigo.



Re: [obm-l] algebra

2019-02-15 Por tôpico Ralph Teixeira
Tome a=x+1, b=y+1 e c=z+1.

As equacoes equivalem a:

ab=9
bc=16
ac=36

que nao sao dificeis de resolver -- multiplique duas delas, divida pela
outra, use que a,b,c>0 Fica a=9/2; b=2; c=8.

Entao x=7/2; y=1 e z=7, e daqui voce tira o que precisar.

Abraco, Ralph.




On Fri, Feb 15, 2019 at 7:54 PM marcone augusto araújo borges <
marconeborge...@hotmail.com> wrote:

> assuma que x, y, z são numeros positivos tais que satisfazem as equações
> abaixo . Determine o valor de xyz + x+y+z
>
> x+y+xy = 8
> y+z+yz = 15
> z+x+ zx = 35
>
> Eu encontrei  xyz + x+y+z + xy +xz + yz = 71, mas...
> o gabarito diz que a resposta é 36
>
> --
> Esta mensagem foi verificada pelo sistema de antivírus e
> acredita-se estar livre de perigo.
>

-- 
Esta mensagem foi verificada pelo sistema de antiv�rus e
 acredita-se estar livre de perigo.



[obm-l] Re: [obm-l] Sequência de Fibonacci

2019-02-13 Por tôpico Ralph Teixeira
Bom, quase qualquer argumento seria *formalizado* usando inducao... Mas se
voce quer apenas uma explicacao convincente que nao use explicitamente o
metodo da inducao finita, tem uma legal (usando que determinante do produto
de matrizes eh o produto dos determinantes!), assim:

Escreva Vn=[Fn;Fn-1], um vetor 2x1. Queremos calcular o determinante da
matriz  Am=[V2m,Vm] (cujas colunas sao V2m e Vm).

Mas em geral [Fn+1, Fn]=[1 1; 1 0][Fn,Fn-1], ou seja, Vn+1=B*Vn onde B=[1
1; 1 0] eh uma matriz 2x2. Note que detB=-1.

Assim Am=[V2m, Vm]=B*[V2m-1,Vm-1]=B*B*[V2m-2,Vm-2]=...=(B^(m-1)) * [Vm+1,V1
]=(B^(m-1))*[Fm+1, Fm; 1,0].

Portanto, detAm=(detB)^(m-1) * det[Fm+1, Fm; 1,0], e acabou.

Abraco, Ralph.

On Wed, Feb 13, 2019 at 9:25 PM Jeferson Almir 
wrote:

> Como provar esse resultado de fibonacci que não seja por indução ??
> F_2m •F_m-1 - F_2m-1•F_m = (-1)^m•F_m
>
> --
> Esta mensagem foi verificada pelo sistema de antivírus e
> acredita-se estar livre de perigo.

-- 
Esta mensagem foi verificada pelo sistema de antiv�rus e
 acredita-se estar livre de perigo.



[obm-l] Re: [obm-l] Re: [obm-l] quadrilátero

2019-02-10 Por tôpico Ralph Teixeira
Infelizmente, o quadrilatero nao pode ser assim. Se 3 e 4 formassem 90
graus, uma das diagonais seria o diametro; como a outra eh perpendicular, o
quadrilatero teria dois pares de lados iguais e isto nao vale. :(

Abraco, Ralph.

On Sun, Feb 10, 2019 at 9:28 PM Pacini Bores  wrote:

> Olá Marcone,
>
> Pense assim: se supusermos que que dois lados consecutivos são 3 e 4 e o
> ângulo entre eles de 90º , então   uma das diagonais será 5 e, tomando
>  x e 2 formando 90º e com diagonais perpendiculares, teremos o quadrilátero
> inscritível. As projeções dos lados  3 e 4, como sendo 1,8  e 3,2 você
> observará que raiz quadrada de 23 será um valor possível. Verifique.
>
> Pacini
>
> Em 09/02/2019 11:26, marcone augusto araújo borges escreveu:
>
> um quadrilátero tem diagonais perpendiculares e as medidas de três dos
> seus lados são 2, 3 e 4. A medida do outro lado pode ser:
>
> a) raiz(20)  b) raiz(21)   c) raiz(22)  d) raiz(23) e) nda
>
> Desculpem pela simplicidade da questão, mas não estou conseguindo
>
> --
> Esta mensagem foi verificada pelo sistema de antivírus e
> acredita-se estar livre de perigo.
>
>
>
> --
> Esta mensagem foi verificada pelo sistema de antivírus e
> acredita-se estar livre de perigo.
>

-- 
Esta mensagem foi verificada pelo sistema de antiv�rus e
 acredita-se estar livre de perigo.



[obm-l] Re: [obm-l] quadrilátero

2019-02-10 Por tôpico Ralph Teixeira
Seja ABCD o quadrilatero (lados a,b,c,d), seja O o ponto de encontro das
diagonais. Note que OA^2+OB^2+OC^2+OD^2 pode ser calculado de duas maneiras
distintas usando Pitagoras, que vao dar a^2+c^2 ou b^2+d^2 dependendo de
como agrupar os termos.

Em suma, sendo x o terceiro lado, teremos x^2+4^2=2^2+3^2 (impossivel), ou
x^2+2^2=3^2+4^2, ou x^2+3^2=2^2+4^2. Portanto as unicas possibilidades sao
sqrt(21) e sqrt(11).

Abraco, Ralph.

On Sun, Feb 10, 2019 at 8:27 PM marcone augusto araújo borges <
marconeborge...@hotmail.com> wrote:

> um quadrilátero tem diagonais perpendiculares e as medidas de três dos
> seus lados são 2, 3 e 4. A medida do outro lado pode ser:
>
> a) raiz(20)  b) raiz(21)   c) raiz(22)  d) raiz(23) e) nda
>
> Desculpem pela simplicidade da questão, mas não estou conseguindo
>
> --
> Esta mensagem foi verificada pelo sistema de antivírus e
> acredita-se estar livre de perigo.
>

-- 
Esta mensagem foi verificada pelo sistema de antiv�rus e
 acredita-se estar livre de perigo.



[obm-l] Re: [obm-l] Re: [obm-l] Re: [obm-l] Problema olimpíada de maio

2019-01-23 Por tôpico Ralph Teixeira
Sim, nao vi porque que algum resto apareceria mais do que os outros...
Achei que eu conseguiria uma funcao que levasse cada classe de restos numa
outra, mas soh consegui pareamentos. Com os dois paremntos, deu.

On Wed, Jan 23, 2019 at 10:27 AM Mauricio de Araujo <
mauricio.de.ara...@gmail.com> wrote:

> Bela solução!! mas qual foi o teu insight? Desconfiança de que havia uma
> distribuição uniforme dos restos possíveis?
> Att.
>
> Em qua, 23 de jan de 2019 às 00:47, Ralph Teixeira 
> escreveu:
>
>> Hm, tive uma ideia, confiram se funciona.
>>
>> Seja S o conjunto dos numeros obtidos pela permutacao dos digitos de 1 a
>> 7, e seja x_i a quantidade de elementos de S que deixam resto i na divisao
>> por 7 (i=0,1,2,3,4,5,6).
>>
>> Agora vamos fazer dois pareamentos. (Ou seja, vamos criar funcoes
>> f,g:S->S tal que f(f(N))=N e g(g(N))=N para todo N em S).
>>
>> PRIMEIRO: No primeiro pareamento, troque cada digito x de N=abcdefg pelo
>> digito 8-x obtendo o numero f(N) (por exemplo, o numero N=1574326 eh
>> pareado com f(N)=7314562). Claramente, f(N) esta em S, e f(f(N))=N. Como a
>> soma desses dois numeros eh N+f(N)=888, que deixa resto 1 na divisao
>> por 7, temos automaticamente que:
>> -- Se N mod 7 = 1, entao f(N) mod 7 = 0; em outras palavras para cada
>> numero N que deixa resto 1, temos exatamente um numero f(N) que deixa resto
>> 0, e vice-versa; portanto x_1=x_0.
>> -- Se N mod 7 = 2, entao f(N) mod 7 = 6; portanto x_2=x_6.
>> -- Analogamente, x_3=x_5.
>>
>> SEGUNDO: Agora, g(N) eh obtido a partir de N trocando cada digito x por
>> 7-x, EXCETO O DIGITO 7 que eh mantido. Por exemplo, se N=1754326 entao
>> g(N)=6723451. Claramente g(N) estah em S, e g(g(N))=N. Agora, N+g(N) mod 7
>> = 777 = 0 (pois aquele 7 extra pode ser jogado fora sem alterar o resto
>> modulo 7). Assim, de maneira analoga ao pareamento anterior, concluimos que:
>> -- x_1=x_6; x_2=x_5; x_3=x_4.
>>
>> Encadeando tudo, concluimos que x_0=x_1=x_2=...=x_6. Assim, o numero
>> pedido eh x_0=#(S)/7=6!.
>>
>> Abraco, Ralph.
>>
>> On Tue, Jan 22, 2019 at 9:57 PM Heitor Gama Ribeiro <
>> heitor...@hotmail.com> wrote:
>>
>>> Consideramos todos os números de 7 dígitos que se obtém permutando de
>>> todas as maneiras possíveis os dígitos de 1234567. Quantos deles são
>>> divisíveis por 7?
>>>
>>> --
>>> Esta mensagem foi verificada pelo sistema de antivírus e
>>> acredita-se estar livre de perigo.
>>>
>>
>> --
>> Esta mensagem foi verificada pelo sistema de antivírus e
>> acredita-se estar livre de perigo.
>
>
>
> --
> --
> Abraços,
> Mauricio de Araujo
> [oɾnɐɹɐ ǝp oıɔıɹnɐɯ]
>
> --
> Esta mensagem foi verificada pelo sistema de antivírus e
> acredita-se estar livre de perigo.

-- 
Esta mensagem foi verificada pelo sistema de antiv�rus e
 acredita-se estar livre de perigo.



[obm-l] Re: [obm-l] Problema olimpíada de maio

2019-01-22 Por tôpico Ralph Teixeira
Hm, tive uma ideia, confiram se funciona.

Seja S o conjunto dos numeros obtidos pela permutacao dos digitos de 1 a 7,
e seja x_i a quantidade de elementos de S que deixam resto i na divisao por
7 (i=0,1,2,3,4,5,6).

Agora vamos fazer dois pareamentos. (Ou seja, vamos criar funcoes f,g:S->S
tal que f(f(N))=N e g(g(N))=N para todo N em S).

PRIMEIRO: No primeiro pareamento, troque cada digito x de N=abcdefg pelo
digito 8-x obtendo o numero f(N) (por exemplo, o numero N=1574326 eh
pareado com f(N)=7314562). Claramente, f(N) esta em S, e f(f(N))=N. Como a
soma desses dois numeros eh N+f(N)=888, que deixa resto 1 na divisao
por 7, temos automaticamente que:
-- Se N mod 7 = 1, entao f(N) mod 7 = 0; em outras palavras para cada
numero N que deixa resto 1, temos exatamente um numero f(N) que deixa resto
0, e vice-versa; portanto x_1=x_0.
-- Se N mod 7 = 2, entao f(N) mod 7 = 6; portanto x_2=x_6.
-- Analogamente, x_3=x_5.

SEGUNDO: Agora, g(N) eh obtido a partir de N trocando cada digito x por
7-x, EXCETO O DIGITO 7 que eh mantido. Por exemplo, se N=1754326 entao
g(N)=6723451. Claramente g(N) estah em S, e g(g(N))=N. Agora, N+g(N) mod 7
= 777 = 0 (pois aquele 7 extra pode ser jogado fora sem alterar o resto
modulo 7). Assim, de maneira analoga ao pareamento anterior, concluimos que:
-- x_1=x_6; x_2=x_5; x_3=x_4.

Encadeando tudo, concluimos que x_0=x_1=x_2=...=x_6. Assim, o numero pedido
eh x_0=#(S)/7=6!.

Abraco, Ralph.

On Tue, Jan 22, 2019 at 9:57 PM Heitor Gama Ribeiro 
wrote:

> Consideramos todos os números de 7 dígitos que se obtém permutando de
> todas as maneiras possíveis os dígitos de 1234567. Quantos deles são
> divisíveis por 7?
>
> --
> Esta mensagem foi verificada pelo sistema de antivírus e
> acredita-se estar livre de perigo.
>

-- 
Esta mensagem foi verificada pelo sistema de antiv�rus e
 acredita-se estar livre de perigo.



[obm-l] Re: [obm-l] se a, b e c são números reais tais que a+b+c = 0, mostre que 2(a^4 + b^4 + c^4) é um quadrado perfeito

2019-01-15 Por tôpico Ralph Teixeira
Voce quer dizer numeros *inteiros*, eu suponho.

Porque entao voce pode usar que
2(a^4+b^4+c^4)=2(a^4+b^4+(a+b)^4)=[2(a^2+b^2+ab)]^2

Abraco, Ralph.

On Tue, Jan 15, 2019 at 11:03 PM marcone augusto araújo borges <
marconeborge...@hotmail.com> wrote:

>
>
> --
> Esta mensagem foi verificada pelo sistema de antivírus e
> acredita-se estar livre de perigo.
>

-- 
Esta mensagem foi verificada pelo sistema de antiv�rus e
 acredita-se estar livre de perigo.



Re: [obm-l] Problema Simples de Probabilidade

2018-11-21 Por tôpico Ralph Teixeira
Bolas B1,B2,B3,P1,P2,P3,P4.

Ha C(7,3)=35 maneiras igualmente provaveis de retirar 3 bolas
simultaneamente (ignoro a ordem).

Destas, tem C(3,2).C(4,1)+C(3,3).C(4,0) = 12+1=13 maneiras de tirar pelo
menos 2 brancas (12 maneiras de tirar 2 brancas e 1 reta, mais uma de tirar
3 brancas).

Entao eu acho 13/35... que nao eh 60%. Errei algo? :(

Abraco, Ralph.

On Wed, Nov 21, 2018 at 11:12 PM Luiz Antonio Rodrigues <
rodrigue...@gmail.com> wrote:

> Olá, pessoal!
> Tudo bem?
> Resolvi o seguinte problema, que é simples, de muitas maneiras e não chego
> na resposta do gabarito, que supostamente é 60%.
>
> Uma urna contém 3 bolas brancas e 4 pretas. São retiradas simultaneamente
> 3 bolas da urna. Qual a probabilidade de que pelo menos 2 bolas brancas
> sejam retiradas?
>
> Alguém pode me ajudar.
> Muito obrigado e um abraço!
> Luiz
>
> --
> Esta mensagem foi verificada pelo sistema de antivírus e
> acredita-se estar livre de perigo.

-- 
Esta mensagem foi verificada pelo sistema de antiv�rus e
 acredita-se estar livre de perigo.



Re: [obm-l] Determinante

2018-11-14 Por tôpico Ralph Teixeira
Sim, eu roubei -- como a resposta era algo ao quadrado, eu fiquei tentando
arrumar uma fatoração simples AB daquela matriz com detA=detB=a1. Bom, e A
e B teriam que ser duas matrizes nxn simples, claro...

A primeira ideia foi colocar a primeira linha de 1´s na A e a primeira
coluna de 1´s na B, para gerar o 1x1+1x1+1x1+...+1x1=n na entrada (1,1) de
AB.

Depois eu pensei que os z_n^2 da diagonal apareceriam se tanto A como B
tivessem z_n na diagonal. Não era o único jeito, mas parecia bonito e
simpático.

Depois eu me enrolei por um tempo... porque eu estava tentando montar uma
matriz nxn, dã, tinham que ser (n+1)x(n+1)! Dã! Daí veio a ideia que algum
dos 1´s da primeira linha e coluna tinham que sumir, e o resto encaixou (eu
nem esperava inicialmente que desse A=B, mas deu).

Abraço, Ralph.

On Wed, Nov 14, 2018 at 4:27 PM Claudio Buffara 
wrote:

> Bela sacada!
> Como você pensou nisso?
> O fato da resposta ser (a1)^2 foi uma pista?
>
> Pergunto porque tenho muito interesse por heurística e pela questão "de
> onde vem as idéias matemáticas?"
>
> []s,
> Claudio.
>
>
> On Tue, Nov 13, 2018 at 10:32 PM Ralph Teixeira  wrote:
>
>> Hmm... Que tal olhar para:
>>
>> 0   1   1   1  ...  1
>> 1 z1   0   0  ...  0
>> 1   0  z2  0  ...  0
>> ...
>> 1   0   0   0 ... zn
>>
>> Digo isso porque, elevando esta matriz ao quadrado...
>>
>> Abraco, Ralph.
>>
>> On Tue, Nov 13, 2018 at 3:45 PM Vanderlei Nemitz 
>> wrote:
>>
>>> Agradeço pelas tentativas. Também estou me quebrando nele, mas não
>>> consigo um padrão, apesar de ser fácil concluir o padrão com os resultados
>>> para n igual a 2 e n igual a 3.
>>>
>>>
>>> Em ter, 13 de nov de 2018 15:06, Anderson Torres <
>>> torres.anderson...@gmail.com escreveu:
>>>
>>>>
>>>>
>>>> Em seg, 12 de nov de 2018 às 22:13, Vanderlei Nemitz <
>>>> vanderma...@gmail.com> escreveu:
>>>>
>>>>> Mas será que não é possível provar genericamente?
>>>>>
>>>>
>>>> Eu tentei verificar na internet, mas não achei nada.
>>>> Deve ter algum truquinho que não estou vendo. Talvez uma diagonalizaçao
>>>> esperta...
>>>>
>>>>
>>>>
>>>>>
>>>>> Em seg, 12 de nov de 2018 21:34, Claudio Buffara <
>>>>> claudio.buff...@gmail.com escreveu:
>>>>>
>>>>>> Pruma múltipla escolha, você fez o necessário: testou casos
>>>>>> particulares e eliminou 4 alternativas.
>>>>>>
>>>>>>
>>>>>>
>>>>>> On Mon, Nov 12, 2018 at 7:57 PM Vanderlei Nemitz <
>>>>>> vanderma...@gmail.com> wrote:
>>>>>>
>>>>>>> Gostaria de uma dica na seguinte questão.
>>>>>>> Já tentei muito coisa!
>>>>>>> Desculpe as limitações para digitar o enunciado. Qualquer dúvida,
>>>>>>> estou à disposição.
>>>>>>> Muito obrigado!
>>>>>>>
>>>>>>> Sejam z1, z2, ..., zn as raízes do polinômio complexo P(z) = z^n +
>>>>>>> a(n-1).z^(n - 1) + ... + a1.z + a0, com a0 diferente de 0. Determine o
>>>>>>> valor do determinante da matriz
>>>>>>> n   z1   z2   ... zn
>>>>>>> z1  1 + z1^2 1... 1
>>>>>>> z2  1 1 + z2^2... 1
>>>>>>> 
>>>>>>> zn  1 1 1 + zn^2
>>>>>>>
>>>>>>> a) [a(n-1)]^2
>>>>>>> b) n
>>>>>>> c) 1 + a(n-1) + ... + a1 + a0
>>>>>>> d) (a1)^2
>>>>>>> e) a0
>>>>>>>
>>>>>>> Testei para um polinômio do segundo e outro do terceiro grau e
>>>>>>> obtive a alternativa d, ou seja, o coeficiente a1 elevado ao quadrado.
>>>>>>> Mas como provar?
>>>>>>>
>>>>>>> Muito obrigado!
>>>>>>>
>>>>>>>
>>>>>>> <https://www.avast.com/sig-email?utm_medium=email_source=link_campaign=sig-email_content=webmail>
>>>>>>>  Livre
>>>>>>> de vírus. www.avast.com
>>>>>>> <https://www.avast.com/sig-email?utm_medium=email_source=link_campaign=sig-email_content=webmail>.
>>>>>>>
>>>>>>> <#m_-1856527885579563443_m_1438946694965666227_m_2240385784410830415_m_-2565710904076108649_m_6162287954846621097_m_-4046382275174238934_m_-274481415220420387_m_-3160108196652599415_DAB4FAD8-2DD7-40BB-A1B8-4E2AA1F9FDF2>
>>>>>>>
>>>>>>> --
>>>>>>> Esta mensagem foi verificada pelo sistema de antivírus e
>>>>>>> acredita-se estar livre de perigo.
>>>>>>
>>>>>>
>>>>>> --
>>>>>> Esta mensagem foi verificada pelo sistema de antivírus e
>>>>>> acredita-se estar livre de perigo.
>>>>>
>>>>>
>>>>> --
>>>>> Esta mensagem foi verificada pelo sistema de antivírus e
>>>>> acredita-se estar livre de perigo.
>>>>
>>>>
>>>> --
>>>> Esta mensagem foi verificada pelo sistema de antivírus e
>>>> acredita-se estar livre de perigo.
>>>
>>>
>>> --
>>> Esta mensagem foi verificada pelo sistema de antivírus e
>>> acredita-se estar livre de perigo.
>>
>>
>> --
>> Esta mensagem foi verificada pelo sistema de antivírus e
>> acredita-se estar livre de perigo.
>
>
> --
> Esta mensagem foi verificada pelo sistema de antivírus e
> acredita-se estar livre de perigo.

-- 
Esta mensagem foi verificada pelo sistema de antiv�rus e
 acredita-se estar livre de perigo.



Re: [obm-l] Determinante

2018-11-13 Por tôpico Ralph Teixeira
Hmm... Que tal olhar para:

0   1   1   1  ...  1
1 z1   0   0  ...  0
1   0  z2  0  ...  0
...
1   0   0   0 ... zn

Digo isso porque, elevando esta matriz ao quadrado...

Abraco, Ralph.

On Tue, Nov 13, 2018 at 3:45 PM Vanderlei Nemitz 
wrote:

> Agradeço pelas tentativas. Também estou me quebrando nele, mas não consigo
> um padrão, apesar de ser fácil concluir o padrão com os resultados para n
> igual a 2 e n igual a 3.
>
>
> Em ter, 13 de nov de 2018 15:06, Anderson Torres <
> torres.anderson...@gmail.com escreveu:
>
>>
>>
>> Em seg, 12 de nov de 2018 às 22:13, Vanderlei Nemitz <
>> vanderma...@gmail.com> escreveu:
>>
>>> Mas será que não é possível provar genericamente?
>>>
>>
>> Eu tentei verificar na internet, mas não achei nada.
>> Deve ter algum truquinho que não estou vendo. Talvez uma diagonalizaçao
>> esperta...
>>
>>
>>
>>>
>>> Em seg, 12 de nov de 2018 21:34, Claudio Buffara <
>>> claudio.buff...@gmail.com escreveu:
>>>
 Pruma múltipla escolha, você fez o necessário: testou casos
 particulares e eliminou 4 alternativas.



 On Mon, Nov 12, 2018 at 7:57 PM Vanderlei Nemitz 
 wrote:

> Gostaria de uma dica na seguinte questão.
> Já tentei muito coisa!
> Desculpe as limitações para digitar o enunciado. Qualquer dúvida,
> estou à disposição.
> Muito obrigado!
>
> Sejam z1, z2, ..., zn as raízes do polinômio complexo P(z) = z^n +
> a(n-1).z^(n - 1) + ... + a1.z + a0, com a0 diferente de 0. Determine o
> valor do determinante da matriz
> n   z1   z2   ... zn
> z1  1 + z1^2 1... 1
> z2  1 1 + z2^2... 1
> 
> zn  1 1 1 + zn^2
>
> a) [a(n-1)]^2
> b) n
> c) 1 + a(n-1) + ... + a1 + a0
> d) (a1)^2
> e) a0
>
> Testei para um polinômio do segundo e outro do terceiro grau e obtive
> a alternativa d, ou seja, o coeficiente a1 elevado ao quadrado.
> Mas como provar?
>
> Muito obrigado!
>
>
> 
>  Livre
> de vírus. www.avast.com
> .
>
> <#m_2240385784410830415_m_-2565710904076108649_m_6162287954846621097_m_-4046382275174238934_m_-274481415220420387_m_-3160108196652599415_DAB4FAD8-2DD7-40BB-A1B8-4E2AA1F9FDF2>
>
> --
> Esta mensagem foi verificada pelo sistema de antivírus e
> acredita-se estar livre de perigo.


 --
 Esta mensagem foi verificada pelo sistema de antivírus e
 acredita-se estar livre de perigo.
>>>
>>>
>>> --
>>> Esta mensagem foi verificada pelo sistema de antivírus e
>>> acredita-se estar livre de perigo.
>>
>>
>> --
>> Esta mensagem foi verificada pelo sistema de antivírus e
>> acredita-se estar livre de perigo.
>
>
> --
> Esta mensagem foi verificada pelo sistema de antivírus e
> acredita-se estar livre de perigo.

-- 
Esta mensagem foi verificada pelo sistema de antiv�rus e
 acredita-se estar livre de perigo.



Re: [obm-l] Probabilidade

2018-11-07 Por tôpico Ralph Teixeira
Eu sei que é um roubo, mas... tem que ser mesmo exatamente 15 de cada tipo?
Não seria suficiente fazer alguma hipótese que diz que nenhum item foi
favorecido **no sorteio**?

Digo isso porque vejo um outro problema, também bastante prático, bem mais
fácil de resolver: para cada questão, sorteie aleatoriamente uma letra para
ser o gabarito, todas com a mesma probabilidade, cada questão independente
das demais. Assim, você está gerando um gabarito verdadeiramente aleatório;
na **média**, devem ser cerca de 15 de cada, mas um gabarito que tenha
exatamente 15 letras de cada tipo provavelmente não foi feito
aleatoriamente...

Agora, com este tipo de montagem de gabarito, qual a probabilidade de não
haver 2 letras consecutivas iguais? Pois bem, agora são 4x3x3x3x...x3
possibilidades favoráveis dentre 4^60 no total, e a probabilidade seria
(3/4)^(59) ~ 4.2522 x 10^(-8) ~ (1 em 23 milhões) < chance de ganhar na
mega-sena com 3 apostas simples. Ou seja, seria muito muito raro -- se o
seu gabarito tem essa propriedade, eu aposto que foi por design, não por
sorte.

Abraço, Ralph.

P.S.: Aliás, usando o que o Buffara já tinha feito, a probabilidade de um
gabarito aleatório (do meu jeito) ter *exatamente* 15 de cada letra seria
4.(3^59) / (60!/(15!^4)) ~ 1.986 x 10^(-5) ~ (1 em 50346). Como eu disse,
bem raro, a menos que seja por design.

PS2: Desconte a probabilidade muito pouco rara de eu ter errado alguma
conta. :D

On Wed, Nov 7, 2018 at 3:28 PM Paulo Rodrigues  wrote:

> Muito obrigado pelos avanços.
>
> Se der pra calcular o valor exato melhor, mas se desse pra estimar essa
> probabilidade, eu ficaria satisfeito. Depois explico o contexto prático do
> problema.
>
>
> Paulo Rodrigues
>
>
>
> Em qua, 7 de nov de 2018 às 13:49, Bruno Visnadi <
> brunovisnadida...@gmail.com> escreveu:
>
>> Uma maneira mais simples de colocar os As é imaginar que cada A é uma
>> peça que ocupa 2 espaços, e adicionar um 61º espaço para que seja possível
>> colocar um A na casa 60.
>> Então há 15 As e sobram 61-30 = 31 espaços, e há C(46, 15) maneiras de
>> colocar os As.
>>
>> Em qua, 7 de nov de 2018 às 12:13, Claudio Buffara <
>> claudio.buff...@gmail.com> escreveu:
>>
>>> Fiz mais um pequeno progresso.
>>>
>>> Resolvi um sub-problema.
>>> De quantas formas é possível colocar 15 As nas 60 posições de modo que 2
>>> As não ocupem posições adjacentes.
>>>
>>> Há 4 casos (exaustivos e mutuamente exclusivos) a considerar:
>>> 1) A primeira e a última posição são ocupadas por As:
>>> Nesse caso, uma vez colocados todos os As, sobrarão, entre eles, 14
>>> "espaços" com comprimentos variados.
>>> Chamando de x(k) o comprimento do k-ésimo espaço, teremos as condições:
>>> x(k) >= 1, para 1 <= k <= 14.
>>> e
>>> x(1) + x(2) + ... + x(14) = 45  (*)
>>> Logo, o número de maneiras de colocar os As neste caso é igual ao número
>>> de soluções inteiras positivas de (*): C(44,13)
>>>
>>> 2) Um A ocupa a primeira posição mas a última posição está vazia.
>>> A equação, neste caso, é:
>>> x(1) + x(2) + ... + x(15) = 45  com todos os x(k) >= 1 ==> C(44,14).
>>>
>>> 3) Um A ocupa a última posição mas a primeira está vazia:
>>> Por simetria, C(44,14)
>>>
>>> 4) A primeira e a última posições estão vazias:
>>> A equação é x(1) + ... + x(16) = 45   (x(k) >= 1) ==> C(44,15).
>>>
>>> Logo, o número de maneiras de colocar 15 As em 60 posições de modo que
>>> não fiquem dois As adjacentes é igual a:
>>> C(44,13) + 2*C(44,14) + C(44,15)
>>>
>>> Infelizmente, isso abre um monte de sub-casos chatos pra colocação dos
>>> Bs, de modo que não sei se é um caminho promissor. Provavelmente não.
>>>
>>> []s,
>>> Claudio.
>>>
>>>
>>> On Tue, Nov 6, 2018 at 4:01 PM Claudio Buffara <
>>> claudio.buff...@gmail.com> wrote:
>>>
 O número de casos possíveis é C(60,15)*C(45,15)*C(30,15)*C(15,15) =
 60!/(15!)^4
 (das 60 posições da sequencia, escolhe 15 para colocar os As; das 45
 restantes, escolhe mais 15 pra colocar os Bs; etc...)

 O número de casos favoráveis é mais chatinho.
 Eu sugiro olhar prum caso menor pra ver se aparece algum padrão.
 Por exemplo, 8 questões, com 2 respostas A, 2 B, 2 C e 2 D.
 Esse sai por inclusão-exclusão, mas com uma expressão meio feia e que
 não me parece o melhor caminho pro caso do problema.
 Talvez dê pra achar alguma recorrência ou função geradora.

 []s,
 Claudio.



 On Tue, Nov 6, 2018 at 1:04 PM Paulo Rodrigues 
 wrote:

> Pessoal, alguém pode dar uma mão na seguinte situação:
>
> Um gabarito é formado por uma sequência de 60 letras A, B, C e D sendo
> 15 de cada tipo.
> Qual a probabilidade de não existirem duas letras iguais vizinhas?
>
> Paulo Rodrigues
>
>
> --
> Esta mensagem foi verificada pelo sistema de antivírus e
> acredita-se estar livre de perigo.


>>> --
>>> Esta mensagem foi verificada pelo sistema de antivírus e
>>> acredita-se estar livre de perigo.
>>
>>
>> --
>> Esta mensagem foi 

Re: [obm-l] Probabilidade

2018-11-07 Por tôpico Ralph Teixeira
Não tenho a resposta, mas tenho uma boa intuição se for para um contexto
prático: esta probabilidade será super super baixa... :D :D :D

Uma maneira de estimar é fazer mesmo simulações: faça um programa para
sortear uma ordem, verifique se houve 2 letras iguais adjacentes, repita um
quinquilhão de vezes, veja em quantas deu ou não deu. O problema é que,
como ninha intuição me diz que a probabilidade é baixíssima, eu também
chuto que você vai ter que repetir MUITAS vezes para começa a aparecer
alguma estimativa razoável que não seja 0.

Abraço, Ralph.

On Wed, Nov 7, 2018 at 3:28 PM Paulo Rodrigues  wrote:

> Muito obrigado pelos avanços.
>
> Se der pra calcular o valor exato melhor, mas se desse pra estimar essa
> probabilidade, eu ficaria satisfeito. Depois explico o contexto prático do
> problema.
>
>
> Paulo Rodrigues
>
>
>
> Em qua, 7 de nov de 2018 às 13:49, Bruno Visnadi <
> brunovisnadida...@gmail.com> escreveu:
>
>> Uma maneira mais simples de colocar os As é imaginar que cada A é uma
>> peça que ocupa 2 espaços, e adicionar um 61º espaço para que seja possível
>> colocar um A na casa 60.
>> Então há 15 As e sobram 61-30 = 31 espaços, e há C(46, 15) maneiras de
>> colocar os As.
>>
>> Em qua, 7 de nov de 2018 às 12:13, Claudio Buffara <
>> claudio.buff...@gmail.com> escreveu:
>>
>>> Fiz mais um pequeno progresso.
>>>
>>> Resolvi um sub-problema.
>>> De quantas formas é possível colocar 15 As nas 60 posições de modo que 2
>>> As não ocupem posições adjacentes.
>>>
>>> Há 4 casos (exaustivos e mutuamente exclusivos) a considerar:
>>> 1) A primeira e a última posição são ocupadas por As:
>>> Nesse caso, uma vez colocados todos os As, sobrarão, entre eles, 14
>>> "espaços" com comprimentos variados.
>>> Chamando de x(k) o comprimento do k-ésimo espaço, teremos as condições:
>>> x(k) >= 1, para 1 <= k <= 14.
>>> e
>>> x(1) + x(2) + ... + x(14) = 45  (*)
>>> Logo, o número de maneiras de colocar os As neste caso é igual ao número
>>> de soluções inteiras positivas de (*): C(44,13)
>>>
>>> 2) Um A ocupa a primeira posição mas a última posição está vazia.
>>> A equação, neste caso, é:
>>> x(1) + x(2) + ... + x(15) = 45  com todos os x(k) >= 1 ==> C(44,14).
>>>
>>> 3) Um A ocupa a última posição mas a primeira está vazia:
>>> Por simetria, C(44,14)
>>>
>>> 4) A primeira e a última posições estão vazias:
>>> A equação é x(1) + ... + x(16) = 45   (x(k) >= 1) ==> C(44,15).
>>>
>>> Logo, o número de maneiras de colocar 15 As em 60 posições de modo que
>>> não fiquem dois As adjacentes é igual a:
>>> C(44,13) + 2*C(44,14) + C(44,15)
>>>
>>> Infelizmente, isso abre um monte de sub-casos chatos pra colocação dos
>>> Bs, de modo que não sei se é um caminho promissor. Provavelmente não.
>>>
>>> []s,
>>> Claudio.
>>>
>>>
>>> On Tue, Nov 6, 2018 at 4:01 PM Claudio Buffara <
>>> claudio.buff...@gmail.com> wrote:
>>>
 O número de casos possíveis é C(60,15)*C(45,15)*C(30,15)*C(15,15) =
 60!/(15!)^4
 (das 60 posições da sequencia, escolhe 15 para colocar os As; das 45
 restantes, escolhe mais 15 pra colocar os Bs; etc...)

 O número de casos favoráveis é mais chatinho.
 Eu sugiro olhar prum caso menor pra ver se aparece algum padrão.
 Por exemplo, 8 questões, com 2 respostas A, 2 B, 2 C e 2 D.
 Esse sai por inclusão-exclusão, mas com uma expressão meio feia e que
 não me parece o melhor caminho pro caso do problema.
 Talvez dê pra achar alguma recorrência ou função geradora.

 []s,
 Claudio.



 On Tue, Nov 6, 2018 at 1:04 PM Paulo Rodrigues 
 wrote:

> Pessoal, alguém pode dar uma mão na seguinte situação:
>
> Um gabarito é formado por uma sequência de 60 letras A, B, C e D sendo
> 15 de cada tipo.
> Qual a probabilidade de não existirem duas letras iguais vizinhas?
>
> Paulo Rodrigues
>
>
> --
> Esta mensagem foi verificada pelo sistema de antivírus e
> acredita-se estar livre de perigo.


>>> --
>>> Esta mensagem foi verificada pelo sistema de antivírus e
>>> acredita-se estar livre de perigo.
>>
>>
>> --
>> Esta mensagem foi verificada pelo sistema de antivírus e
>> acredita-se estar livre de perigo.
>
>
> --
> Esta mensagem foi verificada pelo sistema de antivírus e
> acredita-se estar livre de perigo.

-- 
Esta mensagem foi verificada pelo sistema de antiv�rus e
 acredita-se estar livre de perigo.



Re: [obm-l] Problema de Trigonometria

2018-11-06 Por tôpico Ralph Teixeira
Eles disseram que a expressão eh uma identidade **em x**. Abrindo a
expressão da direita e organizando, o que foi dado eh que:
sinx+2cosx=(Asiny)sinx+(Acosy)cosx vale para todo x real.

Como A e y sao NUMEROS (nao dependem de x), o unico jeito de isso acontecer
eh se os coeficientes de sinx e cosx de um lado coincidirem com os
coeficientes do outro lado, respectivamente (se você ainda não acredita
nisso, explico no "Mais Embaixo"). Então devemos ter:
Asiny=1
Acosy=2
Elevando ao quadrado e somando, vem A^2=5, ou seja, A=raiz(5) (pois eles
falaram que A>0).

Abraco, Ralph.

M.E.: a afirmacao que eu fiz equivale a dizer que sinx e cosx sao funcoes
linearmente independentes... em suma, estou dizendo que Bsinx+Ccosx=0 (para
todo x) se, e somente se, B=C=0. De fato, se B fosse nao nulo, teriamos
rearrumando que tanx=-C/B PARA TODO x REAL!! Isso eh claramente absurdo,
tanx nao eh uma funcao constante que nao depende de x! Entao a conclusao eh
que B=0, e portanto Ccosx=0 para todo x real, e dali C=0 tambem.

Para conectar o problema original com este papo do paragrafo anterior,
pegue aquela identidade em x em cima e escreva-a assim:
(Asiny-2) sinx + (Acosy-2) cosx = 0
Como os caras entre parenteses sao numeros, nao dependem de x, voce pode
chama-los de B e C... e entao recai no que eu falei sobre linearmente
independentes ali, ou seja, devemos ter B=C=0, e entao Asiny=1 e Acosy=2,
como falamos antes.

On Tue, Nov 6, 2018 at 9:59 PM Luiz Antonio Rodrigues 
wrote:

> Olá, boa noite!
> Transcrevi abaixo uma questão da Fuvest (SP). Já vi a resolução em vários
> sites, mas achei tudo muito estranho...
> Alguém pode me ajudar?
> Muito obrigado e um abraço!
> Luiz
>
> Sabe-se que existem números reais A e y, sendo A > 0, tais que:
>
> senx + 2cosx=Acos(x-y)
>
> para todo x real. Qual o valor de A?
>
> --
> Esta mensagem foi verificada pelo sistema de antivírus e
> acredita-se estar livre de perigo.

-- 
Esta mensagem foi verificada pelo sistema de antiv�rus e
 acredita-se estar livre de perigo.



[obm-l] Re: [obm-l] Questão do ITA

2018-10-10 Por tôpico Ralph Teixeira
Note que x=5 é um possível valor que resolve aquela equação (mas,
sinceramente, não interessa, eu faria o raciocínio abaixo com qualquer
número).

Então qualquer polinômio que satisfaça f(1)=5, f(-1)=10 e f(0)=20
automaticamente satisfaz todas as condições do enunciado (note que
a_0=f(0)). Em outras palavras, qualquer polinômio cujo gráfico passe pelos
pontos (-1,10),(0,20),(1,5) serve.

Agora escolha um ponto (z,0) qualquer como 4o ponto (onde z não é -1, 0 nem
1). Como quaisquer 4 pontos (com "x"s diferentes) determinam um único
polinômio de grau 3, haverá um polinômio de grau 3 que passa pelos pontos
dados e que tem raiz z. Como z pode ser negativo, positivo, raiz(2), ou 42,
nenhuma das respostas (A)-(D) pode valer (respectivamente!). Então tem que
ser (E).

Abraço, Ralph.

On Wed, Oct 10, 2018 at 5:41 AM Vanderlei Nemitz 
wrote:

> Bom dia, pessoal!
> Encontrei essa questão, que diz ser do ITA (eu particularmente não
> encontrei na internet).
> Como a resposta é E (nenhuma das anteriores), não sei se é possível provar
> que as anteriores são falsas. Eu não consegui concluir coisa alguma.
>
> *Seja f(x) = am.x^m + am–1.x^(m–1) + ... + a1.x + a0, onde am, am–1, ...,
> a1, a0 são reais, am diferente de 0 e a0 diferente de 0. Se f(1) é solução
> real da equação 2^(x–3) + 2^(x–4) = 2^(x–2) – 2^(x–1) + 14, f(–1) = 2.f(1)
> e a0 = 2.f(–1), então podemos afirmar:*
>
> *a) f(x) tem somente raízes reais positivas.*
>
> *b) f(x) tem somente raízes reais negativas.*
>
> *c) f(x) tem somente raízes reais inteiras.*
>
> *d) f(x) não tem raízes reais inteiras.*
>
> *e) nda*
> Alguém tem alguma ideia?
> Muito obrigado!
>
> --
> Esta mensagem foi verificada pelo sistema de antivírus e
> acredita-se estar livre de perigo.

-- 
Esta mensagem foi verificada pelo sistema de antiv�rus e
 acredita-se estar livre de perigo.



[obm-l] Re: [obm-l] Dúvida

2018-08-22 Por tôpico Ralph Teixeira
Acho que nao... Ah, se eu entendi corretamente, (3,6,9) e (3,5,12) seria um
contra-exemplo.

Abraco, Ralph.


On Wed, Aug 22, 2018 at 8:06 PM Pedro José  wrote:

> Boa noite.
>
> Sejam duas sequências em ordem crescente com ai,bi >0 e k elementos ambas.
> se:
> (a1+a2+a3+...+ak)/(b1+b2+b3+...+bk)=a1a2a3a3...ak/(b1b2b3...bk) podemos
> dizer que
> ai=bi para 0
> Grato,
> PJMS
>
> --
> Esta mensagem foi verificada pelo sistema de antivírus e
> acredita-se estar livre de perigo.

-- 
Esta mensagem foi verificada pelo sistema de antiv�rus e
 acredita-se estar livre de perigo.



Re: [obm-l] Matrizes

2018-08-21 Por tôpico Ralph Teixeira
Lema: Se A e B sao quadradas e AB=I, entao BA=I tambem.

Usando o Lema, fica facil:

(A+I)(B+I)=I, entao (B+I)(A+I)=I, entao BA=-A-B=AB.

Abraco, Ralph.

On Tue, Aug 21, 2018 at 11:09 PM Vanderlei Nemitz 
wrote:

> Boa noite, pessoal!
> Resolvi a seguinte questão, mas de uma forma um tanto complicada.
> Gostaria de uma solução mais simples.
> Muito obrigado!
> Vanderlei
>
> *Sejam A e B matrizes reais n x n tais que AB + A + B = 0. Prove que AB =
> BA.*
>
> --
> Esta mensagem foi verificada pelo sistema de antivírus e
> acredita-se estar livre de perigo.

-- 
Esta mensagem foi verificada pelo sistema de antiv�rus e
 acredita-se estar livre de perigo.



Re: [obm-l] geometria plana

2018-07-29 Por tôpico Ralph Teixeira
Sim! Quando aplica-se qualquer transformacao linear a um objeto, a razao
entre o volume da figura nova e o da figura antiga eh constante e igual ao
determinante da transformacao! Entao ambas as areas ficariam multiplicadas
pelo mesmo numero, e a razao se manteria!

Outro detalhe: teria que ver se a transformacao linear leva medianas em
medianas... Mas isso tambem eh verdade e vem direto da definicao de
"transf. linear", pontos medios se preservam.

Abraco, Ralph.

On Sun, Jul 29, 2018 at 5:53 PM Claudio Buffara 
wrote:

> Idéia que me ocorreu: todo triângulo é afim-equivalente a um triângulo
> equilátero.
> Mediante translações, as medianas de um triângulo equilátero de lado 1
> formam um triângulo equilátero cujos lados medem raiz(3)/2 e, portanto,
> cuja área é 3/4.
> Será que uma transformação afim preserva a razão entre as áreas do
> triângulo original e do triângulo “medianico”?
>
> Abs,
> Cláudio.
>
> Enviado do meu iPhone
>
> Em 28 de jul de 2018, à(s) 19:08, matematica10complicada <
> profdouglaso.del...@gmail.com> escreveu:
>
> Então,podemos fazer o seguinte:
>
> Considere um triângulo ABC, cujas medianas são AM, BN, CP, e baricentro
> G desta forma
>
> 1)Monte um paralelogramo BNQM de forma que MQ intercepte AC em R.
>
> 2)Como o baricentro divide em seis áreas iguais, temos que a área do
> triângulo AGN será 1/6.
>
> 3)É fácil ver que MQ=BN, e AQ=CP.
>
> 4) Desta forma a área procurada será a do triângulo AMQ que é o dobro
> da área do triângulo AMR=3/8.
>
> Portanto a resposta é 3/4.
>
>
> Douglas Oliveira.
> Grande Abraço.
>
> Em 28 de julho de 2018 16:32, marcone augusto araújo borges <
> marconeborge...@hotmail.com> escreveu:
>
>> Seja um triangulo ABC cuja area eh igual a 1. Determinar a area do
>> triangulo cujos  lados sao iguais às medianas do triangulo ABC
>>
>> --
>> Esta mensagem foi verificada pelo sistema de antivírus e
>> acredita-se estar livre de perigo.
>>
>
>
> --
> Esta mensagem foi verificada pelo sistema de antivírus e
> acredita-se estar livre de perigo.
>
>
> --
> Esta mensagem foi verificada pelo sistema de antivírus e
> acredita-se estar livre de perigo.
>

-- 
Esta mensagem foi verificada pelo sistema de antiv�rus e
 acredita-se estar livre de perigo.



Re: [obm-l] geometria plana

2018-07-28 Por tôpico Ralph Teixeira
Vamos fazer este por vetores... Mas primeiro um tiquinho de notacao: dados
dois vetores v e w no plano, vou escrever [v,w] para o determinante da
matriz cujas colunas sao v e w; em outras palavras, [v,w] eh a area do
paralelogramo cujos lados sao v e w, com sinal determinado pela orientacao.

Uma propriedade importante do determinante eh que ele eh linear em cada uma
das suas coordenadas (ou seja, ele eh "bilinear"). Em outras palavras, voce
pode usar a propriedade distributiva como se ele fosse um produto:
[ka+b,c]=k[a,c]+[b,c] e [a,kb+c]=k[a,b]+[a,c] para quaisquer vetores a,b,c,
e qualquer escalar k. Outra propriedade essencial eh que [v,w]=-[w,v] para
quaisquer v e w; em particular, [x,x]=0 para todo vetor x.

Com isso em mente, o problema eh facil. Escreva AB=2v e AC=2w (vetores).
Supondo spdg uma das orientacoes possiveis, o problema diz que [2v,2w]=2,
isto eh, [v,w]=1/2.

Agora, as medianas sao (faca uma figura) v-2w, w-2v e v+w (dos vertices
para os pontos medios). Note que a soma desses 3 vetores eh zero, entao o
triangulo cujos lados sao as medianas eh exatamente o triangulo cujos lados
sao esses 3 vetores!

Portanto, a area pedida eh a metade de  [w-2v,v-2w]... Usando a propriedade
distributiva, esse produto fica
[w,v]-2[w,w]-2[v,v]+4[v,w]=-1/2-0-0+4/2=3/2. Ou seja, a resposta eh 3/4.

Abraco, Ralph.

On Sat, Jul 28, 2018 at 4:43 PM marcone augusto araújo borges <
marconeborge...@hotmail.com> wrote:

> Seja um triangulo ABC cuja area eh igual a 1. Determinar a area do
> triangulo cujos  lados sao iguais às medianas do triangulo ABC
>
> --
> Esta mensagem foi verificada pelo sistema de antivírus e
> acredita-se estar livre de perigo.
>

-- 
Esta mensagem foi verificada pelo sistema de antiv�rus e
 acredita-se estar livre de perigo.



[obm-l] Re: [obm-l] Re: Números complexos

2018-07-16 Por tôpico Ralph Teixeira
Oops, foi a fatoração! Devia ser (x+1)(x^2-x+1)=0, sim?

On Mon, Jul 16, 2018 at 2:00 PM Alexandre Antunes <
prof.alexandreantu...@gmail.com> wrote:

>
> Boa tarde,
>
> Se fizermos x^3+1^3=0
>
> Podemos fatorar: (x-1)(x^2+×+1)=0
>
> Certo?
>
> Estou achando um resultado
> -1
> -1/2 +raiz (3)i/2
> -1/2 -raiz (3)i/2
>
> E o resultado (resposta prevista) está diferente ... Será que "dei
> bobeira"?
>
> Antecipadamente agradeço.
>
> Em Seg, 16 de jul de 2018 12:56, Alexandre Antunes <
> prof.alexandreantu...@gmail.com> escreveu:
>
>>
>> Bom dia,
>>
>> Quais as raízes cúbicas de -1?
>>
>
> --
> Esta mensagem foi verificada pelo sistema de antivírus e
> acredita-se estar livre de perigo.

-- 
Esta mensagem foi verificada pelo sistema de antiv�rus e
 acredita-se estar livre de perigo.



Re: [obm-l] Propriedade dos Reais

2018-07-14 Por tôpico Ralph Teixeira
Vou colocar dois argumentos:

1) MAIS LOGICO:
(x<=x)
significa
(x wrote:

> Olá, pessoal!
> Boa tarde!
> Porque dizemos que x<=x para todo x real?
> É algo que eu não consigo entender...
> Muito obrigado e um abraço!
> Luiz
>
> --
> Esta mensagem foi verificada pelo sistema de antivírus e
> acredita-se estar livre de perigo.

-- 
Esta mensagem foi verificada pelo sistema de antiv�rus e
 acredita-se estar livre de perigo.



[obm-l] Re: [obm-l] Equação 4 grau

2018-06-26 Por tôpico Ralph Teixeira
Tem algo estranho ali, confere o enunciado?

Tomando P(x)=x^4-4x-1, note que P(-1)=4 e P(0)=-1, entao tem uma raiz entre
-1 e 0... o que nao encaixa com nenhuma das alternativas??

Mais: P(1)=-4 e P(2)=7, entao tem outra raiz entre 1 e 2... Huh?

Abraco, Ralph.

On Tue, Jun 26, 2018 at 3:22 PM Daniel Quevedo  wrote:

> As raizes reais da equação x^4 -4x=1 pertencem ao intervalo:
> A) (1,11)
> B) (2, 12)
> C) (3, 13)
> D) (4, 14)
> E) ( 5, 15)
>
> R: c
> --
> Fiscal: Daniel Quevedo
>
> --
> Esta mensagem foi verificada pelo sistema de antivírus e
> acredita-se estar livre de perigo.

-- 
Esta mensagem foi verificada pelo sistema de antiv�rus e
 acredita-se estar livre de perigo.



[obm-l] Re: [obm-l] Equação Funcional

2018-06-11 Por tôpico Ralph Teixeira
Puxa, se fosse g(x)=(x-1)/x ali dentro do segundo termo, eu sabia fazer
rápido... :( Era só escrever y=g(x), z=g(y), e então:
f(x)+f(y)=1+x
f(y)+f(z)=1+y
f(z)+f(x)=1+z
pois é fácil ver que g(z)=g(g(g(x)))=x. Resolvendo esse sisteminha,
acharíamos f(x).

Porém, com esse enunciado... Hm, alguém confere aqui o raciocínio abaixo,
porque acho que eu consigo mostrar que **não dá** para resolver isso, mas
estou morrendo de sono, então provavelmente escrevi alguma bobagem imensa.

Observe que g(x)=(1-x)/x é injetiva (e sua inversa é g^(-1)(y)=1/(1+y)).
Dado um x_0=a, crie a sequência {x_k} com k inteiro onde x_(k+1)=g(x_k) --
observe que crio isto incluindo k negativo, o que é possível desde que
nenhum dos números da órbita seja 0 ou -1. Vou chamar o **conjunto** de
valores {x_k} de "órbita" do número a.

Pois bem, a equação funcional só dá informações sobre os valores de f
dentro de cada órbita! Ela diz que f(x_k)+f(x_(k+1))=1+x_k (*), e mais
nada, ou seja, ela não relaciona os valores de f em órbitas distintas! Se a
órbita é infinita, isto é, se os x_k são todos distintos, você pode
ESCOLHER f(a) como quiser e calcular os outros f(x_k) usando (*) como
recorrência.

Agora você me pergunta: porque a órbita não fecha? Bom, você tem razão,
para vários valores de "a" a órbita fecha, isto é, poderia ser x_P=x_0=a
para algum P<>0... Mas a equação x_P=a quer dizer g(g(g(...g(a))...)=a, que
é uma equação quadrática (né?), e portanto tem no máximo 2 raízes reais.
Então, mesmo que consideremos todos os P possíveis, o conjunto dos a que
fazem a órbita fechar é enumerável... Bom, os reais não são enumeráveis,
então há várias órbitas infinitas Acho.

Abraço, Ralph.

P.S.: Se eu tivesse bom senso, conferia isso antes de mandar para a
lista... Ah, dane-se, mesmo que eu esteja errado este tipo de raciocínio é
interessante, não?
P.S.2: Se o enunciado falar que f é *contínua*, aí talvez dê para fazer
algo usando o limite de x_k...

On Mon, Jun 11, 2018 at 8:32 AM Jeferson Almir 
wrote:

> Seja f(x) uma função real definida em R -{0,1} tal que
>
> f(x) + f( 1-x | x ) =1 + x determine f (x) .
>
> Obs:  ( 1-x | x) é 1-x dividido por x .
>
> --
> Esta mensagem foi verificada pelo sistema de antivírus e
> acredita-se estar livre de perigo.

-- 
Esta mensagem foi verificada pelo sistema de antiv�rus e
 acredita-se estar livre de perigo.



[obm-l] Re: [obm-l] Re: [obm-l] Alguém pode ajudar?

2018-05-15 Por tôpico Ralph Teixeira
Hmmm... Acho que eles permitem que usemos m e n negativos...

Por exemplo, podia ser m=2000 e n=-1. Então a fração seria 0/(2000^2-1),
que pode ser simplificada para 0/1 dividindo por
d=2000^2-1=400-1=399...
...cuja soma dos algarismos eh 57, como eles parecem querer.

Para provar que esse d eh maximo, note que d|m+2000n e d|n+2000m, portanto
d|2001(m+n).

Mas se d tivesse fator comum com m+n, ele apareceria tambem em
(2000m+n)-(m+n)=1999m. Mas d nao poderia ter fator comum com m (pois, tendo
com m+n, teria com n tambem, e (m,n)=1), entao o unico fator possivel de d
em m+n eh aquele 1999.

Mas, como d|2001(m+n) e o unico fator possivel de d em m+n eh 1999, entao
d|1999.2001=(2000)^2-1... E portanto d<=2000^2-1.

Abraco, Ralph.

2018-05-15 23:18 GMT-03:00 Anderson Torres :

> > 2) os inteiros m e n são primos entre si. Sabendo que a fração (m +
> > 2000n)/(n +2000m) pode ser simplificado cancelando o divisor comum d. A
> soma
> > dos algarismos do maior valor que d pode assumir é igual a:
> > R: 57
>
> d|m+2000n
> d|n+2000m
> d|1999(m-n)
>
> 1999 é primo
>
> Se d=1999, 1999|m+2000n, 1999|m+n, basta escolher m=1, n=1998, que
> funciona.
>
> A soma dos dígitos de 1999 na base 10 é 28
>
> P.S.: para deixar a lista mais organizada, envie apenas um problema
> por e-mail. A não ser que você queira enviar a prova toda.
>
> Em outras palavras: se quiser enviar problemas soltos, envie um por vez.
>
> --
> Esta mensagem foi verificada pelo sistema de antivírus e
>  acredita-se estar livre de perigo.
>
>
> =
> Instru�ões para entrar na lista, sair da lista e usar a lista em
> http://www.mat.puc-rio.br/~obmlistas/obm-l.html
> =
>

-- 
Esta mensagem foi verificada pelo sistema de antiv�rus e
 acredita-se estar livre de perigo.



[obm-l] Re: [obm-l] Re: [obm-l] Re: [obm-l] Re: [obm-l] Re: [obm-l] Fwd: não sei como fazer, tentei desigualdades de médias e não saiu

2018-05-14 Por tôpico Ralph Teixeira
Oi, Daniel.

Por que há duas opções: ou pq=k+1 e r=k-1, ou pq=k-1 e r=k+1, e subtraindo
dá pq-r=+-2. Isso vem de pqr=(k+1)(k-1) e do fato de p,q,r serem primos,
então não tem como você "separar" os fatores primos de p entre k-1 e k+1
(idem para q e r).

Bom, para ser exato, eu esqueci de considerar outras opções: poderia ser
pqr=k+1 e 1=k-1 (absurdo!) ou pqr=k-1 e 1=k+1 (mais absurdo!). Assim, dois
dos primos p,q,r têm que corresponder a um dos fatores {k+1,k-1}, e o
terceiro primo tem que ser sozinho o outro fator. Reordenando se
necessário, eu posso supor que pq é um dos fatores e r é o outro, daí minha
afirmação {pq,r}={k+1,k-1} (como igualdade de conjuntos).

Abraço, Ralph.

2018-05-14 8:55 GMT-03:00 Daniel Quevedo <daniel...@gmail.com>:

> Perfeito é essa a resposta. Só não entendi o passo pq-r =2 ou -2 . Não
> deveria ser apenas um número da forma 2Q ou -2Q, ou seja par? Pq vc afirma
> q é +-2?
>
> Em dom, 13 de mai de 2018 às 23:20, Ralph Teixeira <ralp...@gmail.com>
> escreveu:
>
>> Ah, assim fica bem melhor.
>>
>> Temos pqr=(k+1)(k-1). Como p, q e r sao primos, entao (trocando a ordem
>> de p,q,r se necessario) {pq,r}={k+1,k-1}. Ou seja, pq-r=2 ou -2.
>>
>> Entao p+q+(pq+-2)=2001, ou seja ((p+1)/2)((q+1)/2)=501 ou 500
>>
>> As unicas fatoracoes de 501 em dois fatores sao 1.501 e 3.167, que
>> rapidamente verificam-se inuteis.
>> As unicas fatoracoes de 500 em dois fatores sao 1.500, 2.250, 4.125,
>> 5.100, 10.50, 20.25. Verificando uma a uma para ver quais dao p, q primos,
>> encontramos apenas
>> (p,q)=(3,499), portanto r=2001-p-q=1499 (ok, tambem primo) e k=r-1=1498.
>> Entao 22 eh a resposta?
>>
>> Abraco, Ralph.
>>
>> 2018-05-13 22:30 GMT-03:00 Daniel Quevedo <daniel...@gmail.com>:
>>
>>>
>>> Em dom, 13 de mai de 2018 às 20:12, Pacini Bores <pacini.bo...@globo.com>
>>> escreveu:
>>>
>>>> Desculpe-me esqueci d colocar um dado na questão na hora d escrever. Os
>>> números p, q é r são primos ímpares. Havia colocado apenas ímpares.
>>>
>>>> Oi Daniel,
>>>>
>>>> Estranho, pois p=999, q= 1001 e r =1; teremos  p+q+r=2001 , pqr+1=
>>>> 100= (1000)^2.
>>>>
>>>> Ou seja, k=1000 ?
>>>>
>>>> Pacini
>>>>
>>>> Em 13/05/2018 2:56, Daniel Quevedo escreveu:
>>>>
>>>>
>>>>
>>>> - Mensagem encaminhada -
>>>> De: Daniel Quevedo <daniel...@gmail.com>
>>>> Data: dom, 13 de mai de 2018 às 02:54
>>>> Assunto:
>>>> Para: ob...@mat-puc.rio.br <ob...@mat-puc.rio.br>
>>>>
>>>>
>>>> Sabendo que p, q e r são números impares distintos com p+q+r= 2001 e
>>>> que k é um inteiro positivo tal que pqr +1 =k^2, a soma dos algarismos do
>>>> único valor possível para k é igual a:
>>>> A) 20
>>>> B) 21
>>>> C) 22
>>>> D) 23
>>>> E) 24
>>>> --
>>>> Fiscal: Daniel Quevedo
>>>> --
>>>> Fiscal: Daniel Quevedo
>>>>
>>>> --
>>>> Esta mensagem foi verificada pelo sistema de antivrus e
>>>> acredita-se estar livre de perigo.
>>>>
>>>>
>>>>
>>>> --
>>>> Esta mensagem foi verificada pelo sistema de antivírus e
>>>> acredita-se estar livre de perigo.
>>>>
>>> --
>>> Fiscal: Daniel Quevedo
>>>
>>> --
>>> Esta mensagem foi verificada pelo sistema de antivírus e
>>> acredita-se estar livre de perigo.
>>>
>>
>>
>> --
>> Esta mensagem foi verificada pelo sistema de antivírus e
>> acredita-se estar livre de perigo.
>
> --
> Fiscal: Daniel Quevedo
>
> --
> Esta mensagem foi verificada pelo sistema de antivírus e
> acredita-se estar livre de perigo.

-- 
Esta mensagem foi verificada pelo sistema de antiv�rus e
 acredita-se estar livre de perigo.



[obm-l] Re: [obm-l] Re: [obm-l] Re: [obm-l] Fwd: não sei como fazer, tentei desigualdades de médias e não saiu

2018-05-13 Por tôpico Ralph Teixeira
Ah, assim fica bem melhor.

Temos pqr=(k+1)(k-1). Como p, q e r sao primos, entao (trocando a ordem de
p,q,r se necessario) {pq,r}={k+1,k-1}. Ou seja, pq-r=2 ou -2.

Entao p+q+(pq+-2)=2001, ou seja ((p+1)/2)((q+1)/2)=501 ou 500

As unicas fatoracoes de 501 em dois fatores sao 1.501 e 3.167, que
rapidamente verificam-se inuteis.
As unicas fatoracoes de 500 em dois fatores sao 1.500, 2.250, 4.125, 5.100,
10.50, 20.25. Verificando uma a uma para ver quais dao p, q primos,
encontramos apenas
(p,q)=(3,499), portanto r=2001-p-q=1499 (ok, tambem primo) e k=r-1=1498.
Entao 22 eh a resposta?

Abraco, Ralph.

2018-05-13 22:30 GMT-03:00 Daniel Quevedo :

>
> Em dom, 13 de mai de 2018 às 20:12, Pacini Bores 
> escreveu:
>
>> Desculpe-me esqueci d colocar um dado na questão na hora d escrever. Os
> números p, q é r são primos ímpares. Havia colocado apenas ímpares.
>
>> Oi Daniel,
>>
>> Estranho, pois p=999, q= 1001 e r =1; teremos  p+q+r=2001 , pqr+1=
>> 100= (1000)^2.
>>
>> Ou seja, k=1000 ?
>>
>> Pacini
>>
>> Em 13/05/2018 2:56, Daniel Quevedo escreveu:
>>
>>
>>
>> - Mensagem encaminhada -
>> De: Daniel Quevedo 
>> Data: dom, 13 de mai de 2018 às 02:54
>> Assunto:
>> Para: ob...@mat-puc.rio.br 
>>
>>
>> Sabendo que p, q e r são números impares distintos com p+q+r= 2001 e que
>> k é um inteiro positivo tal que pqr +1 =k^2, a soma dos algarismos do único
>> valor possível para k é igual a:
>> A) 20
>> B) 21
>> C) 22
>> D) 23
>> E) 24
>> --
>> Fiscal: Daniel Quevedo
>> --
>> Fiscal: Daniel Quevedo
>>
>> --
>> Esta mensagem foi verificada pelo sistema de antivrus e
>> acredita-se estar livre de perigo.
>>
>>
>>
>> --
>> Esta mensagem foi verificada pelo sistema de antivírus e
>> acredita-se estar livre de perigo.
>>
> --
> Fiscal: Daniel Quevedo
>
> --
> Esta mensagem foi verificada pelo sistema de antivírus e
> acredita-se estar livre de perigo.
>

-- 
Esta mensagem foi verificada pelo sistema de antiv�rus e
 acredita-se estar livre de perigo.



[obm-l] Re: [obm-l] Re: [obm-l] Re: [obm-l] Função Composta

2018-05-12 Por tôpico Ralph Teixeira
Oops, eh verdade, esqueci de mostrar que f nao tem ponto fixo em Z_2005
(obviamente f nao tem ponto fixo, pois f(f(a))<>a).

Suponha por absurdo que f(a)=a+K.2005 para algum a em {0,1,...2004}, com K
natural. Entao f(a+K.2005)=f(f(a))=a+2005. Agora, usando nossa
propriedadezinha:

f(a+K.2005)-f(a)=K.2005
a+2005 - (a+K.2005) = K.2005
K = 1/2 (absurdo).

Abraco, Ralph.



2018-05-12 2:49 GMT-03:00 Bernardo Freitas Paulo da Costa <
bernardo...@gmail.com>:

> Oi Ralph,
>
> 2018-05-11 20:03 GMT-03:00 Ralph Teixeira <ralp...@gmail.com>:
> > (Vou supor que 0 eh natural; se nao for, apenas troque 0 por 2005 ali
> > embaixo e ajeite as coisas)
> >
> > Primeiro: f eh injetiva. De fato, f(a)=f(b) => f(f(a))=f(f(b)) =>
> > a+2005=b+2005 => a=b.
> >
> > Segundo: para todo n natural, f(n+2005)=f(f(f(n)))=f(n)+2005. Portanto,
> por
> > indução, para qualquer K natural, tem-se
> > f(n+K.2005)=f(n)+K.2005, ou seja, f(n+K.2005)-f(n)=K.2005.
> >
> > VERSÃO CURTA COM TERMINOLOGIA "MOD":
> > Ou seja, mostramos que   a=b (mod 2005) => f(a)=f(b) (mod 2005).
> > Agora, se f(m)=n (mod 2005), entao f(n)=f(f(m))=m+2005=m (mod 2005). Ou
> > seja, f estah bem definida e eh sua propria inversa em Z_2005, o que eh
> > absurdo, pois Z_2005 tem um numero impar de elementos.
>
> Peraí, não entendi direito... se f(n) == n (mod 2005), temos uma
> função que é sua própria inversa mod 2005.  Temos que excluir este
> caso...
>
> > 2018-05-11 10:42 GMT-03:00 Jeferson Almir <jefersonram...@gmail.com>:
> >>
> >> Como provar que nos naturais não existe a função f ( f(n) ) = n + 2005
> ???
> >>
>
> Abraços,
> --
> Bernardo Freitas Paulo da Costa
>
> --
> Esta mensagem foi verificada pelo sistema de antivírus e
>  acredita-se estar livre de perigo.
>
>
> =
> Instru�ões para entrar na lista, sair da lista e usar a lista em
> http://www.mat.puc-rio.br/~obmlistas/obm-l.html
> =
>

-- 
Esta mensagem foi verificada pelo sistema de antiv�rus e
 acredita-se estar livre de perigo.



[obm-l] Re: [obm-l] Função Composta

2018-05-11 Por tôpico Ralph Teixeira
(Vou supor que 0 eh natural; se nao for, apenas troque 0 por 2005 ali
embaixo e ajeite as coisas)

Primeiro: f eh injetiva. De fato, f(a)=f(b) => f(f(a))=f(f(b)) =>
a+2005=b+2005 => a=b.

Segundo: para todo n natural, f(n+2005)=f(f(f(n)))=f(n)+2005. Portanto, por
indução, para qualquer K natural, tem-se
f(n+K.2005)=f(n)+K.2005, ou seja, f(n+K.2005)-f(n)=K.2005.

VERSÃO CURTA COM TERMINOLOGIA "MOD":
Ou seja, mostramos que   a=b (mod 2005) => f(a)=f(b) (mod 2005).
Agora, se f(m)=n (mod 2005), entao f(n)=f(f(m))=m+2005=m (mod 2005). Ou
seja, f estah bem definida e eh sua propria inversa em Z_2005, o que eh
absurdo, pois Z_2005 tem um numero impar de elementos.

VERSÃO LONGA, QUE EH O MESMO RACIOCÍNIO ESCRITO DE OUTRO JEITO:
Em outras palavras, mostramos que se a e b deixam o mesmo resto na divisão
por 2005, f(a) e f(b) também o fazem.

Agora olhe para o conjunto {f(0),f(1),f(2),f(3),...,f(2004)} e pense que
restos estes números deixam na divisão por 2005.
-- Não ha dois restos iguais! Se fosse, digamos, f(25)-f(19)=K.2005,
teríamos f(25)-f(19)=f(19+K(2005))-f(19), e, pela injetividade,
19+K(2005)=25, absurdo.
-- Mas então todos os restos de 0 a 2004 estão presentes ali naquele
conjunto...
-- ...porem, se f(a)=K.2005+b onde b eh o resto de f(a) na divisão por
2005, então f(b)=f(b+K.2005)-K.2005=f(f(a))-K.2005=a+(1-K).2005. Ou seja,
se f(a) deixa resto b, então f(b) deixa resto a.

Assim, f determinaria um PAREAMENTO dos números 0, 1, 2, 3, .., 2004 via
estes restos de divisao: f(a)=b (mod 2005) implica f(b)=a (mod 2005), e
vice-versa.

Porem, não pode existir este pareamento (são 2005 restos, numero impar!),
absurdo. Portanto, f não existe.

Abraco, Ralph.

2018-05-11 10:42 GMT-03:00 Jeferson Almir :

> Como provar que nos naturais não existe a função f ( f(n) ) = n + 2005 ???
>
> --
> Esta mensagem foi verificada pelo sistema de antivírus e
> acredita-se estar livre de perigo.

-- 
Esta mensagem foi verificada pelo sistema de antiv�rus e
 acredita-se estar livre de perigo.



[obm-l] Re: [obm-l] Re: [obm-l] Desigualdade com potências

2018-04-29 Por tôpico Ralph Teixeira
Que tal assim:

POR BAIXO (BEM folgado): Como 3^3=27<32=2^5, temos
3^100<(3^3)^34<(2^5)^34=2^170. Portanto
3^100+2^100<2^170+2^100<2^170+2^170=2^171<2^200=4^100.
POR CIMA (mais apertado!): Como 3^7=2187>2^11=2048, temos
3^100=9.(3^98)>9.(2^154)>(2^3).(2^154)=2^157. Somando 2^100, ficamos abaixo
de 2.(2^157)=2^158=4^79.

Abraco, Ralph.

2018-04-29 13:09 GMT-03:00 Anderson Torres :

> 2018-04-29 8:45 GMT-03:00 Douglas Oliveira de Lima
> :
> > Prove que 4^79<2^100+3^100<4^100, usando matemática elementar.
> >
>
> O desejo de trapacear isso com log é muito forte :)
>
> Isso equivale a mostrar que
>
> 2^158-2^100<3^100<2^200-2^100
>
> Ou
>
> (2^58-1)*2^100<3^100<(2^100-1)*2^100
>
> Ou talvez
>
> 2^58 < (3/2)^100+1 < 2^100
>
> Daqui, tenho poucas ideias para a desigualdade mais à esquerda...
>
> > Douglas Oliveira.
> >
> > --
> > Esta mensagem foi verificada pelo sistema de antivírus e
> > acredita-se estar livre de perigo.
>
> --
> Esta mensagem foi verificada pelo sistema de antivírus e
>  acredita-se estar livre de perigo.
>
>
> =
> Instruções para entrar na lista, sair da lista e usar a lista em
> http://www.mat.puc-rio.br/~obmlistas/obm-l.html
> =
>

-- 
Esta mensagem foi verificada pelo sistema de antiv�rus e
 acredita-se estar livre de perigo.



[obm-l] Re: [obm-l] Re: [obm-l] Re: [obm-l] Re: [obm-l] Teoria dos números

2018-03-19 Por tôpico Ralph Teixeira
Opa, opa, opa! Pedro, voce achou uma formula assim generica, z=-(x+y)/2,
que resolve esta equacao? Beleza, excelente ideia, temos um caminho!

Porque, se z=-(x+y)/2 eh SEMPRE solucao disso (independente de "inteiros"
ou nao), quer dizer que essa coisa horrorosa, passando tudo para o outro
lado, tem z+(x+y)/2 como fator, ou seja, x+y+2z como fator. Analogamente,
vai ter 2x+y+z e x+2y+z tambem!

Em suma, a gente pode voltar na primeira equacao com a sua ideia, jogar
tudo para a esquerda (exceto pelo 1 chato que nao aparece na sua
expressao), e fatorar. Vejamos... Acho que fica assim:

1/2*(2x+y+z)(x+2y+z)(x+y+2z)=1
(2x+y+z)(x+2y+z)(x+y+2z)=2

Confiram se eu nao errei contas... Mas agora ficou **bem** facil! :D

Abraco, Ralph.

2018-03-19 14:33 GMT-03:00 Pedro José :

> Bom dia!
>
> Estou só conjecturando. Pois, não consegui nenhuma restrição.
> A única coisa que consegui, mas não me adiantou de nada, é que:
> x,y pares ou x,y ímpares e z = -(x+y)/2 é solução de
>
> *(x + y)(y + z)(z + x)/2 + (x + y + z)3 =  – xyz*
> Também, não consegui provar que é a única família de solução da equação
> acima para inteiros.
>
> Em 19 de março de 2018 14:14, Claudio Buffara 
> escreveu:
>
>> Podem existir soluções não triviais envolvendo inteiros negativos.
>>
>> 2018-03-19 10:17 GMT-03:00 Pedro José :
>>
>>> Bom dia!
>>>
>>> Poderia postar a solução? Não consegui achar nenhuma restrição para
>>> trabalhar num subconjunto  pequeno dos inteiros.
>>> Creio que vá ser apenas a trivial (0,0,1) e suas permutações.
>>>
>>> grato,
>>> PJMS
>>>
>>> Em 13 de março de 2018 20:19, Douglas Oliveira de Lima <
>>> profdouglaso.del...@gmail.com> escreveu:
>>>
 Essa achei legal e estou postando.

 *Resolva nos inteiros a seguinte equação:  (x + y)(y + z)(z + x)/2 + (x
 + y + z)3 = 1 – xyz* .

 Abraço do
 Douglas Oliveira

 --
 Esta mensagem foi verificada pelo sistema de antivírus e
 acredita-se estar livre de perigo.
>>>
>>>
>>>
>>> --
>>> Esta mensagem foi verificada pelo sistema de antivírus e
>>> acredita-se estar livre de perigo.
>>>
>>
>>
>> --
>> Esta mensagem foi verificada pelo sistema de antivírus e
>> acredita-se estar livre de perigo.
>>
>
>
> --
> Esta mensagem foi verificada pelo sistema de antivírus e
> acredita-se estar livre de perigo.
>

-- 
Esta mensagem foi verificada pelo sistema de antiv�rus e
 acredita-se estar livre de perigo.



[obm-l] Re: [obm-l] Álgebra

2018-03-13 Por tôpico Ralph Teixeira
Sim! Dá 80abc(a²+b²+c²)!

...

...

Ah, você quer o JEITO... Huh... é bom, er... taquei no Scientific
Workplace e mandei ele simplificar tudo desculpa. Talvez esteja até
correto. :P

Mas com a resposta em mãos alguém vai arrumar uma maneira bonita e criativa
de chegar na mesmaresposta no braço, né? Né? Né?

...

:D

Abraços preguiçosos, Ralph.

P.S.: Deve ter um jeito óbvio de ver que só os termos do tipo 3,1,1 ficam.
Ah, sim: a expressão é ímpar em cada uma das variáveis, então todos os
expoentes de cada variável têm que ser ímpares na resposta. Mas o polinômio
é homogêneo, ou seja, a soma dos expoentes de cada termo é 5, então todos
os termos são da forma a^m.b^n.c^p onde m+n+p=5 são ímpares. Acho que só
3+1+1 satisfaz ambas as condições? Como a expressão é invariante por
permutação de variáveis, então só haverá um coeficiente, multiplicando os
três monômios a^3bc, ab^3c, e abc^3, ou seja, já sei que tem que dar algo
do tipo Kabc(a^2+b^2+c^2). Para achar K, taque a=b=c=1, e calibre K. Hm,
acho que resolveu!

2018-03-13 18:51 GMT-03:00 Douglas Oliveira de Lima <
profdouglaso.del...@gmail.com>:

> Olá meus amigos, vocês conhecem um jeito bom de simplificar isso
> (a+b+c)^5-(a-b+c)^5-(a+b-c)^5-(b+c-a)^5
>
> Abraços
> Douglas Oliveira
>
> --
> Esta mensagem foi verificada pelo sistema de antivírus e
> acredita-se estar livre de perigo.

-- 
Esta mensagem foi verificada pelo sistema de antiv�rus e
 acredita-se estar livre de perigo.



[obm-l] Re: [obm-l] Re: [obm-l] Problema de minimização

2018-03-11 Por tôpico Ralph Teixeira
...e portanto a elipse de focos A e B passando por O tem que ser tangente
aa elipse de focos C e D passando por O Fica como exercicio pensar o
que uma coisa tem a ver com a outra.

(O que podia ser visto de outras formas, diga-se de passagem, se voce sabe
que a normal a tal elipse eh a bissetriz de AOB).

Abraco, Ralph.



2018-03-11 20:29 GMT-03:00 Claudio Buffara :

> É isso aí!
> Uma aplicação simples mas elegante da desigualdade triangular.
> E o ponto O não parece ser tão difícil de conjecturar. Afinal, o ponto de
> intersecção das diagonais talvez seja o “ponto notável”
> mais óbvio de um quadrilátero (certamente é o mais fácil de construir -
> duas aplicações da régua e nenhuma do compasso).
>
> E quando, digamos, o vértice D tende ao vértice C do quadrilátero, o ponto
> O de intersecção das diagonais não tende ao ponto de Fermat do triângulo
> ABC pois este é o que tem a menor soma das distâncias aos vértices enquanto
> que O, no limite, minimiza a soma PA + PB + 2PC.
>
> Enviado do meu iPhone
>
> Em 11 de mar de 2018, à(s) 17:20, Douglas Oliveira de Lima <
> profdouglaso.del...@gmail.com> escreveu:
>
> Seja o quadrilátero ABCD cujas diagonais são AC e BD, e O o ponto de
> intersecção das diagonais.Â
> Seja também um ponto P em seu interior e as distâncias PA, PB, PC, PD,
> temos por desigualdade triângularÂ
> que PA+PC>=AC e PB+PD>=BD. Claramente vemos que o ponto P coincide com o
> ponto O quando a soma das diagonaisÂ
> coincide com a igualdade. Desta forma o ponto procurado é o encontro das
> diagonais.
>
>
> Forte abraço.
> Douglas Oliveira.Â
>
> Em 10 de março de 2018 21:07, Claudio Buffara 
> escreveu:
>
>> Aqui vai um bonitinho que eu nunca tinha visto:
>>
>> Dado um quadrilátero convexo, determine o ponto cuja soma das
>> distâncias aos vértices do quadrilátero é mínima.
>>
>> Interessante que quando a distância entre dois vértices adjacentes
>> dados tende a zero (e o quadrilátero “tende†a um triângulo), o ponto
>> de mínimo não parece tender ao ponto de Fermat do triângulo (exceto
>> quando o triângulo tem um ângulo >= 120 graus.
>>
>> Abs,
>> Claudio.
>>
>> Enviado do meu iPhone
>> --
>> Esta mensagem foi verificada pelo sistema de antivírus e
>> Â acredita-se estar livre de perigo.
>>
>>
>> =
>> Instru�ões para entrar na lista, sair da lista e usar a lista em
>> http://www.mat.puc-rio.br/~obmlistas/obm-l.html
>> =
>>
>
>
> --
> Esta mensagem foi verificada pelo sistema de antivírus e
> acredita-se estar livre de perigo.
>
>
> --
> Esta mensagem foi verificada pelo sistema de antivírus e
> acredita-se estar livre de perigo.
>

-- 
Esta mensagem foi verificada pelo sistema de antiv�rus e
 acredita-se estar livre de perigo.



Re: [obm-l] se 24 divide mn+1...

2018-02-08 Por tôpico Ralph Teixeira
Hm, tem que tomar cuidado com "primos" quando estamos tratando modulo n.
Por exemplo, no caso do 7 temos 7=1*7=3*5 mod 8, entao 7 nao eh "primo"
modulo 8 (no sentido de ter apenas uma fatoracao).

Isto dito, eh facil ver que as duas unicas decomposicoes de 7 modulo 8 sao
essas ali, e ambas somam 8, entao no final o que o Lucas disse funciona.

Abraco, Ralph.

2018-02-08 15:15 GMT-02:00 Lucas Reis :

> Se mn+1 é divisível por 3, mn deixa resto 2 quando é dividido por 3. Como
> 2 é primo (2=2*1) um dos números deve deixar resto 2 e o outro resto 1, e
> assim m+n deixa resto 0 na divisão por 3. O mesmo argumento vale pra mn+1
> na divisão por 8, e nesse caso o primo é 7. Como n+m é divisível por 8 e
> por 3, m+n é múltiplo de 24.
>
> Em 8 de fev de 2018 9:46 AM, "marcone augusto araújo borges" <
> marconeborge...@hotmail.com> escreveu:
>
>> para m e n naturais, se 24 divide mn+1, então 24 divide m+n?
>>
>> --
>> Esta mensagem foi verificada pelo sistema de antivírus e
>> acredita-se estar livre de perigo.
>>
>
> --
> Esta mensagem foi verificada pelo sistema de antivírus e
> acredita-se estar livre de perigo.
>

-- 
Esta mensagem foi verificada pelo sistema de antiv�rus e
 acredita-se estar livre de perigo.



[obm-l] Re: [obm-l] Aritmética

2018-02-05 Por tôpico Ralph Teixeira
Suponho que naturais aqui sejam {1,2,3,...}

Eu faria no braço mesmo. No que se segue, leia vírgulas como "ou":

mn+1 | 24
mn+1 = 2,3,4,6,8,12,24
mn = 1,2,3,5,7,11,23

Como todos esses são primos (exceto 1... mas não faz diferença) teremos
{m,n} ={1,1},{1,2},...

Então m+n=1+1,1+2,...,1+23, que são os divisores de 24 de novo.

Abraço, Ralph.

2018-02-05 9:45 GMT-02:00 Pedro Júnior :

> Sejam m e n números naturais. Prove que
>
> mn + 1 | 24 =
> >
> m + n | 24
> 
> .
>
> Agradecido.
>
> --
>
> Pedro Jerônimo S. de O. Júnior
>
> Professor de Matemática
>
> Geo João Pessoa – PB
>
> --
> Esta mensagem foi verificada pelo sistema de antivírus e
> acredita-se estar livre de perigo.

-- 
Esta mensagem foi verificada pelo sistema de antiv�rus e
 acredita-se estar livre de perigo.



[obm-l] Re: [obm-l] Re: [obm-l] Re: [obm-l] Re: [obm-l] Re: [obm-l] polinômios

2017-11-28 Por tôpico Ralph Teixeira
Pior que quando eu escrevei aquilo, eu pensei mesmo duas vezes se devia
explicar este detalhe... Mas, em vista de discussoes passadas (como esta
que voce citou), achei que podia passar batido e esperar alguem perguntar,
se fosse o caso... Tipo, recentemente, numa olimpiada dessas, houve uma
breve discussao desse tipo para fazer um criterio de correcao -- "vamos
tirar ponto se o cara nao argumentar porque o polinomio tem coeficientes
inteiros?" -- e lembro que a decisao foi: "nao, isso nao vale ponto no
criterio"... :D :D :D :D :D

(Tambem achei que alguem podia reclamar do "nao existem 4 inteiros
distintos cujo produto eh +-1, +-2"... mas essa eh bem mais engolivel,
acho.) :D

Abracos, Ralph. :D

2017-11-28 16:23 GMT-02:00 Carlos Nehab <carlos.ne...@gmail.com>:

> Oi, Mateus et alli
>
> Eu cutuquei o Ralph porque há tempos ele colocou exatamente essa sua
> explicação "vindo em defesa" de uma solução que eu havia postado de outro
> problema". Rsrsr.
> Achei importante explicitar esse detalhe pra galera.
>
> Grande abraço
> Nehab
>
>
> Em 28 de novembro de 2017 12:07, Matheus Secco <matheusse...@gmail.com>
> escreveu:
>
>> Para ver que Q(x), basta ver que (x-a)(x-b)(x-c)(x-d) tem coeficiente
>> lider 1 e ao fazer a divisão longa de P(x) por este polinomio com
>> coeficiente lider 1, não há riscos de introduzir frações.
>>
>> Abs,
>> Secco
>>
>> Em 28 de nov de 2017 11:58 AM, "Carlos Nehab" <carlos.ne...@gmail.com>
>> escreveu:
>>
>> Oi, Ralph
>>
>> E o detalhe que Q(x) tem coeficientes inteiros..., "exprica prá nóis"!
>>
>> Abraços
>> Nehab
>>
>> Em 27 de novembro de 2017 21:51, Ralph Teixeira <ralp...@gmail.com>
>> escreveu:
>>
>>> Acho que eles queriam 4 raizes inteiras distintas.
>>>
>>> Neste caso, temos P(x)=(x-a)(x-b)(x-c)(x-d)Q(x) onde Q(x) tem
>>> coeficientes inteiros e a,b,c,d sao as 4 raizes inteiras distintas.
>>>
>>> Se P(x)=2 tivesse raiz inteira, digamos, x=n, entao teriamos
>>> P(n)=(n-a)(n-b)(n-c)(n-d)Q(n)=2. Mas entao n-a, n-b, n-c e n-d seriam 4
>>> inteiros distintos cujo produto seria +-1 ou +-2, o que nao eh possivel.
>>>
>>> Abraco, Ralph.
>>>
>>> 2017-11-27 20:09 GMT-02:00 André Lauer <andre_la...@hotmail.com.br>:
>>>
>>>> Boa noite, preciso de ajuda no seguinte problema:
>>>> Um polinômio P(x) tem coeficientes inteiros e admite quatro raízes
>>>> inteiras. Prove que a equação P(x) = 2 não admite raízes inteiras.
>>>>
>>>> --
>>>> Esta mensagem foi verificada pelo sistema de antivírus e
>>>> acredita-se estar livre de perigo.
>>>>
>>>
>>>
>>> --
>>> Esta mensagem foi verificada pelo sistema de antivírus e
>>> acredita-se estar livre de perigo.
>>
>>
>>
>> --
>> Esta mensagem foi verificada pelo sistema de antivírus e
>> acredita-se estar livre de perigo.
>>
>>
>>
>> --
>> Esta mensagem foi verificada pelo sistema de antivírus e
>> acredita-se estar livre de perigo.
>
>
>
> --
> Esta mensagem foi verificada pelo sistema de antivírus e
> acredita-se estar livre de perigo.
>

-- 
Esta mensagem foi verificada pelo sistema de antiv�rus e
 acredita-se estar livre de perigo.



[obm-l] Re: [obm-l] polinômios

2017-11-27 Por tôpico Ralph Teixeira
Acho que eles queriam 4 raizes inteiras distintas.

Neste caso, temos P(x)=(x-a)(x-b)(x-c)(x-d)Q(x) onde Q(x) tem coeficientes
inteiros e a,b,c,d sao as 4 raizes inteiras distintas.

Se P(x)=2 tivesse raiz inteira, digamos, x=n, entao teriamos
P(n)=(n-a)(n-b)(n-c)(n-d)Q(n)=2. Mas entao n-a, n-b, n-c e n-d seriam 4
inteiros distintos cujo produto seria +-1 ou +-2, o que nao eh possivel.

Abraco, Ralph.

2017-11-27 20:09 GMT-02:00 André Lauer :

> Boa noite, preciso de ajuda no seguinte problema:
> Um polinômio P(x) tem coeficientes inteiros e admite quatro raízes
> inteiras. Prove que a equação P(x) = 2 não admite raízes inteiras.
>
> --
> Esta mensagem foi verificada pelo sistema de antivírus e
> acredita-se estar livre de perigo.
>

-- 
Esta mensagem foi verificada pelo sistema de antiv�rus e
 acredita-se estar livre de perigo.



[obm-l] Re: [obm-l] Lógica

2017-11-26 Por tôpico Ralph Teixeira
Ah, errei a ultima linha:

***Existem*** x e y tais que (y eh racional) e (x nao eh irracional).

O que funciona (por exemplo, com x=2).

:D

2017-11-26 21:16 GMT-02:00 Ralph Teixeira <ralp...@gmail.com>:

> Oi, Israel.
>
> Primeiro: a negacao de p=>q nao eh uma implicacao! A negacao de p => q eh:
> "p e nao q".
>
> Segundo: na grande maioria das implicacoes logicas, fica subentendido um
> "para todo" para as variaveis que estiverem livres... Se nao fosse assim,
> voce nao poderia julgar a frase, pois voce nao sabe o valor de x.
>
> Entao, no seu caso:
> (Para todos x,y) Se x eh racional, entao y eh irracional.
> (Para todos x,y) Se y eh racional, entao x eh irracional.
> sao ambas falsas, concordo com voce -- mas note que interpretei como "isto
> nao eh SEMPRE verdade", o que nao significa que eh SEMPRE FALSO.
>
> Agora sim, fazendo a negacao direitinmho, completa, voce tira a seguinte
> frase verdadeira:
>
> ***Existem*** x e y tais que (y eh racional) e (x nao eh irracional).
>
> O que funciona (o exemplo eh exatamente o seu x=1+sqrt(2))!
>
> Abraco, Ralph.
>
>
>
> 2017-11-26 20:28 GMT-02:00 Israel Meireles Chrisostomo <
> israelmchrisost...@gmail.com>:
>
>>
>> Seja y=x-1/x.A proposição: se x é racional então y é irracional, é uma
>> proposição claramente falsa.Mas se uma proposição é falsa, então sua
>> contra-positiva também é falsa.A contra-positiva dessa sentença é:  se y é
>> racional então x é irracional, e por ser a proposição contra-positiva da
>> primeira, então essa sentença também é falsa.Mas se essa sentença é falsa,
>> então se y é racional então x só pode ser racional.Agora veja que se
>> x=1+sqrt{2}, podemos ver claramente que y será racional, ou seja, o fato de
>> y ser racional não implicaria que x é racional.O que eu fiz de errado?
>> --
>> Israel Meireles Chrisostomo
>>
>> --
>> Esta mensagem foi verificada pelo sistema de antivírus e
>> acredita-se estar livre de perigo.
>
>
>

-- 
Esta mensagem foi verificada pelo sistema de antiv�rus e
 acredita-se estar livre de perigo.



[obm-l] Re: [obm-l] Lógica

2017-11-26 Por tôpico Ralph Teixeira
Oi, Israel.

Primeiro: a negacao de p=>q nao eh uma implicacao! A negacao de p => q eh:
"p e nao q".

Segundo: na grande maioria das implicacoes logicas, fica subentendido um
"para todo" para as variaveis que estiverem livres... Se nao fosse assim,
voce nao poderia julgar a frase, pois voce nao sabe o valor de x.

Entao, no seu caso:
(Para todos x,y) Se x eh racional, entao y eh irracional.
(Para todos x,y) Se y eh racional, entao x eh irracional.
sao ambas falsas, concordo com voce -- mas note que interpretei como "isto
nao eh SEMPRE verdade", o que nao significa que eh SEMPRE FALSO.

Agora sim, fazendo a negacao direitinmho, completa, voce tira a seguinte
frase verdadeira:

***Existem*** x e y tais que (y eh racional) e (x nao eh irracional).

O que funciona (o exemplo eh exatamente o seu x=1+sqrt(2))!

Abraco, Ralph.



2017-11-26 20:28 GMT-02:00 Israel Meireles Chrisostomo <
israelmchrisost...@gmail.com>:

>
> Seja y=x-1/x.A proposição: se x é racional então y é irracional, é uma
> proposição claramente falsa.Mas se uma proposição é falsa, então sua
> contra-positiva também é falsa.A contra-positiva dessa sentença é:  se y é
> racional então x é irracional, e por ser a proposição contra-positiva da
> primeira, então essa sentença também é falsa.Mas se essa sentença é falsa,
> então se y é racional então x só pode ser racional.Agora veja que se
> x=1+sqrt{2}, podemos ver claramente que y será racional, ou seja, o fato de
> y ser racional não implicaria que x é racional.O que eu fiz de errado?
> --
> Israel Meireles Chrisostomo
>
> --
> Esta mensagem foi verificada pelo sistema de antivírus e
> acredita-se estar livre de perigo.

-- 
Esta mensagem foi verificada pelo sistema de antiv�rus e
 acredita-se estar livre de perigo.



[obm-l] Re: [obm-l] Problema de álgebra

2017-09-15 Por tôpico Ralph Teixeira
Bom, suponho que queremos alguma solucao que nao use tecnicas de Calculo?

Que tal assim: x, y e z sao raizes do polinomio:

t^3-t^2+at-P=0

onde P eh o que voce quer maximizar.

O polinomio f(t)=t^3-t^2+at-P sempre tem pelo menos uma raiz real (grau 3).
Quando voce muda P, voce translada o grafico de f(t) para cima (ou para
baixo). Assim, a ideia eh levar o grafico para cima o maximo possivel,
mantendo sempre 3 raizes reais -- isto eh, o P maximo acontece quando temos
uma raiz dupla!

Assim, podemos supor spdg x=y=r no P maximo, e portanto z=1-2r. Jogue isso
em xy+xz+yz=a para descobrir esse r otimo em termos de a (confira que esse
r eh real, garantindo a existencia das 3 raizes de fato), e calcule
P=r.r.(1-2r) para descobrir o tal produto maximo.

Abraco, Ralph.


2017-09-15 15:13 GMT-03:00 Leonardo Joau :

> Dados os reais x, y,z, tais que:
>
> x+y+z = 1
>
> xy+xz+yz = a  0
> Calcule o max{xyz} em função de a.
>
>
> Att,
> Leonardo Joau
>
> --
> Esta mensagem foi verificada pelo sistema de antivírus e
> acredita-se estar livre de perigo.

-- 
Esta mensagem foi verificada pelo sistema de antiv�rus e
 acredita-se estar livre de perigo.



[obm-l] Re: [obm-l] Re: [obm-l] Re: [obm-l] Problema difícil.

2017-09-13 Por tôpico Ralph Teixeira
Oi, Douglas.

Acho que o que você fez é um bom começo.

Vamos adaptar: pense ao invés nos números de 1009 a 2017 (conjunto A).

i) Eles podem todos parear com os números de 1 a 1008?
ii) Então pelo menos um produto usando os elementos de A vai dar NO MÍNIMO
NO MÍNIMO...
iii) Esse número do item anterior, pode ser o máximo de todos eles? Como?

Abraço, Ralph.



2017-09-13 7:11 GMT-03:00 Douglas Oliveira de Lima <
profdouglaso.del...@gmail.com>:

> Então Bernardo, eu pensei numa parada mas não tenho certeza , pensei que
> os números 997,998,999,...,1994 Não poderiam ocupar as posições de 1 a
> 1997, logo pelo menos um deles ocuparia uma posição não inferior a 998, aí
> pensei no 997.998=995006.
>
> Em 12 de set de 2017 18:39, "Bernardo Freitas Paulo da Costa" <
> bernardo...@gmail.com> escreveu:
>
>> 2017-09-12 17:51 GMT-03:00 Douglas Oliveira de Lima
>> :
>> > Considere a sequência de números 1,2,3,4,5,...,2017.
>> > E uma certa ordenação deles a1, a2, a3, ..., a2017.
>> > Agora multiplique respectivamente os números das duas sequencias
>> > determinando assim uma nova sequência 1.a1
>> , 2.a2, 3.a3, ...,
>> 2017.a2017.
>> >
>> > Qual o menor valor que o maior produto da última sequência pode assumir?
>>
>> Esse problema não é tão difícil quanto parece.  O que você tentou fazer?
>>
>> Abraços,
>> --
>> Bernardo Freitas Paulo da Costa
>>
>> --
>> Esta mensagem foi verificada pelo sistema de antivírus e
>>  acredita-se estar livre de perigo.
>>
>>
>> =
>> Instru�ões para entrar na lista, sair da lista e usar a lista em
>> http://www.mat.puc-rio.br/~obmlistas/obm-l.html
>> =
>>
>
> --
> Esta mensagem foi verificada pelo sistema de antivírus e
> acredita-se estar livre de perigo.

-- 
Esta mensagem foi verificada pelo sistema de antiv�rus e
 acredita-se estar livre de perigo.



Re: [obm-l] Fibonacci teoria dos numeros

2017-09-05 Por tôpico Ralph Teixeira
P.S.: ah, agora que eu vi, o Anderson jah tinha resolvido essa exatamente
do mesmo jeito que eu.

2017-09-05 19:18 GMT-03:00 Ralph Teixeira <ralp...@gmail.com>:

> Bom, a gente pode olhar a sequencia de Fibonacci modulo n. Daqui para a
> frente, vamos fazer TUDO modulo n.
>
> Agora olhe para todos os pares (F_i,F_{i+1}). Ha apenas n^2 possibilidades
> para tais pares, portanto em algum momento eles tem de repetir. Seja (F_a,
> F_{a+1}) o par com o menor "a" possivel que repete depois na sequencia,
> quer dizer, F_a=F_b e F_{a+1}=F_{b+1} com b>a.
>
> Por "desinducao" finita, eu afirmo que a=1. Afinal caso contrario eu
> poderia olhar para F_{a-1}=F_{a+1}-F_a = F_{b+1}-F_b = F_{b-1}, e portanto
> (F_{a-1},F_a) tambem serviria!
>
> Mas entao F_{b-1}=F_{b+1}-F_b=F_2-F_1=1-1=0 mod n, ou seja, F_{b-1} eh
> divisivel por n.
>
> Abraco, Ralph.
>
> 2017-09-05 16:25 GMT-03:00 Pedro José <petroc...@gmail.com>:
>
>> Boa tarde!
>>
>> Douglas,
>>
>> esse problema você viu em algum local ou foi uma conjectura sua?
>>
>> Fiz em computador a sequência de Fibonacci mod 29791, o o F_14 = 377
>> mod29791 e F_15 = 610 mod2971 e F_28844 = 377 mod29791 e F_28845 = 610
>> mod29791, o que caracteriza que haverá um padrão de repetição na geração
>> dos números de Fibonacci mod 29791. E como não tem um elemento F_i = 0 mod
>> para i <28844, não haverá mais nenhum termo F_j = 0. Pois para qualquer j
>> >= 28844 haverá um i < 28844, onde F_j = F_i <>0 mod 29791, logo 29791, não
>> tem um único múltiplo na sequência de Fibonacci e portanto, a hipótese é
>> falsa.
>>
>> Deve ter um modo mais elegante para mostrar que a proposição é fasla.
>>
>> Sds,
>> PJMS
>>
>>
>>
>>
>>
>>
>> Em 5 de setembro de 2017 10:16, Pedro José <petroc...@gmail.com>
>> escreveu:
>>
>>> Bom dia!
>>>
>>> Eu pensei que entendera, porém, os números não são sequenciais.
>>> Se nós tivermos dois números consecutivos F_j e F_j+1  congruentes
>>> módulo m, pela lei de geração da sequencia de Fibonacci teremos que F_j-1 =
>>> 0 módulo m.
>>> O enunciado deveria ter uma restrição ... existe um número de Fibonacci, não
>>> nulo, que é múltiplo n, para evitar a corrente que considera zero como
>>> o primeiro termo da sequencia, pois, aí ficaria elementar a solução.
>>> Acho que o caminho é provar que como F_1 = F_2  ==> que para algum j
>>> : F_j = F_j+1 módulo m o que leva a F_j-1= 0 módulo m.
>>> Mas por casa de pombo só não daria, por exemplo, se consideramos A_1 = 3
>>> e A_2 = 7 e Ai = A_i-1 + A_i-2, não haveria um número múltiplo de 8.
>>> A sequência mod 8 ficaria: 3, 7, 2, 1, 3, 4, 7, 3, 2, 5, 7, 4, 3, 7, 2,
>>> 1, 3, 4, 7, 3, 2, 5, 7, 4...
>>>
>>> Saudações,
>>> PJMS
>>>
>>>
>>> Em 4 de setembro de 2017 16:48, Pedro José <petroc...@gmail.com>
>>> escreveu:
>>>
>>>> Boa tarde!
>>>>
>>>> Nehab,
>>>>
>>>> não consegui entender o restante da solução, mas ele usou o sinal de
>>>> igual para congruência por comodidade de edição, e até pela lei de formação
>>>> da sequência, só o segundo e terceiro termos são iguais, quando se admite
>>>> que comece de zero, ou os dois primeiros, para a corrente que não considera
>>>> como o primeiro termo da sequencia..
>>>> Por exemplo, 13 = 23 mod 10 mas (13, 23) = 1. Portanto, não fere o
>>>> princípio de que dois números consecutivos na sequência de Fibonacci sejam
>>>> primos entre si.
>>>> Até aí captei e entendi, pelo princípio da casa de pombos. Estou
>>>> tentando entender o restante.
>>>>
>>>> Saudações,
>>>> PJMS
>>>>
>>>> Em 4 de setembro de 2017 14:53, Carlos Nehab <carlos.ne...@gmail.com>
>>>> escreveu:
>>>>
>>>>> Oi, Douglas.
>>>>>
>>>>> Acho que o mdc entre Fibbonaccis consecutivos é sempre 1...
>>>>>
>>>>> Nehab
>>>>>
>>>>>
>>>>> <https://www.avast.com/sig-email?utm_medium=email_source=link_campaign=sig-email_content=webmail>
>>>>>  Livre
>>>>> de vírus. www.avast.com
>>>>> <https://www.avast.com/sig-email?utm_medium=email_source=link_campaign=sig-email_content=webmail>.
>>>>>
>>>>> <#m_-6736870374412433224_m_-7782833122447588826_m_4365848760417512581_m

Re: [obm-l] Fibonacci teoria dos numeros

2017-09-05 Por tôpico Ralph Teixeira
Bom, a gente pode olhar a sequencia de Fibonacci modulo n. Daqui para a
frente, vamos fazer TUDO modulo n.

Agora olhe para todos os pares (F_i,F_{i+1}). Ha apenas n^2 possibilidades
para tais pares, portanto em algum momento eles tem de repetir. Seja (F_a,
F_{a+1}) o par com o menor "a" possivel que repete depois na sequencia,
quer dizer, F_a=F_b e F_{a+1}=F_{b+1} com b>a.

Por "desinducao" finita, eu afirmo que a=1. Afinal caso contrario eu
poderia olhar para F_{a-1}=F_{a+1}-F_a = F_{b+1}-F_b = F_{b-1}, e portanto
(F_{a-1},F_a) tambem serviria!

Mas entao F_{b-1}=F_{b+1}-F_b=F_2-F_1=1-1=0 mod n, ou seja, F_{b-1} eh
divisivel por n.

Abraco, Ralph.

2017-09-05 16:25 GMT-03:00 Pedro José :

> Boa tarde!
>
> Douglas,
>
> esse problema você viu em algum local ou foi uma conjectura sua?
>
> Fiz em computador a sequência de Fibonacci mod 29791, o o F_14 = 377
> mod29791 e F_15 = 610 mod2971 e F_28844 = 377 mod29791 e F_28845 = 610
> mod29791, o que caracteriza que haverá um padrão de repetição na geração
> dos números de Fibonacci mod 29791. E como não tem um elemento F_i = 0 mod
> para i <28844, não haverá mais nenhum termo F_j = 0. Pois para qualquer j
> >= 28844 haverá um i < 28844, onde F_j = F_i <>0 mod 29791, logo 29791, não
> tem um único múltiplo na sequência de Fibonacci e portanto, a hipótese é
> falsa.
>
> Deve ter um modo mais elegante para mostrar que a proposição é fasla.
>
> Sds,
> PJMS
>
>
>
>
>
>
> Em 5 de setembro de 2017 10:16, Pedro José  escreveu:
>
>> Bom dia!
>>
>> Eu pensei que entendera, porém, os números não são sequenciais.
>> Se nós tivermos dois números consecutivos F_j e F_j+1  congruentes módulo
>> m, pela lei de geração da sequencia de Fibonacci teremos que F_j-1 = 0
>> módulo m.
>> O enunciado deveria ter uma restrição ... existe um número de Fibonacci, não
>> nulo, que é múltiplo n, para evitar a corrente que considera zero como o
>> primeiro termo da sequencia, pois, aí ficaria elementar a solução.
>> Acho que o caminho é provar que como F_1 = F_2  ==> que para algum j
>> : F_j = F_j+1 módulo m o que leva a F_j-1= 0 módulo m.
>> Mas por casa de pombo só não daria, por exemplo, se consideramos A_1 = 3
>> e A_2 = 7 e Ai = A_i-1 + A_i-2, não haveria um número múltiplo de 8.
>> A sequência mod 8 ficaria: 3, 7, 2, 1, 3, 4, 7, 3, 2, 5, 7, 4, 3, 7, 2,
>> 1, 3, 4, 7, 3, 2, 5, 7, 4...
>>
>> Saudações,
>> PJMS
>>
>>
>> Em 4 de setembro de 2017 16:48, Pedro José 
>> escreveu:
>>
>>> Boa tarde!
>>>
>>> Nehab,
>>>
>>> não consegui entender o restante da solução, mas ele usou o sinal de
>>> igual para congruência por comodidade de edição, e até pela lei de formação
>>> da sequência, só o segundo e terceiro termos são iguais, quando se admite
>>> que comece de zero, ou os dois primeiros, para a corrente que não considera
>>> como o primeiro termo da sequencia..
>>> Por exemplo, 13 = 23 mod 10 mas (13, 23) = 1. Portanto, não fere o
>>> princípio de que dois números consecutivos na sequência de Fibonacci sejam
>>> primos entre si.
>>> Até aí captei e entendi, pelo princípio da casa de pombos. Estou
>>> tentando entender o restante.
>>>
>>> Saudações,
>>> PJMS
>>>
>>> Em 4 de setembro de 2017 14:53, Carlos Nehab 
>>> escreveu:
>>>
 Oi, Douglas.

 Acho que o mdc entre Fibbonaccis consecutivos é sempre 1...

 Nehab


 
  Livre
 de vírus. www.avast.com
 .

 <#m_-7782833122447588826_m_4365848760417512581_m_408333944165922029_m_4928629599140568768_m_2006884623661450834_DAB4FAD8-2DD7-40BB-A1B8-4E2AA1F9FDF2>

 Em 4 de setembro de 2017 07:24, Anderson Torres <
 torres.anderson...@gmail.com> escreveu:

> Em 31 de agosto de 2017 16:30, Douglas Oliveira de Lima
>  escreveu:
> > Olá, como posso mostrar que para algum inteiro e positivo n, existe
> um
> > número de Fibonacci que é múltiplo de n?
>
> Casa dos Pombos! Maybe?
>
> Bem, pegue os pares de Fibonaccis consecutivos, (F0, F1), (F1, F2),
> (F2, F5),... módulo M.
>
> Por PCP, dois deles , digamos (Fk, F(k+1)) e (F(k+j),F(k+j+1)) serão
> iguais.
>
> Assim, Fk=F(k+j) e F(k+1)=F(k+j+1).
>
> Mas aí, F(k+1)-F(k)=F(k+j+1)-F(k+j) e portanto F(k-1) = F(k+j-1).
>
> Prosseguindo dessa forma, chegaremos em F(j)=F(0)=0.
>
>
>
> >
> > Douglas Oliveira.
> >
> >
> > --
> > Esta mensagem foi verificada pelo sistema de antivírus e
> > acredita-se estar livre de perigo.
>
> --
> Esta mensagem foi verificada pelo sistema de antivírus e
>  acredita-se estar livre de perigo.
>
>
> 
> 

[obm-l] Re: [obm-l] Geometria Analítica

2017-08-22 Por tôpico Ralph Teixeira
Usando Geometria: seja M o ponto medio de AB. Note que M eh fixo.

O Teorema de Apolonio 
diz que

PA^2+PB^2 = 2(PM^2+a^2)

(obs: isso vale mesmo que P esteja na reta AB). Entao PM^2=k^2/2 - a^2 eh
fixo. Assim, tipicamente o lugar geometrico de P eh um circulo de centro M
e raio quadrado k^2/2 - a^2...

Digo "tipicamente" porque temos que analisar se esse raio existe mesmo...
Entao:
a) Se k^2<2a^2, entao o L.G. serah vazio
b) Se k^2=2a^2, entao o L.G. serah apenas o ponto M.
c) Se k^2>2a^2, entao realmente dah aquele circulo que eu citei -- mas
tecnicamente tem que ver se os pontos onde esse circulo corta a reta AB
tambem servem, porque PAB nao seria tecnicamente um triangulo (resposta:
sim, servem!).

Usando Vetores: (uso  para produto interno)
+=k^2
2-2-2++=k^2
-=(k^2--)/2
Agora complete quadrados
-2+<(A+B)/2,(A+B)/2> = (k^2
--)/2+<(A+B)/2,(A+B)/2>
 = k^2/2  -<(A-B)/2,(A-B)/2> = k^2/2 - a^2
||P - (A+B)/2|| ^ 2 = k^2/2 - a^2
Ou seja, a distancia de P a M=(A+B)/2 eh fixa e igual a k^2/2-a^2

Abraco, Ralph.

2017-08-22 19:31 GMT-03:00 André Lauer :

> Boa noite, preciso de ajuda no seguinte problema:
> São dados dois pontos A e B. Determine o lugar geométrico de P tal que
> d(A,P)^2 + d(P,B)^2 = k^2 onde k é uma constante dada.
> Se d(A,B) = 2a, determine para que valores de k o problema tem solução.
>
> --
> Esta mensagem foi verificada pelo sistema de antivírus e
> acredita-se estar livre de perigo.
>

-- 
Esta mensagem foi verificada pelo sistema de antiv�rus e
 acredita-se estar livre de perigo.



Re: [obm-l] Unicidade do sucessor (Axiomas de Peano)

2017-08-18 Por tôpico Ralph Teixeira
Oi, Pedro.

Como está, não podemos provar. Você poderia ter P={0,1,1',2,2',3,3',...}
onde os sucessores de 0 são ambos 1 e 1'; o sucessor de n é (n+1); e o
sucessor de n' é (n+1)'. Este conjunto P satisfaz todos os axiomas que você
colocou.

Acho que o axioma (2) costuma INCLUIR o fato de que todo número tem um
ÚNICO sucessor, ou seja, que "sucessor" é, de fato, uma **função**.

Abraço, Ralph.

2017-08-18 10:59 GMT-03:00 Pedro Chaves :

> Caros Colegas,
> Vamos admitir que as quatro afirmações seguintes sejam os quatro primeiros
> axiomas de Peano, restando apenas o quinto (princípio de indução
> matemática).
> 1) Zero é um número natural.
> 2) Todo número natural tem um sucessor, que também é um número natural.
> 3) Zero não é sucessor de nenhum número natural.
> 4) Dois números naturais que têm sucessores iguais são, eles próprios,
> iguais.
>
> Sejam s e t sucessores do natural n.  Como podemos provar que s = t?
> Isto é, como podemos provar, usando somente os quatro axiomas acima, que
> todo número natural tem um único sucessor?
>
> Abraços.
> Pedro Chaves
> --
>
>
> --
> Esta mensagem foi verificada pelo sistema de antivírus e
> acredita-se estar livre de perigo.
>

-- 
Esta mensagem foi verificada pelo sistema de antiv�rus e
 acredita-se estar livre de perigo.



[obm-l] Re: [obm-l] Re: [obm-l] Re: [obm-l] Construção Geométrica

2017-08-14 Por tôpico Ralph Teixeira
Ah, bem observado! De fato, eu SUPUS que CD corta OA e OB, o que nao estava
explicito no problema.

Caso CD nao corte OA e OB, serah que a resposta eh mesmo o triangulo
degenerado POP?

Abraco, Ralph.

2017-08-14 5:43 GMT-03:00 Rogerio Ignacio <rogerioigna...@gmail.com>:

> Observo que,, nas condições do problema, med(Ô) < 90º
>
> Em 13 de agosto de 2017 21:50, Ralph Teixeira <ralp...@gmail.com>
> escreveu:
>
>> Sejam C e D os simetricos de P com relacao a OA e OB, respectivamente.
>>
>> Dados pontos X e Y quaisquer em OA e OB, note que o perimetro do
>> triangulo PXY serah:
>>
>> PX+XY+YP = CX + XY + YD
>>
>> Mas CX+XY+YD<=CD, com igualdade se e somente se C,X,Y e D estao em linha
>> reta. Entao a solucao eh usar os pontos X e Y onde a reta CD corta OA e OB,
>> respectivamente.
>>
>> Abraco, Ralph.
>>
>>
>>
>> 2017-08-13 18:54 GMT-03:00 Marcelo de Moura Costa <mat.mo...@gmail.com>:
>>
>>>
>>> Boa noite a todos,
>>>
>>> Estou com o seguinte problema de construção geométrica, proposto pelo
>>> programa Euclidea (adaptei o enunciado):
>>>
>>> Dado um ângulo AOB, e um ponto P interno ao ângulo, construa um
>>> triângulo com vértice em P e nas semirretas do ângulo OA e OB de maneira
>>> que o perímetro seja mínimo.
>>>
>>> Já pensei na solução de Heron para o problema dos dois pontos do mesmo
>>> lado da reta, mas não saiu nada.
>>>
>>> Agradeceria muito a atenção dos colegas.
>>>
>>> Abraços
>>>
>>>
>>> <https://www.avast.com/sig-email?utm_medium=email_source=link_campaign=sig-email_content=webmail>
>>>  Livre
>>> de vírus. www.avast.com
>>> <https://www.avast.com/sig-email?utm_medium=email_source=link_campaign=sig-email_content=webmail>.
>>>
>>> <#m_-5025016543957464356_m_-8626142748963422830_m_1890556287314091725_DAB4FAD8-2DD7-40BB-A1B8-4E2AA1F9FDF2>
>>>
>>> --
>>> Esta mensagem foi verificada pelo sistema de antivírus e
>>> acredita-se estar livre de perigo.
>>
>>
>>
>> --
>> Esta mensagem foi verificada pelo sistema de antivírus e
>> acredita-se estar livre de perigo.
>
>
>
> --
> Esta mensagem foi verificada pelo sistema de antivírus e
> acredita-se estar livre de perigo.

-- 
Esta mensagem foi verificada pelo sistema de antiv�rus e
 acredita-se estar livre de perigo.



[obm-l] Re: [obm-l] Construção Geométrica

2017-08-13 Por tôpico Ralph Teixeira
Sejam C e D os simetricos de P com relacao a OA e OB, respectivamente.

Dados pontos X e Y quaisquer em OA e OB, note que o perimetro do triangulo
PXY serah:

PX+XY+YP = CX + XY + YD

Mas CX+XY+YD<=CD, com igualdade se e somente se C,X,Y e D estao em linha
reta. Entao a solucao eh usar os pontos X e Y onde a reta CD corta OA e OB,
respectivamente.

Abraco, Ralph.



2017-08-13 18:54 GMT-03:00 Marcelo de Moura Costa :

>
> Boa noite a todos,
>
> Estou com o seguinte problema de construção geométrica, proposto pelo
> programa Euclidea (adaptei o enunciado):
>
> Dado um ângulo AOB, e um ponto P interno ao ângulo, construa um triângulo
> com vértice em P e nas semirretas do ângulo OA e OB de maneira que o
> perímetro seja mínimo.
>
> Já pensei na solução de Heron para o problema dos dois pontos do mesmo
> lado da reta, mas não saiu nada.
>
> Agradeceria muito a atenção dos colegas.
>
> Abraços
>
>
> 
>  Livre
> de vírus. www.avast.com
> .
> <#m_1890556287314091725_DAB4FAD8-2DD7-40BB-A1B8-4E2AA1F9FDF2>
>
> --
> Esta mensagem foi verificada pelo sistema de antivírus e
> acredita-se estar livre de perigo.

-- 
Esta mensagem foi verificada pelo sistema de antiv�rus e
 acredita-se estar livre de perigo.



Re: [obm-l] Problema de Probabilidade

2017-08-08 Por tôpico Ralph Teixeira
Ah, se voce preferir, pode dividir a tabela por jogador mesmo, assim:

/// A B CD E FG  Total
JV   60   60   60   60   45   45   25  355
JP   40   40   40   40   55   55   75  345
Tot 100 100 100 100 100 100 100  700

a) Pr(JV)=355/700
b) Pr(E|JV)=45/355

Abraco, Ralph.

2017-08-08 11:26 GMT-03:00 Ralph Teixeira <ralp...@gmail.com>:

> Este problema sai formalmente usando a Regra de Bayes Mas eu sempre
> achei que, quando o problema eh pequeno, fica muito mais facil de entender
> o que estah havendo e resolver varios itens usando usando uma tabela.
>
> (Obs.: antes que alguem critique: minha tabela NAO reflete o que VAI
> acontecer quando jogarmos x jogos; eh apenas uma tabela CUJAS PROPORCOES
> sao identicas aas probabilidades, e que portanto pode ser usada para
> calcular qualquer probabilidade condicional.)
>
> Para economizar bits, vou denotar alguns eventos assim:
>
> P1: evento "o adversario veio do grupo 1 {A,B,C,D}"
> P2: evento "o adversario veio do grupo 2 {E,F}"
> P3: evento "o adversario foi G"
> JV: J vence seu jogo
> JP: J perde seu jogo
>
> Entao, vou supor 700 jogos no total e usar que 4/7 deste vao para P1, 2/7
> para P2 e 1/7 para P3 (suponho que "selecionado aleatoriamente" signifique
> "uniformemente"):
>
> ///  P1   P2   P3   Tot
> JV
> JP
> Tot 400 200 100 700
>
> (Obs.2: 700 eh um numero arbitrario para as contas ficarem redondas; use
> qualquer outra coisa se desejar, nao importa, pois vamos fazer apenas
> proporcoes mesmo.)
>
> Agora vamos usar as condicionais dadas: Pr(JV|P1)=0,6, por exemplo. Isto
> significa que, daqueles 400 jogos em que o adversario vem de P1, J vence
> 0,6*400=240 deles. Analogamente, Pr(JV|P2)=0,45 e Pr(JV|P3)=0,25. Assim,
> completo a tabela:
>
> ///  P1P2 P3   Tot
> JV 240   90 25  355
> JP 160  11075  345
> Tot 400  200  100 700
>
> Agora eh muito facil responder QUALQUER coisa. Vejamos:
>
> a) Queremos Pr(JV). Temos da tabela Pr(JV)=355/700
> b) Queremos Pr(P2|JV), ou quase isso. Bom, SABENDO que J venceu, estamos
> na linha 1, estamos nos restringindo a algum daqueles 355 jogos. Neste
> caso, a probabilidade do jogador ter vindo do grupo 2 seria:
> Pr(P2 | JV) = 90/355
> Entao a resposta eh 45/355 (pois ha 2 jogadores no grupo 2, igualmente
> provaveis)
>
> Abraco, Ralph.
>
> 2017-08-08 10:21 GMT-03:00 Luiz Antonio Rodrigues <rodrigue...@gmail.com>:
>
>> Olá, pessoal!
>> Bom dia!
>> Será que alguém pode me ajudar com o problema abaixo? Estou quebrando a
>> cabeça e não consigo resolvê-lo.
>> Muito obrigado e um abraço!
>> Luiz
>>
>> Um jogador J entra em um torneio de tênis com jogos eliminatórios. Seu
>> primeiro adversário será selecionado aleatoriamente a partir de um conjunto
>> de 7 jogadores: {A,B,C,D,E,F,G}. Contra 4 adversários (A,B,C,D) desse
>> conjunto, a probabilidade de vitória de J é 0,6; contra dois adversários
>> desse conjunto (E,F), a probabilidade de vitória de J é 0,45 e contra o
>> adversário restante (G), a probabilidade de vitória de J é 0,25.
>> a) Qual a probabilidade de vitória de J na primeira partida do torneio?
>>
>>
>> b) Suponha que a primeira partida já tenha sido realizada. Você fica
>> sabendo que J venceu esse jogo. Qual a probabilidade de que J tenha jogado
>> contra E?
>>
>>
>>
>> --
>> Esta mensagem foi verificada pelo sistema de antivírus e
>> acredita-se estar livre de perigo.
>
>
>

-- 
Esta mensagem foi verificada pelo sistema de antiv�rus e
 acredita-se estar livre de perigo.



[obm-l] Re: [obm-l] Re: [obm-l] Polinômios

2017-07-25 Por tôpico Ralph Teixeira
Vou ajeitar a ideia do Bruno, que eh muito boa -- vou botar um parametro
arbitrario na frente do primeiro polinomio:

Entao, crio P(x)=k(x-2)(x-3)(x-4) -> P(1)=-6k (onde k<>0)

Entao R(x)=k(x-2)(x-3)(x-4)+6k eh tal que R(1)=0; mais ainda,
R(2)=R(3)=R(4)=6k, portanto R(x) deixa o mesmo resto 6k na divisao por
(x-2), (x-3) ou (x-4).

(Ou entao, pegue o polinomio Q(x) do Bruno, e multiplique por uma constante
real arbitraria k<>0)

Abraco, Ralph.

2017-07-25 21:41 GMT-03:00 Bruno Visnadi :

> Seja P(x) = (x-2)(x-3)(x-4) = x³ - 9x² + 26x - 24 -> P(1) = -6
>
> Seja Q(x) = P(x) + 6 -> Q(1) = 0 -> Q(x) é múltiplo de (x-1)
>
> Perceba que Q(x) deixa resto 6 por (x-2), (x-3) e (x-4). Todo polinômio no
> formato Q(x) + n*P(x), para todo n, deixa resto 6 por
> (x-2), (x-3) e (x-4).
>
> Em 25 de julho de 2017 21:22, Pedro Júnior 
> escreveu:
>
>> Prove que existem infinitos polinômios de grau 3 de coeficientes reais
>> que são divisíveis por x - 1 e que deixam o mesmo resto por x - 2, x - 3 e
>> x - 4.
>>
>> Quem tiver uma boa dica fica meus agradecimentos.
>>
>> --
>> Esta mensagem foi verificada pelo sistema de antivírus e
>> acredita-se estar livre de perigo.
>
>
>
> --
> Esta mensagem foi verificada pelo sistema de antivírus e
> acredita-se estar livre de perigo.
>

-- 
Esta mensagem foi verificada pelo sistema de antiv�rus e
 acredita-se estar livre de perigo.



Re: [obm-l] Problema estranho

2017-07-11 Por tôpico Ralph Teixeira
Ah, melhor ainda: depois que seus números forem inteiros, some uma certa
constante a todos eles de forma que um deles seja 0. Agora divida por 2,
quantas vezes você quiser (eles vão ser sempre todos pares pelo argumento
de paridade anterior!). Então são todos inteiros divisíveis por poências
arbitrariamente grandes de 2 Pode isso, Arnaldo? Bom, pode, mas só tem
um jeito -- são todos 0.

2017-07-11 18:01 GMT-03:00 Ralph Teixeira <ralp...@gmail.com>:

> Bom, eu sei resolver se todos os números forem racionais. Deve ter um
> jeito de usar isso para o caso geral...
>
> A propriedade desse conjunto não se altera se todos os elementos do
> conjunto forem multiplicados por um mesmo número, nem se a gente somar uma
> certa constante a todos eles.
>
> Assim, *SE* eles forem todos racionais, a gente pode multiplicar todos
> eles por um m.m.c imenso e supor que são todos inteiros, spdg.
>
> Mas então todos teriam que ter a mesma paridade -- afinal a soma de todos
> eles, menos qualquer um deles, é um número par.
>
> Então, enquanto todos forem pares, divida-os por 2; em algum momento,
> **todos** ficarão ímpares. Quando isso acontecer, some 1, e ficam todos
> pares. Então divida por 2 de novo, e de novo, até ficarem ímpares, então
> some 1 de novo, repita e enxágue.
>
> Esse processo vai parar? Oras, esses inteiros vão diminuir em módulo
> até até até cada um deles virar 0, ou 1! De fato, |x|/2<|x| quando
> x<>0, e |x+1|/2 < |x| para x<>0,1. Então a cada um ou dois passos o valor
> absoluto de todos eles diminui -- a menos que eles sejam 0 ou 1. Ou seja,
> em tempo finito, todos eles vão virar 0 ou 1.
>
> Agora é fácil -- lembra que todos sempre têm a mesma paridade?? Então são
> todos 0, ou todos 1.
>
> ---///---
>
> Para o caso geral, tenho uma ideia, mas não estou com tempo de
> desenvolvê-la -- será que dá para começar com os reais, e multiplicar todos
> eles por algum número real imenso de forma que eles sejam quase inteiros
> (tipo, todos eles a menos de 1/(4n) de algum inteiro)? Talvez dê para
> mostrar então pela propriedade que eles têm que ser inteiros, ou pelo menos
> "comensuráveis" e daí matar o problema.
>
> Abraço, Ralph.
>
>
>
> 2017-07-11 15:06 GMT-03:00 Nowras Ali <nowras@gmail.com>:
>
>> Uma prova por indução me parece o melhor caminho.
>> O Bernardo já provou para o caso base, basta agora tentar
>> provar para n+1, assumindo verdadeiro para n. Tentarei resolver
>> o problema assim que puder.
>>
>> Abraços, Nowras.
>>
>> Em 9 de julho de 2017 18:54, Otávio Araújo <otavio17.ara...@gmail.com>
>> escreveu:
>>
>>>
>>> Já tentei isso, porém não parece ajudar em muita coisa  mas de
>>> qualquer forma obrigado
>>>
>>> > Em 9 de jul de 2017, às 18:00, Bernardo Freitas Paulo da Costa <
>>> bernardo...@gmail.com> escreveu:
>>> >
>>> > Não pensei muito, mas acho que você deveria tentar provar os casos
>>> n=1
>>> > e n=2 "no braço" para ter a intuição.  E, na verdade, o enunciado
>>> > deveria ser: dados a_1, a_2, ... a_{2n+1} números reais, não
>>> > necessariamente distintos, tais que, para cada escolha de 2n dentre
>>> > eles, é possível separar em dois grupos de n cada, com a mesma soma.
>>> > (evitando falar de conjuntos, você pode ter à vontade os elementos
>>> > repetidos).
>>> >
>>> > Assim, o caso n=1 fica: temos a_1, a_2, a_3.  Tomando os elementos
>>> > a_1, a_2, é possĩvel separar em dois grupos de um elemento, com a
>>> soma
>>> > igual.  Logo a_1 = a_2.  Por simetria, a_1 = a_3, e acabou.  Para n=2,
>>> > dá mais trabalho.
>>> >
>>> > 2017-07-08 23:20 GMT+03:00 Otávio Araújo <otavio17.ara...@gmail.com
>>> >:
>>> >> Galera, queria que alguém pudesse resolver essa questão pra mim
>>> ( passei muito tempo nela já kkk):
>>> >> " Seja n um natural positivo e A um conjunto de 2n+1 números
>>> reais, não necessariamente distintos, com a seguinte propriedade:
>>> >> - Todo subconjunto de A com 2n elementos pode ser particionado em
>>> dois conjuntos de n elementos tais que a soma dos elementos de cada um
>>> desses dois conjuntos de n elementos são iguais.
>>> >>   Prove que todos os elementos de A são iguais."
>>> >>
>>> >>
>>> >>
>>> >>
>>> >>
>>> >>
>>> >>
>>> >> --
>>> >> Esta mensagem foi 

  1   2   3   4   5   6   7   8   >